MCQs

¡Supera tus tareas y exámenes ahora con Quizwiz!

A brick mason was hired by a builder under a written one-year contract, at an annual salary of $45,000, with employment to begin on March 1. Because the builder was unable to secure enough building contracts to keep all its employees busy during the season beginning March 1, it notified the brick mason on February 15 that it could not afford to employ him as a mason. At the same time, however, the builder offered to employ the mason, for the same contract period, as a night guard at an annual salary of $25,000. The mason declined the offer and remained unemployed during the year. No employment for brick masons was available in the community during the year, but the mason could have obtained other employment as a day laborer that would have paid up to $25,000 a year. At the end of the year, in an action against the builder for breach of contract, how much, if anything, is the mason entitled to recover? A$20,000 (the $45,000 contract price less the $25,000 the mason could have earned in other employment). B$45,000 (the contract price). CNothing, because the builder did not act in bad faith when it discharged the mason. DNothing, because the mason did not mitigate his damages.

$45,000 (the contract price). Discussion of correct answer:(B) is correct. This question involves the principle of mitigation of damages. Upon a breach of contract, the aggrieved party may not recover for any losses it could have reasonably avoided. In the employment context, an aggrieved employee need only make reasonable efforts to secure a position that is reasonably equivalent to the job lost and, accordingly, she need not accept substitute work when it is in a different field, or offers significantly lower pay or less desirable terms and conditions of employment than those of the lost job. In this case, the brick mason was not required to take the job of either night guard or day laborer, which paid substantially less than his brick mason job. Therefore, he may recover the full $45,000 contract price in damages. (A) is therefore incorrect because the $25,000 in other, lesser employment will not reduce the brick mason's recovery. (C) is wrong because the builder's lack of bad faith will not prevent the brick mason from recovering for the damages causes by the builder's breach of contract. Finally, (D) is wrong because, as discussed above, the brick mason was not required to take the lesser employment offered in order to mitigate his damages.

A man was admitted to a hospital after complaining of persistent severe headaches. While he was there, hospital staff failed to diagnose his condition, and he was discharged. Two days later, the man died of a massive brain hemorrhage due to a congenital defect in an artery. The man's wife has brought a wrongful death action against the hospital. The wife offers expert testimony that the man would have had a "reasonable chance" (not greater than 50%) of surviving the hemorrhage if he had been given appropriate medical care at the hospital. In what type of jurisdiction would the wife's suit most likely be successful? AA jurisdiction that applies traditional common law rules concerning burden of proof. BA jurisdiction that allows recovery based on strict liability. CA jurisdiction that allows recovery for the loss of the chance of survival. DA jurisdiction that recognizes loss of spousal consortium.

A jurisdiction that allows recovery for the loss of the chance of survival. Discussion of correct answer:Jurisdictions that allow recovery for the loss of the chance of survival have created an exception to the traditional common law rules for establishing cause in fact. Under the traditional rules, the wife would be required to prove that reasonable action on the part of the hospital (presumably a correct diagnosis) would, more likely than not, have led to the man's survival. Here, the wife cannot establish that the chances of the man's survival would have been greater than 50% even if he had been given appropriate medical care. A jurisdiction that allows recovery for loss of the chance of survival, however, would allow the wife to recover for the reduction in her husband's chance of surviving that was caused by the failure to properly diagnose.

A homeowner sued a contractor over a dispute involving an addition to his home, but lost his case in a nonjury trial. Two months later, the homeowner discovered new evidence that could not have been discovered beforehand, even with reasonable diligence. Which of the following motions could the homeowner make to attempt to have the court rehear his claim? AA renewed motion for judgment as a matter of law. BA motion for relief from a judgment or order. CA motion for a new trial. DA motion for a judgment as a matter of law.

A motion for relief from a judgment or order. Discussion of correct answer:Under Rule 60(b)(2), on motion and on just terms, the court may relieve a party or its legal representative from a final judgment, order, or proceeding when there is newly discovered evidence that, with reasonable diligence, could not have been discovered in time to move for a new trial under Rule 59(b). Because this was a nonjury trial, a motion for a new trial would have to come under Rule 59(b), which has a 28-day time limit. The evidence here was discovered after two months. As such, the homeowner's only option would be to file a motion for relief from a judgment or order.

In what situation would a motion to dismiss under Fed. R. Civ. P. 12(b)(6) be appropriate? AA bartender filed a lawsuit against a man for assault, but failed to join another man involved in making the threats. BA pedestrian filed a lawsuit against a driver, but failed to serve the driver within 90 days of filing the complaint. CA woman filed a lawsuit against her neighbor alleging that he had engaged in harassment to try and drive her to move out, but failed to state any specific facts. DAn employee filed a lawsuit against his employer, but failed to engage in any discovery or otherwise move the case forward in 18 months.

A woman filed a lawsuit against her neighbor alleging that he had engaged in harassment to try and drive her to move out, but failed to state any specific facts. Discussion of correct answer:Pleadings must contain "a short and plain statement of the claim showing that the pleader is entitled to relief" [Fed. R. Civ. P. 8(a)(2)]. Under Rule 12(b)(6), if this requirement is not met, the party against whom the claim is alleged may move to dismiss for failure to state a claim for which relief can be granted [Fed. R. Civ. P. 12(b)(6)]. A pleading that states a claim for relief must contain [Fed. R. Civ. P. 8]: (1) a short and plain statement of the grounds upon which the court's jurisdiction rests and a statement of a claim, which, if true, would entitle the claimant to relief; and (2) a demand of relief. Importantly, the "short and plain statement of the claim" must include "sufficient factual matter" to state a plausible claim that is more than a "sheer possibility that a defendant has acted unlawfully" [Bell Atlantic Corp. v. Twombly, 550 U.S. 544 (2007); Ashcroft v. Iqbal, 556 U.S. 662 (2009)]. Therefore, if the woman failed to state any facts, a Rule 12(b)(6) motion would be appropriate.

A student at a private university sued the university in federal court for negligence after he fell from scaffolding in a university-owned theater building. At trial, after briefing from both parties, the court permitted the jury to hear testimony that there had been several previous accidents in the same building. The jury found for the student, and the university appealed. One of the university's arguments on appeal is that the testimony about the previous accidents should have been excluded as irrelevant and highly prejudicial. Which standard of review applies to this argument? AAbuse of discretion. BClearly erroneous. CDe novo. DHarmless error.

Abuse of discretion. Discussion of correct answer:A determination as to whether evidence is irrelevant or highly prejudicial and should be excluded is within the trial court's discretion because it requires an understanding of the entire case and the factual context in which the evidence is being offered. Therefore, it is reviewed on appeal using an abuse-of-discretion standard.

A construction worker sued an insulation manufacturer in federal court, claiming that he had developed a chronic health condition as a result of 20 years of exposure to the manufacturer's insulation at his work sites. The manufacturer answered, denying all liability and stating that it had never supplied its insulation to the worker's employer. The worker's attorney deposed the manufacturer's president, and the manufacturer's attorney deposed the worker. Immediately thereafter, the manufacturer moved for summary judgment on the ground that the worker had no evidence showing that the insulation had ever been used by the worker's employer. What would be the worker's best response to the motion for summary judgment? AArgue that more time is needed for additional discovery to show the manufacturer's liability, and attach a declaration describing the desired discovery. BArgue that the motion should be denied, because a central issue in the case will be the manufacturer's credibility on the question of its distribution of the insulation, and only a jury can decide questions of credibility. CArgue that the motion should be denied, because the manufacturer failed to attach any evidence to its motion to show that the insulation was not used by the worker's employer. DMake a cross-motion for summary judgment arguing that the manufacturer has introduced no evidence to show that its insulation did not harm the worker.

Argue that more time is needed for additional discovery to show the manufacturer's liability, and attach a declaration describing the desired discovery. Discussion of correct answer:Answer (A) is correct. "If a nonmovant shows by affidavit or declaration that, for specified reasons, it cannot present facts essential to justify its opposition, the court may: (1) defer considering the motion or deny it; (2) allow time to obtain affidavits or declarations or to take discovery; or (3) issue any other appropriate order" [Fed. R. Civ. P. 56(d)]. Here, there have only been two depositions, so the motion for summary judgment is likely premature.

Under the authority of a federal voting rights statute, some states drew congressional districts in a manner calculated to increase the likelihood that members of historically disadvantaged minority racial groups would be elected. The U.S. Supreme Court declared these districts to be unconstitutional, as improper racial gerrymanders. In response to this ruling, Congress passed a new statute that explicitly denies the Supreme Court appellate jurisdiction over all future cases challenging the constitutionality of action taken under the authority of the federal voting rights statute. Which of the following is the most persuasive argument for the constitutionality of the new statute restricting the Supreme Court's appellate jurisdiction? AArticle III of the Constitution explicitly states that the Supreme Court's appellate jurisdiction is subject to such exceptions and regulations as Congress shall make. BThe constitutional principle of separation of powers authorizes Congress to pass statutes calculated to reduce the effects of Supreme Court decisions that interfere with the exercise of powers that have been delegated to the legislative branch. CThe establishment and apportionment of congressional districts directly affect interstate commerce, and the Constitution authorizes Congress to use its plenary authority over such commerce for any purpose it believes will promote the general welfare. DThe Fifteenth Amendment authorizes Congress to enforce the amendment's voting rights provisions by appropriate legislation, and Congress could reasonably determine that this restriction on the Supreme Court's appellate jurisdiction is an appropriate means to that end.

Article III of the Constitution explicitly states that the Supreme Court's appellate jurisdiction is subject to such exceptions and regulations as Congress shall make. Discussion of correct answer:Answer (A) is correct because the Constitution explicitly suggests that the Supreme Court's appellate jurisdiction may be limited. Article III of the Constitution lists the types of cases over which the Supreme Court has jurisdiction. In describing the Supreme Court's appellate jurisdiction, Article III says that the Court will have appellate jurisdiction "with such Exceptions, and under such Regulations as the Congress shall make." There is a lot of uncertainty and debate about exactly what this means, but the traditional understanding is that this means Congress can carve out exceptions to the Supreme Court's appellate jurisdiction. As a result, there would be a strong argument that the language of Article III allows Congress to limit the Supreme Court's appellate jurisdiction, such as by precluding the Supreme Court from having appellate jurisdiction over voting rights cases.

A plaintiff, a former city employee, sued the city for his alleged wrongful discharge from a civil service position. The plaintiff alleged that his supervisor had discharged him in retaliation after she learned that he had told the police he thought the supervisor might be embezzling. At trial, the plaintiff has called the supervisor as an adverse witness, and the supervisor has testified that the plaintiff was fired for incompetence. The plaintiff's attorney then asks the supervisor, "Isn't it true that before the discharge you were told that [the plaintiff] had reported to the police that you were pilfering money from the office coffee fund?" For what purpose(s) is the plaintiff's question permissible? AOnly to establish the supervisor's improper motive in discharging the plaintiff. BOnly to impeach the supervisor's veracity as a witness because of her dishonesty. COnly to impeach the supervisor's veracity as a witness because of her personal bias against her accuser, the plaintiff. DBoth to impeach by showing bias and to establish improper motive in discharging the plaintiff.

Both to impeach by showing bias and to establish improper motive in discharging the plaintiff. Copyright © 2019 by the National Conference of Bar Examiners. All rights reserved. Discussion of correct answer:The general rule is that evidence of a person's character or character trait is inadmissible to prove that on a particular occasion he acted in accordance with that character or trait. However, evidence of character may be admissible when offered for a purpose other than to show conduct in conformity with one's character, such as for the purpose of impeachment or to prove motive [Fed. R. Evid. 404(b)(2)]. Here, the fact that the supervisor was told about the petitioner taking money from the office coffee fund prior to the petitioner's termination can be used to impeach the supervisor's testimony by showing his bias against the petitioner. Additionally, the specific act of taking money from the office coffee fund is admissible for the limited purpose of establishing the supervisor's improper motive for terminating the petitioner.

It was a difficult decision, but a mechanic decided to leave the street gang he had belonged to for the past six years. The mechanic realized that, if he stayed with the gang, he risked death at the hands of a rival gang. The mechanic also realized that, if he left the gang, the remaining members would turn on him. The mechanic knew that the gang leader would be especially angered by what he viewed as the mechanic's disloyalty. One day while the mechanic stood at the bus stop, a shot came from a passing car. The mechanic clutched his chest and exclaimed as he fell to the ground, "The gang leader did it!" The mechanic died immediately after that. The gang leader is now on trial for the mechanic's murder. Based on the above facts alone, how should the admissibility of the mechanic's statement be determined? ABy the judge, with instructions to the jury that it may consider the dying declaration if it determines that the mechanic knew he was dying. BBy the judge, without assistance from the jury. CBy the jury, after a preliminary determination from the judge. DBy the jury, without a preliminary determination from the judge.

By the judge, without assistance from the jury. Discussion of correct answer:This is the best choice because Federal Rule of Evidence 804(b)(2) provides for the admissibility of a dying declaration where: (1) the declarant is unavailable; (2) the declaration was made while the declarant believed his death was imminent; (3) the declaration concerned the cause or circumstances of what the declarant believed to be his imminent death; and (4) the declaration is offered in either a homicide prosecution or a civil proceeding. The FRE broadened the traditional use of dying declarations in that the declarant need not die (only be unavailable) and the declaration can be used in civil actions. The above facts would allow the judge to admit the statement as a dying declaration.

The owner of a large tract of land has subdivided the property into 40 smaller parcels and plans to develop the land as a residential community catering to cat owners. The owner creates a development scheme for the property and begins to sell lots to eager buyers. As part of the development scheme, the owner constructs an extensive set of pathways where residents can walk their cats throughout the property. In the deed to each lot, he inserts a restriction that the lot will not be used for any purpose other than the construction of a single-family residence. The owner has no land-use plans for the land other than the residential lots and the pathway system. The deeds contain a provision guaranteeing access to the pathways for the owners of each lot. Each deed also contains a clause obligating every owner of a tract to pay an annual fee, to be paid in monthly installments, for the maintenance of the pathways running through the community. Finally, the owner inserts a provision in each deed stating that both the benefit of using the system of pathways and the burden of the annual maintenance fee shall transfer to any subsequent owner of a lot. The greatest obstacle to the owner's establishing each element of his development scheme is likely to be which of the following? AThe owner's scheme imposes a restraint on alienation for each lot. BThe owner's scheme renders title to each lot unmarketable. CCourts would be reluctant to recognize the owner's scheme, because it binds future owners to monthly payments over an indeterminate period of time. DRecognition of the owner's scheme by a court constitutes a state action, which would conflict with the Fourteenth Amendment to the Constitution.

Courts would be reluctant to recognize the owner's scheme, because it binds future owners to monthly payments over an indeterminate period of time. Discussion of correct answer:Courts do not like to enforce arrangements that burden future land owners with payments over an indeterminate period of time, such as the maintenance fees for the pathway system. Therefore, this would be the greatest obstacle out of the choices listed.

In which of the following situations would a trial court be most likely to sustain an objection to a leading question? ADefense counsel asks a defense witness: "You weren't wearing your glasses that night, were you?" BPlaintiff's attorney asks Smith, a witness for the plaintiff who has been very uncooperative: "Now, Smith, you were present at the scene of the accident, were you not?" CDefense counsel asks a witness for plaintiff: "Weren't you jealous enough to steal the ruby necklace?" DPlaintiff's counsel asks a 10-year-old child testifying for the plaintiff: "Doesn't your mommy always take you to the park on Saturday?"

Defense counsel asks a defense witness: "You weren't wearing your glasses that night, were you?" Discussion of correct answer:Pursuant to Federal Rules of Evidence 611(c), the use of leading questions is not normally permitted during direct examination. However, in order to save time and to obtain relevant evidence, leading questions on direct examination will sometimes be permitted (e.g., to obtain preliminary information like names or addresses, or to examine a child, a mentally impaired witness, an expert witness, or a hostile witness.) Leading questions are usually permitted upon cross-examination. Here, defense counsel is examining his own witness, which is grounds for sustaining this type of objection.

Police detectives received information from a local department store that a certain individual was suspected of stealing numerous items over several weeks from their store. An alert loss prevention officer was able to ascertain the license plate number of the vehicle in which that the suspect drove away. Detectives traced the license plate to the residence of the defendant. Detectives confirmed that the physical description of the defendant matched the physical description of the suspect and the defendant was subsequently identified in a photo lineup by store employees. Detectives obtained a search warrant to search the premises belonging to the defendant to search for merchandise from the department store as well as theft detection device removal instruments. The police arrived to serve the search warrant and upon entry into the residence, immediately located large piles of unopened merchandise as well as several baggies of cocaine. The police seized the cocaine and the unopened merchandise and later confirmed that they were stolen from a variety of stores. The defendant was charged with several counts of theft as well as possession of cocaine. The defendant's attorney moved to suppress the cocaine and the merchandise from stores other than the department store listed in the warrant. How should the court rule on the defendant's motion? AGranted with respect to the cocaine, but not the merchandise. BGranted with respect to both the cocaine and the merchandise. CDenied regarding the cocaine only. DDenied regarding both the cocaine and the merchandise.

Denied regarding both the cocaine and the merchandise. Discussion of correct answer:The scope of a search warrant is the entire area covered by the warrant's description. Therefore, the entire premises belonging to the defendant could be searched. It should be noted that although the police were limited to searching for merchandise from that particular department store, unless the merchandise that they found was obviously belonging to a different merchant, they were privileged to seize it. Under the plain view doctrine, the police can seize items that are obviously evidentiary in nature. Therefore, the cocaine was lawfully seized.

A pedestrian domiciled in State A was crossing a street in State B when he was hit by a car driven by a citizen of a foreign country. Both the pedestrian and the driver suffered injuries. The pedestrian filed a negligence action against the driver in a federal district court in State B, seeking $100,000 in damages. The driver believes that the pedestrian was crossing the street illegally and is therefore responsible for the accident. The driver seeks an attorney's advice on how best to respond to the action. Assume that State B is a contributory negligence state. How should the attorney advise the driver to respond? AFile an answer raising the affirmative defense of contributory negligence and asserting a counterclaim for negligence, seeking damages for the driver's injuries. BFile an answer raising the affirmative defense of contributory negligence and move for judgment on the pleadings. CMove to dismiss for lack of personal jurisdiction, because the driver is not a citizen of State B. DMove to dismiss for lack of subject-matter jurisdiction, because the driver is not a U.S. citizen.File an answer raising the affirmative defense of contributory negligence and asserting a counterclaim for negligence, seeking damages for the driver's injuries.

Discussion of correct answer: Answer (A) is correct. An answer is a pleading in which the responding party admits or denies the opposing party's allegations and lists any defenses. An affirmative defense is a defense that relies on factual issues not presented in the complaint. An affirmative defense does not necessarily deny the allegations of the complaint, but it pleads additional facts. Affirmative defenses, such as contributory negligence, must be included in the party's answer [Fed. R. Civ. P. 8(c)]. A counterclaim may be brought by any party against any other opposing party and can be compulsory or permissive. A compulsory counterclaim is one that arises out of the same transaction or occurrence as the original claim in the lawsuit. Here, because the accident happened in a contributory negligence state, contributory negligence can defeat the plaintiff's entire claim, and it should be included in the answer. In addition, because the defendant is claiming injuries from the same accident, he should bring his claim for damages as a compulsory counterclaim.

A seller contracted to manufacture 1,000 toasters for a buyer for a specified price. The contract contained a provision that clearly stated: "This contract may not be assigned, and any violation of this prohibition voids the contract." After the contract was signed, the seller informed the buyer that the toasters would be manufactured by a competitor of the seller. Citing the non-assignment provision, the buyer claimed that it was no longer bound by the contract. Toasters manufactured by the competitor were of equal quality to toasters manufactured by the seller. Is the buyer bound by the contract? ANo, because "this contract may not be assigned" means that duties may not be delegated, and the seller delegated a duty. BNo, because the seller assigned a right despite the contractual prohibition. CYes, because even though the seller breached the contract, there are no damages since the competitor's toasters are of equal quality to the seller's toasters. DYes, because the non-assignment provision is not enforceable since public policy favors free assignment and delegation.No, because "this contract may not be assigned" means that duties may not be delegated, and the seller delegated a duty.

Discussion of correct answer:(A) is correct. Although the contract clause at issue refers to "assignments," it applied to both an assignment of rights and delegation of duties. In this case, the seller attempted to delegate its duties to the competitor. The general rule is that all obligations can be delegated, subject to two primary exceptions. One (not applicable here) is the personal services exception. The other is when the contract prohibits delegation; contract provisions barring delegation are fully enforceable. In this case, the buyer was entitled to enforce the prohibition against the seller's assignment (here, a delegation) of its duties to manufacture and supply the toasters.

A homeowner was injured when he slipped and fell in a puddle of water on his sunroom floor; the water had accumulated on the floor during a rainstorm because of leaks in the roof. The roof's manufacturer had supplied nondefective materials to the installer, who was a franchisee (and not an employee) of the manufacturer. The leaks resulted from the carelessness of the installer during the installation of the roof. The installer's truck, which had been parked in front of the homeowner's house during the roof installation, bore the manufacturer's logo. The manufacturer was aware that the truck and the literature supplied by the installer both displayed the manufacturer's logo. Is there any basis for a claim by the homeowner against the manufacturer? ANo, because a franchisor has no duty to supervise the conduct of a franchisee. BNo, under the rule that a manufacturer is liable only for defects in a product that existed at the time the product left the hands of the manufacturer. CYes, because the installer was a franchisee of the manufacturer. DYes, under the rule of apparent agency.Yes, under the rule of apparent agency. Copyright © 2019 by the National Conference of Bar Examiners. All rights reserved.

Discussion of correct answer:Answer (D) is correct because the manufacturer could be liable for the installer's negligence under a theory of apparent agency. While employers can be held liable for the torts of their employees under respondeat superior, a person who hires an independent contractor to do work is generally not liable for the torts of the independent contractor. For example, if you hire a painter to paint your house, the painter is an independent contractor rather than an employee, and you are not liable if someone is injured by the painter's negligence. An exception arises when there is apparent agency. If you create the impression that the painter is your employee rather than an independent contractor, then you may be liable for the painter's torts under respondeat superior. The idea is that if you make it appear as though someone is acting as your agent, you cannot later disavow that apparent agency and deny that you are liable for the apparent agent's tortious acts. The manufacturer knew that the installer was using the manufacturer's logo on the installer's truck and sales literature. The homeowner might therefore argue that the manufacturer gave the impression that there was an employment or agency relationship between the manufacturer and installer, and that the installer was acting as the manufacturer's agent rather than as a mere independent contractor.

A 20-year-old enlisted in the armed forces reserves to train as a medic and earn extra money while he worked his way through undergraduate school. The 20-year-old was a member of a church that practiced pacifism, but the 20-year-old believed that a few weekends and a couple of weeks a year dedicated to learning how to care for the injured would not violate his church's edicts, even though it was a military organization that he joined. Shortly after he completed his basic training, the 20-year-old's unit was called up to serve in an ongoing war. Not yet trained as a medic, the 20-year-old was ordered to serve in combat. Despondent over this turn of events and realizing that he'd made a big mistake, the 20-year-old filed for conscientious objector status so that he might be reassigned to civilian duty and therefore maintain his religious beliefs without personal conflict. In response, a military official informed the 20-year-old that the military would first have to evaluate the 20-year-old's history with the pacifist religion to determine his sincerity, and that second, for reassignment to civilian duty, the 20-year-old would have to sign a form waiving his rights to veteran's educational benefits, a rule that applies to all military personnel reassigned to civilian service. The 20-year-old filed a legal action claiming that the military was violating his rights of free exercise of religion. Assuming correct forum and standing, and that educational benefits can in fact be waived, what is the likely outcome of the 20-year-old's case? AIn favor of the 20-year-old, because the government cannot determine the truth or falsity of someone's religious beliefs. BIn favor of the 20-year-old, because the government may not deny benefits to a person based on their religious belief. CIn favor of the military, because its means were narrowly tailored to serve a compelling interest. DIn favor of the military, because the government may regulate conduct, including that motivated by religious belief, as long as the regulation is religion-neutral and is of general applicability.

In favor of the military, because the government may regulate conduct, including that motivated by religious belief, as long as the regulation is religion-neutral and is of general applicability. Discussion of correct answer:This correct answer choice states the standard set by the Supreme Court in Employment Division v. Smith [494 U.S. 872 (1990)]. Although the 20-year-old was seeking reassignment for religious reasons, the regulation requiring waiver of educational benefits was one of general applicability and applied regardless of religion. In light of the Smith case, the 20-year-old most likely will not be able to refuse to sign the waiver of educational benefits if he wants civilian reassignment--at least not based on violation of the Free Exercise Clause. A different outcome might occur if it turned out that the only grounds for civilian reassignment are religious, but the facts don't suggest it. Furthermore, although the government cannot determine the truth or falsity of a person's religion, they can determine that person's sincerity with regard to his religious beliefs. If the military investigates the 20-year-old's membership in the church for this purpose, it is not a violation of the Free Exercise Clause.

An airliner based out of a foreign country operated flights that traveled daily from the country's capital city to State B. Many residents of the foreign capital flew on the airliner's flights to State B in order to visit extended family or vacation. One of the airliner's flights crashed in State A while on its way to State B. Evidence indicated that the plane was not properly prepared for flight while on the runway in the foreign capital and this caused the crash. Attorneys representing the estates of the 200 passengers who had been on the plane - including 100 foreign passengers, 97 passengers from State B, and three passengers from State A - filed a class action suit against the airliner in State A federal district court, asserting claims based on the laws of the foreign country and alleging damages of $100 million. The airliner was concerned that it would not receive a fair hearing in State A, and wanted to avoid going to trial there. Which of the following legal arguments should the airliner assert? ALack of personal jurisdiction. BLack of subject-matter jurisdiction. CImproper venue. DForum non conveniens.

Forum non conveniens. Discussion of correct answer:The doctrine of forum non conveniens allows a court to decline to exercise its jurisdiction and dismiss an action if the court where the action was brought is a seriously inconvenient forum and an adequate alternative forum exists. Here, the defendant is domiciled in a foreign country, as are the majority of the plaintiffs making up the class, and the claims are based on the laws of the foreign country. Furthermore, the facts indicate that evidence related to the cause of the crash is likely to be found in the foreign country, as the plane was not properly prepared for flight in the capital city. All of these factors weigh in favor of a finding of forum non conveniens.

A lawyer specialized in personal injury work. A woman had been disabled in an automobile accident. She was struck by a car driven by a driver. The woman hired the lawyer to represent her. After several years had passed without the woman receiving any compensation for her injuries, the woman discovered that the lawyer had failed to have the driver served with the lawsuit. The time for service had passed so the woman was denied any recovery. The woman then filed a suit against the lawyer for malpractice. The woman will be able to establish a breach of the standard of care owed to her by the lawyer in which of the following situations? AOnly if an attorney expert from the same community testifies that the lawyer did not act as a reasonably prudent lawyer in good standing. BOnly if an attorney expert testifies that the lawyer did not act as a reasonably prudent lawyer in good standing. CIf the judge agrees that the jurors, based on their common knowledge, can determine if the lawyer acted as a reasonably prudent lawyer in good standing. DIf the judge advises the jury that the lawyer did not act as a reasonably prudent lawyer in good standing.

If the judge agrees that the jurors, based on their common knowledge, can determine if the lawyer acted as a reasonably prudent lawyer in good standing. Discussion of correct answer:Whether a defendant has breached his duty to the plaintiff is a question of fact to be determined by the jury. In most professional malpractice cases, the jury receives expert testimony as to whether the defendant has failed to act in accordance with general standards of practitioners in good standing. However, in some instances no special knowledge is necessary to determine that a professional has performed inadequately. That judgment can be made from the jurors' experience. If the judge determines that the jury is competent enough to judge the reasonableness of a failure to serve a lawsuit, then no expert testimony will be necessary to prove breach.

A state enacted a statute that provided scholarships to students who attained high academic achievement and were from families with low or modest incomes. A portion of the statute stated that no part of the scholarship funds could be used by a student to pursue a degree in theology. This portion of the statute was reflective of the state's constitution, which provided that no public money could be used to fund religious worship or studies. The scholarship aid was available to be used at religiously affiliated colleges, so long as the college was nationally accredited and the scholarship recipient did not pursue a degree in theology. A student eligible for the scholarship wished to attend a religiously affiliated college in the state to pursue a degree in theology. When informed that he could not receive the scholarship and pursue a degree in theology, the student sued the state to enjoin it from refusing to award the scholarship. How should the court rule? AIn favor of the student, because under strict scrutiny analysis, the state had singled out religion for unfavorable treatment, and the state's anti-establishment concerns were not compelling. BIn favor of the student, because a person's religious beliefs are protected, and a government may not punish an individual by denying benefits or imposing burdens based on religious belief. CIn favor of the state, because although the state's funding of theological studies would be permitted under the Establishment Clause, the Free Exercise Clause did not require the state to do so. DIn favor of the state, because under strict scrutiny analysis, while the state had singled out religious study for unfavorable treatment, the state's anti-establishment concerns were compelling.

In favor of the state, because although the state's funding of theological studies would be permitted under the Establishment Clause, the Free Exercise Clause did not require the state to do so. Discussion of correct answer:The First Amendment provides that "Congress shall make no law respecting an establishment of religion or prohibiting the free exercise thereof." The Establishment Clause and the Free Exercise Clause are often in tension, as there exist areas in which state action is permitted under the Establishment Clause but is not mandated by the Free Exercise Clause. This question, modeled after Locke v. Davey [540 U.S. 714 (2004)], describes one such area. Here, the state could have chosen to grant scholarships to students for use in pursuing a theology degree without violating the Establishment Clause. However, the state's exclusion of the pursuit of a theology degree from its otherwise-inclusive scholarship aid program does not violate the Free Exercise Clause, because the state's interest in not funding the pursuit of devotional degrees is substantial, and the exclusion of such funding places a relatively minor burden on scholarship recipients, as it did not prohibit the free exercise of their religion. The Supreme Court has stated that discrimination because a person or institution is religious, in and of itself, is not permissible. The activity must be the focus of the analysis. As such, this is the best response.

A defendant is on trial for attempted fraud. The state charges that the defendant switched a price tag from a cloth coat to a more expensive fur-trimmed coat and then presented the latter for purchase at the cash register. The defendant has testified in her own behalf that someone else must have switched the tag. On cross-examination, the prosecutor asks whether the defendant was convicted on two prior occasions of misdemeanor fraud of a retailer by the same means of switching the price tag on a fur-trimmed coat. Is the question about the prior convictions proper either to impeach the defendant or to prove that the defendant committed the crime? AIt is not proper for either purpose. BIt is proper for both purposes. CIt is proper to impeach the defendant, but not to prove that the defendant committed the crime. DIt is proper to prove that the defendant committed the crime, but not to impeach the defendant.

It is proper for both purposes. Discussion of correct answer:Under Federal Rule of Evidence 404(b), prior bad acts can be admitted to prove the defendant's conduct if offered for some purpose other than to show that the defendant is a bad person. In this case, the bad acts are very similar to the acts in dispute and may be admitted for other purposes such as proving intent, knowledge, lack of accident, and modus operandi (i.e., that the defendant has a tendency to engage in particularized activity that sets her apart from others). Thus the bad acts can be offered as proof that the defendant committed the crime charged. Moreover, the convictions are automatically admissible to impeach the defendant's character for truthfulness; fraud convictions clearly involve dishonesty or false statement, and so the court "must" admit the convictions under Rule 609(a)(2).

A businessman owned a hotel, subject to a mortgage securing a debt he owed to a bank. The businessman later acquired a nearby parking garage, financing a part of the purchase price with a loan from a financing company, secured by a mortgage on the parking garage. Two years thereafter, the businessman defaulted on the loan owed to the bank, which caused the full amount of that loan to become immediately due and payable. The bank decided not to foreclose the mortgage on the hotel at that time, but instead properly sued for the full amount of the defaulted loan. The bank obtained and properly filed a judgment for that amount. A statute of the jurisdiction provides: "Any judgment properly filed shall, for 10 years from filing, be a lien on the real property then owned or subsequently acquired by any person against whom the judgment is rendered." There is no other applicable statute, except the statute providing for judicial foreclosure of mortgages, which places no restrictions on deficiency judgments. Shortly thereafter, the bank brought an appropriate action for judicial foreclosure of its mortgage on the hotel and of its judgment lien on the parking garage. The financing company was joined as a party defendant, and appropriately sued for foreclosure of its mortgage on the parking garage, which was also in default. All procedures were properly followed, and the confirmed foreclosure sales resulted in the following: The net proceeds of the sale of the hotel to a third party were $200,000 less than the bank's mortgage balance. The net proceeds of the sale of the parking garage to a fourth party were $200,000 more than the financing company's mortgage balance. How should the $200,000 surplus arising from the sale of the parking garage be distributed? AIt should be paid to the bank. BIt should be paid to the businessman. CIt should be paid to the financing company. DIt should be split equally between the bank and the financing company.

It should be paid to the bank. Discussion of correct answer:The foreclosure sale of the bank's mortgage on the hotel was insufficient to pay the businessman's debt to the bank. The bank had received a judgment against the businessman for the entire amount of the defaulted loan. This judgment was properly filed, and the judgment lien applied to all property owned by the businessman during the 10-year time period, including the parking garage. (The bank may have decided on this course of action because it deemed the businessman's equity in the garage significant and the timing bad for a hotel foreclosure.) After the financing company was paid in full from the funds generated by the foreclosure sale of its mortgage on the parking garage, the additional funds generated by that sale would be paid to the bank not as a deficiency judgment, but because of the unsatisfied amount of the prior money judgment.

Congress was very concerned about the rise in gun-related deaths and injuries that had been occurring in the schools. To combat the problem, Congress passed a statute that required states to enact legislation that made it a crime to sell, possess, or transport any firearm within 100 yards of any school, if that firearm had been purchased or sold through or using any instrumentality of interstate commerce. Is the statute constitutional? AYes, because the statute will permit a case-by-case review of the adequacy of an interstate commerce connection between the permissible objectives of the federal statute and any individual criminal prosecution. BYes, because the statute is a proper attempt to regulate interstate commerce in its inseverable aggregates. CNo, because Congress cannot commandeer the states to enact and enforce legislation. DNo, because the connection between interstate commerce and the purpose of the regulation is too remote to justify the exercise of the Commerce Clause power.

No, because Congress cannot commandeer the states to enact and enforce legislation. Discussion of correct answer:In United States v. Lopez [514 U.S. 549 (1995)], the Supreme Court invalidated a federal statute that imposed criminal penalties on persons who possessed a firearm within 1,000 feet of a school building. The court held that the Gun-Free School Zones Act of 1990 was unconnected to interstate commerce and did not have a real effect on multistate commercial concerns. As a result, the federal law could not be upheld under Congress's commerce power. In this question, we have a subtle variation, inasmuch as Congress is requiring the states to enact legislation criminalizing firearm possession. In all likelihood, this would be deemed unconstitutional because Congress cannot "commandeer the legislative processes of the states by directly compelling them to enact and enforce a federal regulatory program" [New York v. United States, 505 U.S. 144 (1992)]. In New York, the court held that there are limits to Congress's right to interfere with the states' lawmaking processes, and Congress will violate the Tenth Amendment if it exceeds those limits.

A political group that is against nuclear power sued a local nuclear power plant that was supplying power throughout the state. The group claimed the nuclear power plant did not have the proper safeguards and was emitting radiation that was polluting the water in the area. The nuclear power plant responded that any of its emissions were regulated by the U.S. Environmental Protection Agency under the authority of the Nuclear Power Act, which specifically covered nuclear power plant safety issues. However, the Agency had yet to set standards regarding this particular plant, though the plant met all broad, non-specific safety requirements set by the EPA. Can the political group proceed with its suit? AYes, because the Agency has yet to set standards regarding this particular plant, so federal common law applies. BYes, because there is no indication the Act specifically preempted the federal common law for nuisance. CNo, because there is no federal common law under the Erie doctrine. DNo, because Congress intends the Agency and the Act to regulate nuclear power.

No, because Congress intends the Agency and the Act to regulate nuclear power. Discussion of correct answer:Congressional action will take precedence over common law rules, if any. There is only federal common law for the interpretation of federal statutes and Congressional intent. Here, because Congress has enacted legislation regarding the issue of nuclear power, the political group must sue according to the rules provided by the Act [American Elec. Power Co. v. Connecticut, 564 U.S. 410 (2011)]. Because there are no standards for the plant yet under the Act, and because there is no common law nuisance action to bring, the group cannot proceed with its suit.

An individual investor purchased stock through a company's stock offering. When the price of the stock plummeted, the investor sued the company in a state court in State A, claiming that the company's offering materials had fraudulently induced him to purchase the stock and seeking $25,000 in damages. A university that had purchased the company's stock through the same offering sued the company in federal court in State B, claiming that the offering materials violated federal securities laws and seeking $1 million in damages. The individual investor's suit proceeded to trial. The state court ruled that the company's offering materials contained false information and awarded the investor a $25,000 judgment. The university immediately moved for partial summary judgment in its federal action against the company, arguing that the state court judgment bound the federal court on the issue of whether the company's offering materials contained false information. Neither State A nor State B permits nonmutual issue preclusion. Should the court grant the university's motion? ANo, because State A does not permit nonmutual issue preclusion. BNo, because the federal court sits in a state that does not permit nonmutual issue preclusion. CYes, because federal law permits nonmutual issue preclusion. DYes, because the issue of whether the materials contained false information was actually litigated and necessarily decided.

No, because State A does not permit nonmutual issue preclusion. Discussion of correct answer:Answer (A) is correct. Issue preclusion prevents relitigation of issues that were fully and fairly litigated and were necessarily decided in a proceeding that reached a final judgment on the merits. Nonmutual issue preclusion is when a party seeks to prevent litigation of an issue that the other party has unsuccessfully litigated before a different party. If the two claims were brought in two different jurisdictions, the issue of what law to apply arises. If the first case was decided in a state court, the second court will apply the preclusion rules of the first court. This is true whether the second court is a state or federal court. Consequently, because the first court applied the rules of State A, the second court will not allow nonmutual issue preclusion

Two friends were charged with kidnapping a young woman. One friend's attorney moved to have the two men tried separately and this was granted. The first friend to be tried was acquitted of the charge. At the second friend's trial, his attorney called the first friend's current housemate to testify. The housemate testified that after the first friend was acquitted, the first friend told him that he had kidnapped the woman, because he wanted to have a romantic relationship with her and that the second friend had nothing to do with it. Should the housemate's testimony be admitted into evidence? ANo, because it is hearsay not within any exception. BNo, because the first friend is estopped from contradicting the jury's acquittal verdict. CYes, because it is a statement by a co-conspirator. DYes, because it is a statement against a penal interest.

No, because it is hearsay not within any exception. Discussion of correct answer:Pursuant to Federal Rule of Evidence 801(c), "hearsay" is an out-of-court statement offered in evidence to prove the truth of the matter asserted. Some out-of-court statements are admissible as an exception to the hearsay rule if the statement is relevant in proving something other than the truth of the matter that is asserted. In this question, the housemate's testimony is being used to prove the truth of the matter of the first friend's participation in the kidnapping, as well as the lack of involvement on the part of the second friend and is thus hearsay pursuant to Rule 801(c). Because the first friend had already been acquitted of the crime at the time he made the statement, the statement does not come within an exception to the hearsay rule. It cannot be admitted.

A consumer website allowed members of the public to post comments about companies with which they had done business. One consumer posted a comment stating that the restaurant had the worst food he had ever eaten and that he was sure that its chef must be deliberately trying to make patrons sick. The restaurant filed suit in federal court alleging trademark infringement, as it had not granted the consumer website the right to use its name. The consumer website filed an answer two weeks later that generally denied all of the allegations that the restaurant had made. Two weeks after filing the answer, the consumer website filed a motion to dismiss for failure to state a claim on which relief can be granted. In its reply to the motion, the restaurant stated that the court should deny the motion as being untimely. Is the restaurant correct? AYes, because the motion was filed after the answer. BYes, because the 21 days to answer the complaint had already passed. CNo, because motions to dismiss can be filed after the first responsive pleading. DNo, because motions to dismiss for failure to state a claim need not be raised in the first responsive pleading.

No, because motions to dismiss for failure to state a claim need not be raised in the first responsive pleading. Discussion of correct answer:Many grounds for dismissal must be raised in a first responsive pleading, whether it be a motion to dismiss or an answer. If a party submits an answer that does not address these grounds, he cannot then address them in a subsequent motion. However, several grounds for dismissal are not waivable--subject-matter jurisdiction, failure to join a required party, and failure to state a claim on which relief can be granted.

A bus passenger was seated next to a woman whom he did not know. The woman stood to exit the bus, leaving a package on the seat. The passenger lightly tapped the woman on the back to get her attention and to inform her that she had forgotten the package. Because the woman had recently had back surgery, the tap was painful and caused her to twist and seriously injure her back. If the woman sues the passenger to recover for the back injury, will she be likely to prevail? ANo, because she is presumed to have consented to the ordinary contacts of daily life. BNo, because she was not put in apprehension by the touching. CYes, because the passenger intentionally touched her. DYes, because the passenger's intentional touching seriously injured her.

No, because she is presumed to have consented to the ordinary contacts of daily life. Discussion of correct answer:The woman gave no indication that she did not want to be subjected to the ordinary contacts that are part of life in a crowded society. In the absence of such an indication from her, the passenger was entitled to believe that she implicitly consented to a light tap to get her attention. The passenger's touch was intentional, but it was neither unreasonable nor inconsistent with ordinary social norms privileging such contacts.

An engineer was a citizen of State C and filed a case against a corporation in state court in State A. The corporation was incorporated in State B and had its principal place of business in State A. The complaint alleged that the corporation harmed the engineer in two unrelated transactions and that the plaintiff was harmed in each transaction in the amount of $50,000. The first claim was defamation and the second claim was based on a breach of contract. The corporation removed the case to federal court in order to assert a defense under federal law. Was the removal to federal court proper? AYes, because the corporation asserted a federal defense. BYes, because the plaintiff could have filed the case originally in federal court. CNo, because the corporation had its principal place of business in State A. DNo, because there was an insufficient amount in controversy.

No, because the corporation had its principal place of business in State A. Discussion of correct answer:The general rule under 28 U.S.C. Section 1441(a) is that a corporation may remove a case filed in state court to federal court if the case could have been brought originally in federal court. This case could have been brought originally in federal court because the engineer was a citizen of State C and the corporation was a citizen of States A and B, and the amount in controversy exceeded $75,000. The engineer may add together all of the engineer's claims, even two unrelated claims, to get over the amount in controversy threshold. However, there is an exception to the right of removal that applies here. A corporation who is a citizen of the state where the case is brought originally may not remove a case, even if the case could have been brought originally in federal court, if the case if founded solely on diversity (as it was here) and the corporation is a citizen of the state where the case was brought. Here, the corporation was a citizen of State A because it has its principal place of business there, under 28 U.S.C. Section 1332(c)(1). Therefore, under 28 U.S.C. Section 1441(b), the corporation may not remove the case to federal court.

A debtor borrowed $50,000 from a bank and issued to bank a mortgage on property owned by the debtor. The mortgage instrument stated in part: "In the event the debtor defaults on the debt and the bank forecloses, the debtor agrees that her right to redeem shall last for a maximum of six weeks following a declaration of default by the bank." A year later the debtor defaulted on the debt and the bank declared a default. A week later the bank initiated foreclosure proceedings. Six weeks later, before a foreclosure sale had taken place, the debtor tendered the full amount due on the debt. The bank refused to accept the tender and proceeded with the foreclosure. Was the bank's action proper? AYes, because the debtor waived her right to redeem in the mortgage instrument. BYes, because the bank had already initiated foreclosure proceedings. CNo, because the debtor properly redeemed. DNo, because the bank lost its right to foreclose on the property by attempting to "clog" the right of redemption.

No, because the debtor properly redeemed. Discussion of correct answer:A mortgagee is not allowed to "clog" the mortgagor's equitable right of redemption. Any such statement in a mortgage document that attempts to do so will be unenforceable. The debtor's equitable right of redemption exists until a foreclosure sale has taken place.

Two friends drove to a concert in friend A's car. After the concert, friend A's car was involved in an accident with a car driven by the plaintiff. At the time of the accident, it was unclear who was at fault. The next day, friend A complained of an intense pain in his neck, and he could not sleep for fear of being sued in relation for the accident. Friend B took friend A to see his family doctor. Concerned for his friend, and believing that he might be able to supply further information to the doctor concerning the accident, friend B remained in the room while the doctor examined friend A. The doctor asked friend A how he'd injured his neck. Friend A told him, "If you promise not to tell anyone, it was in a car accident. I have narcolepsy and fell asleep at the wheel and crashed into another car." Friend A was eventually sued by the plaintiff, who sought to introduce the doctor's testimony. Friend A objected. Assume that this jurisdiction recognizes the physician-patient privilege. Should the court admit the doctor's testimony? AYes, because friend A's statement was a statement by a party opponent. BYes, because the presence of friend B, a non-essential third party, destroyed the physician-patient privilege. CYes, because the examination was in contemplation of litigation. DNo, because the doctor's conversation with friend A was privileged.

No, because the doctor's conversation with friend A was privileged. Discussion of correct answer:The physician-patient privilege applies when the patient consults the doctor for purposes of diagnosis or treatment. Here, friend A went to the doctor to seek medical advice, and friend A discussed his narcolepsy and the auto accident only in response to the doctor's questions. Given that the conversation between the doctor and friend A was for the purpose of diagnosis and treatment, the physician-patient privilege applies. According to the physician-patient privilege, a patient has a privilege to refuse to disclose and to prevent any other person from disclosing confidential communications made for the purpose of diagnosis or treatment. Friend A, the patient, has the privilege to prevent the doctor from disclosing the communications at trial. While it may appear that the presence of friend B should destroy the confidentiality of the communication, it does not. The mere presence of a third party does not automatically destroy the physician-patient privilege when the person is present: (1) to aid the patient; (2) as a necessary and customary participant in the consultation or treatment; (3) to assist the physician or enable the patient to obtain treatment; or (4) where the information is readily discernable to everyone present. Here, friend B was present because he believed he could provide additional necessary information which would aid the patient in obtaining treatment. As such, his presence was to aid the patient, and his being there will not destroy the physician-patient privilege.

A man was angry at a coworker who had received a promotion. The man believed that the coworker had taken credit for the man's work and had bad-mouthed him to their boss. One day, as the man was leaving the company parking lot in his car, he saw the coworker walking through the lot. On a sudden impulse, the man pushed the accelerator pedal hard and veered toward the coworker with the intention of scaring him. The coworker tried to jump out of the way but slipped and fell and was run over. Although the coworker suffered life-threatening injuries, he survived. In a jurisdiction that follows the common law of homicide, could the man properly be convicted of attempted murder? ANo, because the coworker's slip and fall broke the chain of proximate causation. BNo, because the man lacked the requisite intent. CYes, because the coworker suffered life-threatening injuries. DYes, because the man acted with reckless disregard for human life.

No, because the man lacked the requisite intent. Discussion of correct answer:Attempt is the crime of "almost" committing another crime--the target offense. The actus reus of attempt is an overt act that indicates the defendant went beyond the point of preparation and initiated perpetration. The mens rea for attempt is the intent to do the overt act and the specific intent (purpose) to complete the alleged target offense. The tricky part of this question is that the victim survived. The alleged attempted offense here is murder. Had the victim died the defendant could be convicted of murder because his extremely reckless act manifested a depraved heart which would support a finding that unintentional killing was the result of implied malice. However, the victim did not die. As a result, to convict for attempted murder the proof has to establish the defendant acted with the intent/purpose to commit that target offense, meaning he acted with the intent/purpose to kill. Here the facts indicate he was not intending to kill his victim. Accordingly, the evidence is insufficient to prove the mens rea for attempted murder.

Seven years ago, a man conveyed vacant land by warranty deed to a woman, a bona fide purchaser for value. The woman did not record the warranty deed and did not enter into possession of the land. Five years ago, the man conveyed the same land to a neighbor, also a bona fide purchaser for value, by a quitclaim deed. The neighbor immediately recorded the quitclaim deed and went into possession of the land. Two years ago, the neighbor conveyed the land to a friend, who had notice of the prior conveyance from the man to the woman. The friend never recorded the deed but went into immediate possession of the land. The jurisdiction has a notice recording statute and a grantor-grantee index system. If the woman sues to eject the friend, will the woman be likely to succeed? ANo, because the friend took possession of the land before the woman did. BNo, because the neighbor's title was superior to the woman's title. CYes, because the friend had notice of the conveyance from the man to the woman. DYes, because the woman, unlike the friend, took title under a warranty deed.

No, because the neighbor's title was superior to the woman's title. Discussion of correct answer:(B) is correct. Under a notice statute, an unrecorded conveyance or other instrument is invalid as against a subsequent bona fide purchase for value and without notice. Here, because the woman did not record her interest and the neighbor was a bona fide purchaser without notice, his interest was superior. Consequently, when he conveyed this interest to the friend, the friend's interest became superior to the woman's interest, despite the friend having explicit knowledge of the woman's interest. This is because the friend will be able to shelter under the neighbor's superior title.

A college dormitory has a bike rack outside the front door. A college freshman woke up late on the day of his chemistry final examination. As he exited the dormitory, the freshman decided to take a bike owned by another student to get to class because if he walked to class it would take another 30 minutes. The bike did not have a lock. After he finished his chemistry final, the college freshman returned the bike exactly where he found it in the bike rack. The owner of the bike slept through the entire incident and never knew his bike had been taken. Does the owner of the bike have a cause of action for trespass to chattels? AYes, because the owner of the bike was damaged. BYes, because the college freshman intermeddled with the owner of the bike's possession of the bike. CNo, because the owner of the bike was unaware that the bike had been taken. DNo, because the owner of the bike has not sustained any damages.

No, because the owner of the bike has not sustained any damages. Discussion of correct answer:Trespass to chattel requires an intentional act that interferes with the plaintiff's possession of the property. The interference can amount to a temporary dispossession or something short of dispossession such as intermeddling, which does not directly affect the plaintiff's possession. Letting the air out of a car's vehicle would amount to intermeddling. Unlike other intentional torts, trespass to chattel requires actual damages. The facts indicate the college friend did temporarily dispossess the bike owner of possession, but the facts also indicate that there was no actual harm that resulted. It does not appear any harm was done to the bike nor does it appear that the bike owner was otherwise damaged. The bike owner was asleep during the dispossession and unaware the bike had been taken. While the bike owner will be able to demonstrate there was interference with his possession, he will be unable to prove any actual damages were sustained.

A man filed a federal diversity action against a bus company, seeking damages for injuries he had sustained in an accident while riding a bus owned by the company. The man demanded a jury trial. After the parties' attorneys examined the prospective jurors and exercised their challenges, six jurors and two alternate jurors were chosen. During the trial, two jurors became ill and were replaced by the alternate jurors. At the conclusion of the trial, a third juror also became ill, and the court excused that juror. The parties' attorneys stipulated to the return of a verdict from a five-person jury. The jury then deliberated and returned a verdict for the company. The man timely filed a motion for a new trial, arguing that the five-person jury was not large enough to return a verdict. Should the court grant the motion? ANo, because the court properly excused the three jurors due to illness. BNo, because the parties stipulated to a verdict from a jury of fewer than six jurors. CYes, because there must be at least six jurors on a federal civil jury. DYes, because there must be at least 12 jurors on a federal civil jury.

No, because the parties stipulated to a verdict from a jury of fewer than six jurors. Discussion of correct answer:Answer (B) is correct. Generally, "a jury must begin with at least six and no more than 12 members" [Fed. R. Civ. P. 48(a)]. However, parties may stipulate to a verdict returned by less than six jurors [Fed. R. Civ. P. 48(b)]. Here, because the parties stipulated to a verdict from a five-person jury, the court should not grant the man's motion.

associate professor in the pediatrics department of a local medical school was denied tenure. He asked a national education lobbying organization to represent him in his efforts to have the tenure decision reversed. In response to a letter from the organization on the professor's behalf, the dean of the medical school wrote to the organization explaining truthfully that the professor had been denied tenure because of reports that he had abused two of his former patients. Several months later, after a thorough investigation, the allegations were proven false, and the professor was granted tenure. He had remained working at the medical school at full pay during the tenure decision review process and thus suffered no pecuniary harm. In a suit for libel by the professor against the dean of the medical school, will the professor be likely to prevail? ANo, because the professor invited the libel. BNo, because the professor suffered no pecuniary loss. CYes, because the dean had a duty to investigate the rumor before repeating it. DYes, because the dean's defamatory statement was in the form of a writing.

No, because the professor invited the libel. Discussion of correct answer:The professor can state a prima facie case of defamation, but he cannot prevail because the dean has a valid defense based on the dean's reasonable belief that the professor had invited him to speak. By authorizing his agents to investigate his case, the professor had apparently consented to limited publication in response to their inquiries. Ill will, if it existed, would be irrelevant to this defense.

During a comprehensive evaluation of an adult patient's psychiatric condition, a psychiatrist failed to diagnose the patient's suicidal state. One day after the misdiagnosis, the patient committed suicide. The patient's father, immediately after having been told of his son's suicide, suffered severe emotional distress, which resulted in a stroke. The patient's father was not present at the patient's appointment with the psychiatrist, nor did he witness the suicide. The father has brought an action against the psychiatrist to recover for his severe emotional distress and the resulting stroke. Is the father likely to prevail? ANo, because the father did not sustain a physical impact. BNo, because the psychiatrist's professional duty did not extend to the harms suffered by the patient's father. CYes, because the father was a member of the patient's immediate family. DYes, because the psychiatrist reasonably could have foreseen that a misdiagnosis would result in the patient's suicide and the resulting emotional distress of the patient's father.

No, because the psychiatrist's professional duty did not extend to the harms suffered by the patient's father. Discussion of correct answer:In most situations, a medical professional's duty of care extends only to his or her patient. The patient posed no threat to others. Considerations of privacy and confidentiality usually lead courts to deny a duty on the part of therapists to non-patients when only the patient is at risk.

Police received a tip from an informant that a ring of thieves were hoarding stolen property in the house owned by the ringleader's uncle. The ringleader frequented the uncle's house to sell the stolen goods out of the basement and could be found there every evening after 10:00 pm. The police obtained an arrest warrant and went to the uncle's house to arrest the ringleader the next evening. The uncle refused entry to the police, who pushed past him to find the ringleader. When police entered the living room, they observed bricks of marijuana on the table. Police arrested the uncle and the ringleader and charged them both with narcotics offenses. The uncle's motion to suppress the evidence of the drugs was granted. The ringleader files a similar motion seeking suppression of the drugs. Should the ringleader's motion be granted? AYes, because the police did not have a valid search warrant for the uncle's house. BYes, because there was no probable cause to believe that the drugs belonged to the ringleader. CNo, because the ringleader had no standing to contest the legality of the search. DNo, because the drugs were observed in plain view and there were exigent circumstances.

No, because the ringleader had no standing to contest the legality of the search. Discussion of correct answer:Standing under the Fourth Amendment is personal to the defendant. Thus, in order for the ringleader to have standing to vindicate his own rights under the Fourth Amendment, he must have a reasonable and legitimate expectation of privacy in the place searched or the item seized. The ringleader does not have a legitimate expectation of privacy in someone else's home and therefore, does not have standing.

A defendant has pleaded not guilty to a federal bank robbery charge. The principal issue at trial is the identity of the robber. The prosecutor has called the defendant's wife to testify to the clothing that the defendant wore as he left their house on the day the bank was robbed, expecting her description to match that of eyewitnesses to the robbery. Both the defendant and his wife have objected to her testifying against the defendant. Should the wife be required to testify? ANo, because the defendant has a privilege to prevent his wife from testifying against him in a criminal case. BNo, because the wife has a privilege not to testify against her husband in a criminal case. CYes, because the spousal testimonial privilege does not apply in criminal cases. DYes, because the wife's viewing of the defendant's clothing was not a confidential communication.

No, because the wife has a privilege not to testify against her husband in a criminal case. Discussion of correct answer:This is a correct statement of federal common law, established by the U.S. Supreme Court in Trammel v. United States. If the witness and the defendant are married at the time of trial, the witness cannot be placed in contempt for refusing to testify against the defendant. The rationale for the rule is to preserve marital harmony, which would otherwise be damaged by one spouse testifying against the other.

A woman heard a sound coming from her back porch. Curious, she approached the back door. A man wearing a ski mask was standing in her home pointing a knife in her general direction. The woman remarked, "Joey, I know it's you. If you need money, you could have just asked me instead of pulling this stunt." The man angrily grabbed a gold watch sitting on the dryer next to the back door and left the house. The woman called the police and reported the incident. The man was charged with robbery. Will the man be convicted of robbery? ANo, because the man did not use force or violence in taking the watch. BNo, because the woman did not actually fear the man's threat of violence. CYes, because the man took the watch from an area within the woman's control. DYes, because the man used force, albeit slight, to carry out the taking of the watch.

No, because the woman did not actually fear the man's threat of violence. Discussion of correct answer:Robbery includes all of the elements of larceny, with two additional requirements: (1) the taking must be from the person or presence of the victim (meaning an area within the victim's control); and (2) the taking must be accomplished by actual force or violence or by intimidation or the threat of violence. However, where robbery is based only on the threat of violence, the threat must place the victim in actual fear at the time of the taking. Here, the man accompanied his taking only with a threat of violence (i.e., the holding of a knife pointed in her general direction); thus, his threat must have placed the woman in actual fear at the time of the taking. Because the woman clearly was not in any fear of the man's threats, the man did not commit robbery.

A man brought a federal diversity action against his insurance company, alleging that the company had breached its duty under his insurance policy by refusing to pay for his medical expenses resulting from a mountain-biking accident. At the jury trial, the man presented evidence that he had paid all premiums on the insurance policy and that the policy covered personal-injury-related medical expenses arising from accidents. After he rested his case, the company presented evidence that a provision of the policy excluded payment for injury-related expenses resulting from an insured's "unduly risky" behavior. The company also presented a witness who testified that the accident had occurred in an area where posted signs warned bikers not to enter. The man did not cross-examine the witness. After resting its case, the company moved for judgment as a matter of law. Should the court grant the motion? ANo, because a motion for judgment as a matter of law must first be made at the close of the plaintiff's case-in-chief. BNo, because whether the man's behavior was unduly risky is a question of fact for the jury to resolve. CYes, because the company's uncontradicted evidence of the man's unduly risky behavior means that no reasonable jury could find that the policy covers his injuries. DYes, because the man waived his right to rebut the company's evidence by not addressing the "unduly risky" policy provision in his case-in-chief.

No, because whether the man's behavior was unduly risky is a question of fact for the jury to resolve. Discussion of correct answer:Because a motion for judgment as a matter of law takes the case away from the jury, it can be granted only if the court determines that the evidence is legally insufficient to allow the jury to decide the case. The jury here must determine the meaning of the warning signs and whether the signs alone establish that the man's behavior was unduly risky. A reasonable jury might conclude that the warning signs were designed to keep bikers out of the area for reasons other than risk, given no additional evidence as to why the signs were posted or of other events in which harm occurred to those ignoring the signs.

A retirement fund filed a suit in federal district court against a corporation under ERISA, a federal statute, for contributions that the corporation allegedly owed to the fund on behalf of three employees. The fund served the summons and complaint on the corporation, and the corporation filed an answer denying liability for the contributions. The parties engaged in discovery, during which the corporation deposed the three non-party employees. According to the employees' deposition testimony, the corporation did not owe any contributions to the fund on behalf of the employees. The corporation subsequently filed a motion to dismiss for failure to state a claim upon which relief can be granted, and included the deposition transcripts of the three employees in support of its motion. The fund claimed it had additional evidence to support its complaint. Did the corporation follow proper procedure in filing the motion to dismiss? ANo, because the motion to dismiss is no longer timely. BNo, however, the court may convert it to a motion for summary judgment. CYes, because the defense of failure to state a claim is never waived. DYes, because the corporation has evidence that no contributions are owed, and therefore the lawsuit should be dismissed.

No, however, the court may convert it to a motion for summary judgment. Discussion of correct answer:A motion to dismiss under Fed. R. Civ. P. 12(b) must be made before or along with a responsive pleading. All of the listed defenses, except for failure to state a claim upon which relief can be granted or failure to join a necessary party, are waived if not raised at that time. If a party later files a motion to dismiss based on failure to state claim, and the party includes material not contained in the pleadings, then under Fed. R. Civ. P. 12(d), the court may simply convert the motion to dismiss into a motion for summary judgment. The court can then consider the evidence filed along with the motion, and it will allow all other parties an opportunity to respond with their own evidence. This answer choice correctly states that the corporation did not follow proper procedure because, if it actually intended a motion to dismiss, it should not have included outside evidence. If it intended to include materials outside the pleadings, it should have correctly captioned its motion as a motion for summary judgment.

A landowner owned a piece of property, which he intentionally left vacant. The owner then conveyed the property to his friend "for so long as no buildings are built on the property, then to [an environmental group]." Two years later, the friend built a small shed on the property. The environmental group informed the friend this action violated the condition and the property belonged to the group. Is the environmental group entitled to the property? ANo, the shifting executory interest violates the Rule Against Perpetuities, so the property reverts to the original landowner. BNo, the shifting executory interest violates the Rule Against Perpetuities, so the friend has a fee interest in the property. CYes, because the charity-to-charity rule makes the shifting interest valid despite the Rule Against Perpetuities. DYes, because a charitable trust is not subject to the Rule Against Perpetuities.

No, the shifting executory interest violates the Rule Against Perpetuities, so the property reverts to the original landowner. Discussion of correct answer:The shifting executory interest to the environmental group will violate the Rule Against Perpetuities, because the condition could be violated more than 21 years after the interest was created. Thus, the part of the gift that violates the Rule is treated as void, but the remainder of the gift is intact. This leaves the friend with a fee simple determinable estate with a possibility of reverter in the original landowner, which may be implied. Because the friend here violated the condition, the property reverts back to the original landowner.

A producer contracted to pay an inexperienced performer a specified salary to act in a small role in a play the producer was taking on a six-week road tour. The contract was for the duration of the tour. On the third day of the tour, the performer was hospitalized with a stomach disorder. The producer replaced her in the cast with an experienced actor. One week later, the performer recovered, but the producer refused to allow her to resume her original role for the remainder of the tour. In an action by the performer against the producer for breach of contract, which of the following, if proved, would be the producer's best defense? AThe actor, by general acclaim, was much better in the role than the performer had been. BThe actor was the only replacement the producer could find, and the actor would accept nothing less than a contract for the remainder of the six-week tour. CThe producer offered to employ the performer as the actor's understudy for the remainder of the six-week tour at the performer's original salary, but the performer declined. DBoth the producer and the performer knew that a year earlier the performer had been incapacitated for a short period of time by the same kind of stomach disorder. Copyright © 2013 by the National Conference of Bar Examiners.

The actor was the only replacement the producer could find, and the actor would accept nothing less than a contract for the remainder of the six-week tour. Discussion of correct answer:After the performer became ill, the temporary impracticability doctrine excused the performer's contractual obligation and also gave the producer the right to suspend his performance obligation during the period that the performer's illness prevented her from acting. The critical issue here is whether the producer also had the right to cancel the contract. Circumstances that would give the producer the right to cancel the contract include the degree of uncertainty relating to the nature and duration of the performer's illness and the extent to which a delay in making substitute arrangements would have prevented the producer from continuing the tour. The unwillingness of the actor, the only replacement available, to take a contract for less than the remainder of the six-week tour and the uncertainty surrounding when the performer might return to work would have discharged the producer's performance obligations and justified his cancellation of the contract with the performer.

After being forcibly ejected from a casino, a tourist brought a federal diversity action against the casino, seeking $105,000 in damages. The tourist claimed that the casino's security guard had used excessive force when ejecting him from the casino, causing severe personal injuries. At trial, the tourist's attorney attempted to introduce testimony of the security guard's ex-wife, who had filed several complaints of spousal abuse against the security guard. The casino's attorney objected that the evidence was irrelevant and unfairly prejudicial. The court overruled the objection, and the ex-wife testified. The jury returned a verdict for the tourist, awarding him $82,000 in damages. The casino's attorney believes that the ex-wife's testimony was unfairly prejudicial and should not have been admitted. What is the best way for the casino's attorney to attack the verdict? AAppeal, arguing that the trial court erred in admitting the testimony, which was so prejudicial that the judgment should be reversed. BMove for a new trial, arguing that the court erred in admitting the testimony and that a new trial without the testimony is necessary. CMove for judgment as a matter of law, arguing that the court erred in admitting the testimony and that without the testimony, the evidence was legally insufficient to find for the tourist. DMove for relief from judgment, arguing that the court's admission of the testimony was a mistake. Copyright © 2020 by the National Conference of Bar Examiners. All rights reserved.

The correct answer is: Move for a new trial, arguing that the court erred in admitting the testimony and that a new trial without the testimony is necessary. Discussion of correct answer: If the testimony was improperly admitted, that is an error that can most immediately be urged on the trial court via a motion for a new trial.

A small commercial airplane crashed in State A. The passengers and pilot, all citizens of State B, were killed in the crash. The airline that owned and operated the airplane is incorporated and has its maintenance facilities and principal place of business in State C. One day before the statute of limitations on their claims would have run, the estates of the pilot and each of the passengers filed a wrongful death action against the airline in federal court in State A. The airline was served one week later and wants to prevent the State A federal court from hearing the action. Which of the following motions is most likely to accomplish the airline's goal? AA motion to dismiss the action for improper venue. BA motion to dismiss the action for lack of personal jurisdiction. CA motion to dismiss the action under the doctrine of forum non conveniens. DA motion to transfer the action to a federal court in State C.

The correct answer is:A motion to transfer the action to a federal court in State C. Copyright © 2019 by the National Conference of Bar Examiners. All rights reserved. Discussion of correct answer:Answer (D) is correct. A federal court may transfer an action to any other district in which it might have been brought "for the convenience of parties and witnesses, in the interest of justice" [28 U.S.C. Sec. 1404(a)]. Out of the available choices, a motion to transfer to State C is most likely to avoid the federal court in State A.

To improve the quality of rental housing within its boundaries, a city proposed an ordinance requiring all new and existing rental housing units to provide at least one full bathroom for each bedroom, plumbing and electrical hookups for a washer and dryer, and a covered parking space. A majority of the owners of existing rental housing in the city opposed the ordinance. They argued that it would dramatically decrease the number of low-income rental housing units because owners would be unable to raise rents enough to recoup the investment required to comply. Without denying these contentions, the city enacted the ordinance. A plaintiff who owns low-income rental housing has sued the city, claiming only that the ordinance is unconstitutional on its face. Which of the following best states the burden of persuasion in this action? AThe city must demonstrate that the ordinance is necessary to serve a compelling state interest, because it adversely affects the fundamental right of rental housing owners to use their property in the manner they deem most economically efficient. BThe city must demonstrate that the ordinance is necessary to serve a compelling state interest, because it will have a substantial and disproportionate negative impact on low-income persons. CThe plaintiff must demonstrate that the ordinance is not substantially related to an important state interest, because it requires some owners of rental housing to invest money that they will not be able to recoup from increased rents. DThe plaintiff must demonstrate that there is no rational relationship between the ordinance and any legitimate state interest, because the ordinance regulates economic activity of a type normally presumed to be within state regulatory authority.

The correct answer is:The plaintiff must demonstrate that there is no rational relationship between the ordinance and any legitimate state interest, because the ordinance regulates economic activity of a type normally presumed to be within state regulatory authority. Copyright © 2019 by the National Conference of Bar Examiners. All rights reserved. Discussion of correct answer:Answer (D) is correct rational scrutiny applies. There are some situations where constitutional rights are strongly protected and courts carefully scrutinize government actions that infringe on those rights. For example, courts give strong protection to fundamental rights like privacy and travel, and they apply heightened scrutiny to suspect characteristics like race and quasi-suspect characteristics like gender. Outside of these special areas of strong constitutional protection, however, courts are generally very deferential to the government. Courts assume that if legislative or executive officials make bad decisions, they will pay a political price for it in elections. Courts therefore do not need to aggressively police most types of government actions, and courts apply a weak form of scrutiny that merely requires the government to have some legitimate, rational basis for its actions. In this question, the new housing ordinance may be controversial, but it does not infringe fundamental constitutional rights or discriminate on any of the bases that trigger heightened scrutiny. Rational-basis scrutiny therefore applies.

A stockyard, a State X corporation, provided beef to a large fast food chain which was a State B corporation. One of their shipments contained contaminated beef causing damages in excess of $500,000. The fast food corporation filed an action against the stockyard in the U.S. District Court in State X, to recover their damages. At the time the contaminated shipment of beef was made, an attorney and member of the fast food chain's board of directors prepared and sent the following memorandum to the president of the fast food chain: "I have credible information that this last beef shipment by the stockyard may be contaminated. I would advise a thorough inspection before we accept this shipment." The stockyard filed a timely motion for the pretrial production of the attorney's memorandum. If the president of the fast food chain asserts the work-product privilege in response to the stockyard's motion to compel production of the attorney's memorandum, how should the reviewing court rule? AThe court should deny the stockyard's motion to compel, because the memorandum reflects the attorney's recommendation of a course of action for the fast food chain to take. BThe court should deny the stockyard's motion to compel, because all documents written by licensed attorneys are privileged. CThe court should grant the stockyard's motion to compel, because the memorandum is relevant to his action. DThe court should grant the stockyard's motion to compel, because the attorney is not representing the fast food chain in the present action.

The court should grant the stockyard's motion to compel, because the memorandum is relevant to his action. Discussion of correct answer:The attorney's memorandum to the president of the fast food chain is probably not within the work-product privilege, because there is no indication that it was prepared in anticipation of litigation. Rather, it appears to be an internal memorandum by one member of the board of directors to the president of the corporation.

A woman sued a high-profile politician in federal district court for sexual harassment in a case based on diversity jurisdiction. The applicable court rules in the forum state gave discretion to the presiding judge to allow television cameras to film the proceedings, but required the judge to balance the needs of justice with the public interest in the proceedings. The judge decided to allow television cameras to film the proceedings. At trial, the politician argued that state law dictated that he had immunity from the suit based on his political office, but the judge disagreed and found that the law cited by the politician provided no such immunity. The jury returned a verdict in favor of the woman. The politician appealed on the grounds that the judge committed errors in allowing television cameras and in denying him immunity. How will an appellate court approach each decision?

The court will affirm the camera decision absent some indication the judge abused his discretion, and will review the immunity decision de novo. Discussion of correct answer:Decisions of a trial judge that are neither conclusions of law nor findings of fact will be affirmed absent some indication that the court abused its discretion. Because the decision regarding the cameras in court falls into this category, this answer choice correctly states the review standard. Furthermore, conclusions of law, such as the decision regarding the politician's immunity argument, will be reviewed de novo.

A man purchased a house that needed substantial repairs. The man financed the purchase of the house by borrowing funds from a bank. He signed a note to the bank and secured repayment of the loan with a mortgage. After purchasing the house, the man borrowed money from his credit union to make the repairs. The man signed a note to the credit union; this note was also secured by a mortgage on the house. Both mortgages were promptly recorded in the order granted. The man was personally liable on both loans. The man moved into the house and made the necessary repairs. He later defaulted on the debt to the bank, and the bank initiated judicial foreclosure proceedings, naming both the man and the credit union as parties to the action. An outside party acquired the house at the foreclosure sale. After the expenses of the sale and the balance due the bank have been paid, $5,000 remains in the sale proceeds. The outstanding balance of the credit union loan is $20,000. The man and the credit union both claim the $5,000. There is no applicable statute. Who is entitled to the $5,000? AThe credit union, because the credit union has priority. BThe credit union, because the man is personally liable on the debt. CThe man, because of his equitable right of redemption. DThe man, because the outside party received title to the house subject to the second mortgage.

The credit union, because the credit union has priority. Discussion of correct answer:(A) is correct. The proceeds of a foreclosure sale are distributed in the following order: (1) to the costs of the sale; (2) to the security interest foreclosed; (3) to the junior lienholders terminated by the sale; and then (4) to the mortgagor, if any proceeds remain. Here, the credit union would be entitled to the $5,000 because it is owed $20,000 and it should be paid before any proceeds go to the man.

A landowner died, validly devising his land to his wife "for life or until remarriage, then to" their daughter. Shortly after the landowner's death, his daughter executed an instrument in the proper form of a deed, purporting to convey the land to her friend. A year later, the daughter died intestate, with her mother, the original landowner's wife, as her sole heir. The following month, the wife remarried. The wife then executed an instrument in the proper form of a deed, purporting to convey the land to her new husband as a wedding gift. Who now owns what interest in the land? AThe daughter's friend owns the fee simple. BThe wife owns the fee simple. CThe wife's new husband has a life estate in the land for the wife's life, with the remainder in the daughter's friend. DThe wife's new husband owns the fee simple.

The daughter's friend owns the fee simple. Discussion of correct answer:The landowner's wife had a determinable life estate, evidenced by the words "for life" and "until remarriage" in the landowner's will. The daughter had a vested remainder and an executory interest. Both of the daughter's interests could be assigned to the friend. On the remarriage of the landowner's wife, the wife's determinable life estate ended and the land automatically went to the holder of the future interest, the daughter's friend.

A textile merchant acquired 1,000 bolts of stretch velvet. The textile merchant knew that a famous women's designer planned to center the designer's spring collection on stretch velvet. The designer planned to use the fabric in the designer's maillots, cat suits and short dresses. The merchant also knew that an unexpected textile strike had resulted in a shortage of the fabric. The merchant sent a text message to the designer, saying: "I am sending you 1,000 bolts of stretch velvet at $50 per bolt." The designer, upon receipt of the merchant's text, phoned the merchant and said, "I accept your offer." The merchant shipped the first 300 bolts to the designer. After using the first 300 bolts, the designer phoned the merchant and repudiated the contract. If the merchant sues the designer for breach of contract to take 1,000 bolts of stretch velvet and the designer defends by pleading the Statute of Frauds, which of the following is the most accurate statement of the law? AThe designer is liable for 300 bolts, but the defense is good as to the remaining 700 bolts. BThe defense fails, because, as between merchants, the merchant's text was a memorandum sufficient to meet the requirements of the Statute of Frauds. CThe defense fails, because the Statute of Frauds does not apply to this transaction. DThe defense is good as to all 1,000 bolts.

The designer is liable for 300 bolts, but the defense is good as to the remaining 700 bolts. Discussion of correct answer:Under the Statute of Frauds, a contract to sell goods priced at $500 or more must be in writing. However, an oral contract for such sale is enforceable to the extent that the goods have been received and accepted. Since the designer accepted the first 300 bolts, the designer will be liable to the merchant only for the price of the first 300 bolts.

An usher at a grand old movie theater learned that the theater's manager had recently inherited a rare print of the usher's favorite movie. The usher asked the manager if he could borrow it. The manager declined, but said he would bring the print into the theater so the usher could see it. The usher had been reading up on the subject of ESP quite a bit, and mistakenly believed there was an extremely high likelihood that he could move large objects with his mind. When the manager brought the print into the theater, the usher pointed up to the enormous chandelier above the manager's head and said that if the manager did not give him the print, he would use the power of his mind to make the chandelier fall on the manager's head. The manager looked up at the massive chandelier, laughed and told the usher he was insane to think he could move the chandelier with his mind. Nevertheless, the manager handed the print over to the usher, who was relieved, as he never had any intention of using his self-perceived powers of ESP to hurt the manager. While the usher had intended only to keep the print temporarily and then return it to the manager, he enjoyed it so much that decided to keep it permanently. The usher was charged with robbery. What is the defendant's best defense to the charge of robbery? AThe manager did not fear that the chandelier would fall on him. BThe usher did not have the ability to make the chandelier fall on the manager. CThe usher never intended to use force against the manager. DThe usher did not intend to deprive the manager of the print permanently at the time of the taking.

The manager did not fear that the chandelier would fall on him. Discussion of correct answer:Robbery consists of all the elements of larceny, plus two additional elements: (1) the taking must be from the person or in the presence of the person; and (2) the taking must be accomplished by force or violence or by intimidation of the threat of violence. If the robbery is by threat of violence, the victim must be in actual fear at the time of the taking. Here, the manager indicated he was not in fear of the usher using ESP to hurt him. Thus, while the usher may well be guilty of larceny, he would not be guilty of robbery.

Which is not a proper action following a mortgagor's default to a mortgagee? AThe mortgagor may redeem the property by paying the amount required to bring the loan current at any time before the foreclosure sale. BThe mortgagee may foreclose on the property and, following a proper sale, sue the mortgagor for any deficiency still outstanding on the obligation. CThe mortgagee may accept title to the property in lieu of foreclosure. DThe mortgagor may only redeem the property by paying to the mortgagee the full amount of the obligation, at any time before the foreclosure sale

The mortgagor may only redeem the property by paying to the mortgagee the full amount of the obligation, at any time before the foreclosure sale. Discussion of correct answer:The equitable right of redemption gives the mortgagor the ability to redeem the mortgage, and so stop a pending foreclosure action, by paying the amount required to bring the loan current. If the parties have agreed to an acceleration clause, then this amount may be the entire amount of the debt. However, the mortgagor must only pay the full amount if such a clause exists. Absent this, the mortgagor need only pay the amount in arrears plus any associated fees.

Congress recently enacted the Reproductive Health Act, and appropriated funds for the implementation of health education and clinical programs associated with sex education and pregnancy-related medical treatments. The sex-education programs were to provide clients with information about the prevention of sexually transmitted diseases, as well as information about all aspects of pregnancy prevention and termination. The president, who felt that homosexuality was immoral, ordered all executive branch personnel to refrain from discussing AIDS-related aspects of health care on the job. A physician's assistant working for the National Centers for Disease Control challenged the constitutionality of the president's order. How is the court likely to find on the question of the constitutionality of the president's order? AThe order is constitutional, because the president has authority over executive branch employees. BThe order is constitutional, because the president has substantial discretion in executing the laws. CThe order is unconstitutional, because it violates the president's duty to faithfully execute the laws. DThe order is unconstitutional, because the president cannot refuse to spend funds as appropriated.

The order is unconstitutional, because it violates the president's duty to faithfully execute the laws. Discussion of correct answer:The president's order violates the separation of powers. The Constitution authorizes Congress to make laws and the president to execute the laws. The presidential duty to faithfully execute the laws encompasses the obligation to carry out specific standards set by Congress. Unless the legislative branch gives substantial discretion to the executive branch to determine the manner in which a law is to be executed, the president is bound by Congress's standards. In the case of the Reproductive Health Act, Congress delineated the information and medical treatment to be provided to clients, so the president cannot issue an order in conflict with the congressional standards.

A state election code provided that in a special-purpose election for the Board of Directors of the County Water District, only the landowners could vote, and their voting was in proportion to the assessed valuation of their land. The sole purpose of the County Water District was to provide for storage and distribution of water for farming. Thus, the landowners were the only ones directly affected by the outcome. The best argument in support of the statute and against the application of the "one person, one vote" principle in this situation is which of the following? AThe principle applies only to elections of individuals to a statewide public office. BThe principle does not apply where property rights are involved. CThe principle does not apply, because the actions of such a district principally affect landowners. DThe principle does not apply, because of rights reserved to the states by the Tenth Amendment.

The principle does not apply, because the actions of such a district principally affect landowners. Discussion of correct answer:In general, the right to vote on the basis of "one person, one vote" is a fundamental right guaranteed by the Equal Protection Clause of the Fourteenth Amendment. However, the Supreme Court has held that this principle does not apply to limited-purpose elections [Ball v. James, 451 U.S. 355 (1981)]. Here, the election is a special purpose election for a water district that affects only landowners in the district. Thus, the "one person, one vote" rule can be suspended under these circumstances, and this choice expresses the limited purpose exception to the rule.

A state whose major industry is the manufacture of metals enacts legislation providing that "industrial businesses shall have preference over all other businesses in the allocation of water in times when drought conditions exist as defined by the state's Department of Water Resources." With increasingly hot and dry summers, communities around the state have found it necessary to conserve water and institute water restrictions. A business owner has run a water theme park for a number of years in the state. Because of the climate, the business owner can operate the park for only five months of each year. Any loss of water supply would cause the business owner to lose his profit for the year. The local town government has voluntary water restrictions in place for the months of June through September in any given year, subject to drought conditions. The business owner, believing that the state statute will result in the total loss of his water theme park business if enforced, challenges the statute in federal court on the ground that it violates the takings clause of the state constitution. If the state files a motion to dismiss the business owner's claim, what is the most likely outcome? AThe business owner will prevail, because a controversy exists that must be heard on the merits. BThe business owner will prevail, because he has standing to challenge the state statute. CThe state will prevail, because the federal court does not have jurisdiction over the business owner's action. DThe state will prevail, because the federal court will abstain from hearing the case.

The state will prevail, because the federal court does not have jurisdiction over the business owner's action. Discussion of correct answer:Article III of the U.S. Constitution defines federal court jurisdiction: (1) cases arising under the U.S. Constitution, or under the laws of the United States; (2) cases in which the United States is a party; (3) cases between two or more states, or between a state and the citizens of another state; (4) cases between citizens of different states based on diversity; (5) cases between citizens of the same state claiming lands under grants of different states; (6) cases between a state or its citizens and foreign states or citizens; and (7) cases where ambassadors or other public ministers are affected. The business owner's suit is a case between a citizen of the state and the state government based on a claim that the statute violates the state constitution (specifically, the Takings Clause). Thus, the suit by the business owner does not fall within any of the categories listed in Article III. As such, the federal court does not have jurisdiction to hear the business owner's claim, and the court should dismiss his suit.

A state legislature enacted a statute providing that "any person awarded a state construction contract must employ only citizens of the state and of the United States in performance of the contract." If the state statute is properly challenged as violating the Commerce Clause, which of the following is the weakest defense of the statute's validity? AThe statute will help protect the workers of the state from competition by foreign workers. BThe statute will help assure that workers with jobs directly affecting the performance of public contracts are dedicated to their jobs. CThe statute will help assure a continuously available and stable work force for the execution of public contracts. DThe statute will help assure that only the most qualified individuals work on public contracts.

The statute will help protect the workers of the state from competition by foreign workers. Discussion of correct answer:The fact that the statute protects local workers against foreign competition would be a weak defense to a Commerce Clause challenge. State or local regulation of interstate or foreign commerce is valid unless (1) the law conflicts with valid federal law, or (2) the law discriminates against interstate or foreign commerce, or (3) the law imposes an undue burden on interstate or foreign commerce. This choice would be a weak defense because the state would be conceding that its law burdens interstate or foreign commerce by making it harder for non-local workers to find work.

A manufacturer sued a buyer in federal court for failing to make timely payments under the parties' sales contract. The case was tried to the court solely on documentary evidence. Immediately after the close of the evidence, the judge announced from the bench, "Judgment shall be entered for the manufacturer," and judgment was so entered. The buyer has appealed the judgment. What is the buyer's best argument for persuading the appellate court to reverse the judgment? AThe judgment is clearly erroneous because it was based solely on documentary evidence. BThe manufacturer was required to file proposed findings and conclusions before the trial court ruled. CThe trial court erred because it announced the judgment without giving the parties an opportunity to submit proposed findings and conclusions. DThe trial court erred by not providing findings and conclusions.

The trial court erred by not providing findings and conclusions. Copyright © 2019 by the National Conference of Bar Examiners. All rights reserved. Discussion of correct answer:Answer (D) is correct. In an action tried on the facts without a jury or with an advisory jury, the judge is required to find the facts specially and state his or her conclusions of law separately. The findings and conclusions may be stated on the record after the close of evidence or may appear in an opinion or a memorandum of decision filed by the court [Fed. R. Civ. P. 52(a)(1)]. Without such a statement, an appellate court would not be able to determine why the judge ruled in favor of the manufacturer, and so reversal on these grounds would be appropriate [United States v. Forness, 125 F.2d 928 (1942)].

A landlord of an apartment building received frequent complaints about a tenant who often had loud, all-night parties and who was rude to other residents in the common areas of the building. The landlord successfully pursued an eviction proceeding but the tenant refused to vacate and the police failed to intervene. The landlord decided to pump a massive amount of pesticide into the tenant's apartment through the air conditioning duct, knowing that the highly poisonous chemicals would drive the tenant out, as staying in the apartment would be deadly. Nevertheless, the tenant stayed in the apartment, although he did become quite ill and was eventually taken to the hospital. He remained sick as the pesticide slowly poisoned his body. Two years later, the tenant died as a result of the slow-working effects of the pesticide. This jurisdiction follows all of the traditional common law rules. If the landlord is charged with common law murder, what is his best defense? AThe tenant had no legal right to be in the apartment, which was owned by the landlord. BIt was not foreseeable that the tenant would remain in the apartment despite the presence of the pesticide. CThe landlord did not intend to kill the tenant, but rather to force him to vacate the premises. DThe victim died two years after the act that caused his death.

The victim died two years after the act that caused his death. Discussion of correct answer:Although the landlord's act of pumping the pesticide into the apartment was the factual cause of the tenant's death, at common law, if the victim died more than one year and one day after the defendant's act, the courts would rule that the defendant's act was not the proximate cause of the killing. Note that most states have eliminated this rule or have extended the period within which the defendant is held legally responsible. However, the question specifies that this jurisdiction follows all of the traditional common law rules, which would include the year-and-a-day rule.

The legislature of State Z is engaged in debates on the question of regulating acupuncturists. A well-respected practitioner of this specialty became enraged during the heated debate and, in a spontaneous outburst, shouted from the balcony that all the legislators in favor of new, extremely rigorous licensing requirements for acupuncturists are "pencil-necked, spineless jerks who could use a needle up their bums." The legislators promptly acted in a display of their power to adopt a rigorous licensing law, with a special provision revoking his license to practice as an acupuncturist. When the acupuncturist challenges the revocation of his license, what is the most likely result? AThis is an invalid action in violation of his privileges and immunities under Article IV, Section 2. BThis is invalid, because it deprives him of the right to engage in interstate commerce. CThis is a bill of attainder, and therefore invalid. DThis is valid, because a license is only a privilege, and thus there is no right to procedural due process.

This is a bill of attainder, and therefore invalid. Discussion of correct answer:A law that applies to named individuals or an easily ascertainable group in a manner that inflicts a punishment without a judicial trial is an unconstitutional bill of attainder. Here, the state legislature passed a licensing law which expressly revoked the acupuncturist's license without the benefit of trial. Since he was singled out by name in the law and punished without trial, the State Z law is an unconstitutional bill of attainder.

A homeowner gave a mortgage on a property to a third-party lender for $4 million, $2.5 million to buy the property and $1.5 million to make improvements on it. Which of the following is true? AThis mortgage is not a purchase-money mortgage, because a portion was used to make improvements on the land. BThis mortgage is not a purchase-money mortgage, because the mortgage was given as part of the same transaction in which the homeowner acquired title to the property. CThis mortgage is a purchase-money mortgage, and as such, it does not have priority over any pre-existing liens on the property. DThis mortgage is a purchase-money mortgage, and as such, it has priority over all prior liens that attach to the property through the actions of the homeowner.

This mortgage is a purchase-money mortgage, and as such, it has priority over all prior liens that attach to the property through the actions of the homeowner. Discussion of correct answer:A purchase-money mortgage is a mortgage given to a vendor of the real estate or to a third-party lender to the extent that the loan proceeds are used to acquire title to the real estate. A purchase-money mortgage may also be used to construct improvements on the real estate, if the mortgage is given as part of the same transaction in which title is acquired. Here, the homeowner used the proceeds from the loan to acquire title to the property and to make improvements on the property. Therefore, it is a purchase-money mortgage and will be given priority over a prior lien, such as a judgment lien or a homestead claim, which attaches to the property through the actions of the purchaser-mortgagor, even if the lien was recorded first.

A woman owned a four-unit apartment building and lived in one of the units. When one of her tenants vacated his apartment, the woman placed an advertisement in the local paper that read as follows: "Large two-bedroom apartment available for rent. White male preferred." The woman's preference was motivated by the fact that she liked to have a mix of tenants of both genders and from various racial and ethnic backgrounds in her building, and of the remaining rented units, one was rented to an African American man and the other to a Pacific Islander woman. Based upon these facts, which of the following statements is true? AThe federal Fair Housing Act makes it illegal for the woman to refuse to rent her units to prospective tenants because of their race or gender. BThe woman's motive absolves her from any liability under the federal Fair Housing Act. CThere are no violations of any federal laws under these facts. DUnder the federal Fair Housing Act, the woman was not permitted to state a racial or gender preference in the advertisement.

Under the federal Fair Housing Act, the woman was not permitted to state a racial or gender preference in the advertisement. Copyright © 2019 by the National Conference of Bar Examiners. All rights reserved. Discussion of correct answer:(D) is correct. The federal Fair Housing Act outlaws the refusal to sell or rent a dwelling to any person because of his or her race, color, disability, sex, religion, familial status, or national origin. However, if the building owner has no more than four units and occupies one of the units, the owner is allowed to discriminate. Here, the woman owns four units and lives in one, so she would be allowed to discriminate against potential renters. Even so, the Act also prohibits advertisements that indicate a discriminatory preference in renting. Consequently, the advertisement for white males violated the Act even if the woman could discriminate in who she chose to rent the property to.

A retiree's health had deteriorated so that he found it difficult to conduct his own affairs. The retiree's children convinced him to give a power of attorney to a financial consultant, so that important business transactions could be conducted on the retiree's behalf without any of the children gaining undue influence over his wealth. The power of attorney stated in part, "I hereby authorize the financial consultant to sell and convey all or part of my real property." In order to provide necessary capital for some of the retiree's business operations, the financial consultant sold and conveyed his country estate to a corporation via a general warranty deed. After the transaction had closed and the deed executed by the financial consultant had been delivered to the corporation, the corporation discovered an outstanding mortgage burdening the property, the presence of which violated the covenant against encumbrances. In an action by the corporation against the retiree to recover for the breach of covenant, which of the following best describes the determinative issue of the litigation? AWhether the corporation qualifies as a bona fide purchaser. BWhether a power of attorney including authorization to "sell and convey" includes the power to execute a general warranty deed. CWhether a deed that does not contain any covenants of title is effective to convey the described real property. DWhether the covenant involved is personal or runs with the land in the subject jurisdiction.

Whether a power of attorney including authorization to "sell and convey" includes the power to execute a general warranty deed. Discussion of correct answer:A power of attorney is a form of agency in which the maker authorizes the holder to perform certain acts. The act of selling and conveying property does not necessarily include the further act of making certain promises in connection with such selling and conveying; thus, whether the corporation can recover from the retiree for violation of the covenant against encumbrances depends upon whether the financial consultant's agency under the power of attorney included the power to make the promises contained in the covenants of title that are normally included in a general warranty deed.

A homeowner sued a plumber for damages resulting from the plumber's allegedly faulty installation of water pipes in her basement, which caused flooding. At trial, the homeowner is prepared to testify that when she first detected the flooding, she turned off the water and called the plumber at his emergency number for help and that the plumber responded, "I'll come by tomorrow and redo the installation for free." Is the homeowner's testimony regarding the plumber's response admissible? ANo, because the statement was an offer in compromise. BNo, because it is hearsay not within any exception. CYes, as a subsequent remedial measure. DYes, as evidence of the plumber's fault.

Yes, as evidence of the plumber's fault. Copyright © 2009 by the National Conference of Bar Examiners. All rights reserved. Discussion of correct answer:This is a party admission, admissible as a hearsay exemption under Rule 801(d)(2)(A). A statement made by a party cannot be excluded as hearsay when offered against him by the opponent. Moreover, the statement is probative. A person who makes a statement like this is likely to think he is at fault, and this statement is probative evidence that indeed he was at fault.

Plaintiff Trust Fund sued Corporation in federal district court for delinquent contributions under ERISA, a federal statute. Discovery was extensive and prolonged, with frequent requests for extensions of time. Finally, the court issued an order that the parties complete discovery by the date provided by the court and that all motions be filed within two weeks after the close of discovery. The parties engaged in intense negotiations, hoping to settle the lawsuit. In order to keep the negotiations going, Fund complied with Corporation's request to hold off in engaging in discovery. Fund therefore did not complete the discovery it needed prior to the deadline, and Fund filed no motions. The deadlines passed. The court ordered the parties to provide a status update. Neither party responded. Fund was still attempting to obtain discovery from Corporation. Two weeks after the deadline to provide the status update passed, Corporation filed a motion to dismiss for failure to prosecute. Will the court grant Corporation's motion to dismiss for failure to prosecute? AYes, because Fund failed to comply with the court's orders regarding close of discovery and the status update. BYes, because Fund needed additional evidence to prosecute its case. CNo, because the parties were engaging in settlement negotiations. DNo, because Fund was seeking to complete discovery, and was therefore continuing to prosecute the case.

Yes, because Fund failed to comply with the court's orders regarding close of discovery and the status update. Discussion of correct answer:Simply, if a plaintiff fails to comply with a federal court order, the plaintiff puts itself in danger of dismissal for failure to prosecute, loss of any associated request, and/or sanctions, depending on the circumstances of the case. Here, although the plaintiff had engaged in settlement negotiations with the defendant, it did not involve the court in those negotiations or follow the deadlines and orders put forth by the court.

A customer bought a can of corn at a grocery store. While eating the corn later that evening, the customer was injured by a small piece of glass in the corn. The customer sued the canning company that had processed and canned the corn. At trial, the customer presented evidence that neither the customer nor any third party had done anything after the can of corn was opened that would account for the presence of the glass. Without any other evidence, is the customer likely to prevail? ANo, because it is possible that someone tampered with the can before the customer bought it. BNo, because the customer has not shown any direct evidence that the canning company acted negligently. CYes, because a jury may reasonably infer that the canning company acted negligently. DYes, because the grocery store could not have discovered the piece of glass by reasonable inspection. Copyright © 2019 by the National Conference of Bar Examiners. All rights reserved.

Yes, because a jury may reasonably infer that the canning company acted negligently. Discussion of correct answer: Answer (C) is correct because the doctrine of res ipsa loquitur would allow a jury to infer that the canning company was negligent. A plaintiff asserting a negligence claim generally needs to prove what the defendant did that was negligent. In some instances, though, a plaintiff may not be able to determine exactly what happened that was negligent, but the circumstances will suggest that the defendant probably was negligent because the accident and injury otherwise would not have occurred. Here, the customer cannot determine exactly how the glass got into the can, but one could infer that the canning company must have been negligent in some way because otherwise, there would not have been glass in the corn. The customer presented evidence ruling out the possibility that the glass got into the corn after the can was opened. That leaves a sound basis for inferring that the canning company probably was negligent in some manner.

A prominent private university runs into financial problems and is purchased by the state government using public funds. The state government operates the university as part of its public university system, and changes its tuition rate to allow students who live in the state at the time of their application for free. Students living outside the state when they applied for admission must pay tuition, which amounts to tens of thousands of dollars per year. A group of prospective students from out-of-state have challenged the constitutionality of the new tuition policy under Article IV. Is the tuition policy constitutional? AYes, because a state has plenary power to discriminate against nonresidents when it is distributing benefits. BYes, because college education is not a fundamental right. CNo, because it violates the Equal Protection Clause. DNo, because it violates the Privileges and Immunities Clause.

Yes, because college education is not a fundamental right. Discussion of correct answer:The Privileges and Immunities Clause prevents states from discriminating against nonresidents with respect to "fundamental rights" or "essential activities." However, it may discriminate against nonresidents in the provision of benefits that do not implicate fundamental rights. The Supreme Court has never held that college education is a fundamental right, and thus the tuition policy is consistent with Article IV.

An investor was brought to trial for his participation in a scheme in which his victims were invited to become distributors for a diet pill manufacturer and were given bonus payments, after making a significant investment of their own, for bringing additional distributors into the chain. The investor's defense is that he was brought in by the vice president of the manufacturer and was just following his boss's instructions. The prosecution plans to call one of the distributors that the investor brought in with the promise of huge commissions. That victim will testify that while the investor was on bail, he offered to return her initial investment with interest if she would have a convenient lapse of memory concerning the incident. Should this testimony be admitted? AYes, as evidence of the investor's dishonesty. BYes, because it is probative of the investor's consciousness of guilt. CNo, because as a matter of policy, offers of compromise should not be admitted. DNo, because it carries a high degree of prejudice.

Yes, because it is probative of the investor's consciousness of guilt. Discussion of correct answer:This is admissible to show consciousness of guilt, because the jury may infer the investor would not try to silence a witness if he were not guilty and fearful of that witness's testimony. The rule barring an offer of settlement as an admission of guilt does not apply the same way here as it normally would because this type of "settlement" is not encouraged by social policy--in fact, it is witness tampering.

A real estate holding company had several subsidiaries that owned different types of real estate, including one that owned commercial properties in a particular city. The holding company sued several city commercial tenants in federal court over their failure to pay yearly fees under a federal regulatory scheme. The commercial tenants alleged that the fees were included in their previously negotiated commercial rents, but the court found for the holding company. Three months later, the defendants filed a motion under Federal Rule 60(b)(4) for relief from a judgment on the basis that the subsidiary was the proper plaintiff. Should the court grant the motion? ANo, because the defendant did not raise the issue during the suit. BNo, because the holding company consented to jurisdiction when it filed the suit. CYes, because the holding company lacked standing. DYes, because the motion is timely.

Yes, because the holding company lacked standing. Discussion of correct answer:Many bases for jurisdiction are waivable. A party who is not otherwise subject to personal jurisdiction can subject himself to it by appearing in court and not raising the issue. However, standing is not waivable. Here, a party who had no right to relief sued on behalf of a party who did. Even though neither party raised that issue at trial, the lack of standing made a Rule 60(b)(4) motion to vacate a void judgment appropriate.

A reliable informant was called into the police station to provide information about gang street violence. The informant told detectives that six months prior he had witnessed a meeting between three rival gangs. The gang members discussed a cache of weapons located in the defendant's basement. The informant also observed the gang members in possession of weapons at the meeting. The detective prepared an affidavit and obtained a search warrant for the basement. The search turned up the arsenal of unregistered weapons as described by the informant. The defendant was arrested and charged with unlawful weapons possession. If the defendant files a motion to suppress the weapons based on a lack of probable cause to secure the warrant, will the court grant the motion? ANo, because the weapons seized matched the informant's description. BNo, because the tip was from a reliable informant who witnessed the matters described in the affidavit. CYes, because the informant's information was stale. DYes, because the police never observed the weapons in the basement and the tip was based solely on a hearsay account.

Yes, because the informant's information was stale. Discussion of correct answer:Under Illinois v. Gates [462 U.S. 213 (1983)], the judicial officer issuing the warrant must determine if there is a "substantial basis" for concluding that the evidence of criminality will be at the particular place described on the warrant application. The informant's observations were made six months prior to the warrant application and the likelihood that the guns are still in the basement is diminished because the information is stale.

A cowboy and a developer owned adjacent ranches. The cowboy's ranch, Ranch A, was to the north of the developer's ranch, Ranch B, and deeper into the canyon; the only public road in the area ran along the southern border of the developer's ranch. For years, the cowboy and the developer had an understanding that the cowboy could use a 10' wide dirt road along the western edge of the developer's ranch for ingress and egress to the cowboy's ranch from the public road. In 2022, an oil tycoon bought both ranches but continued to operate them as separate entities. The foreman and hands of Ranch A continued to use the dirt road across Ranch B to get from Ranch A to the public road. In 2032, the oil tycoon sold Ranch A to a baker; included in the deed was the following: "The baker, his heirs, and assigns shall have the use of the dirt road running along the western border of Ranch B for ingress and egress to Ranch A." In 2044, a scientist bought Ranch B from the oil tycoon. The scientist subsequently obtained official approval for and filed a subdivision map creating a planned residential community thereon. The area occupied by the dirt road was designated a public park, with fountains, children's play equipment, and a baseball diamond. When the baker learned of the scientist's plans, he instituted an appropriate action to enjoin obstruction or obliteration of the dirt road. Will the baker obtain the injunction? AYes, because the owner of the servient estate is not permitted to obstruct an express easement. BYes, because the baker has obtained an easement by necessity over the dirt road. CNo, because the baker will be held to his remedy at law--damages. DNo, because circumstances have changed substantially since the easement over the dirt road was created.

Yes, because the owner of the servient estate is not permitted to obstruct an express easement. Discussion of correct answer:The baker has an express easement over the dirt road on Ranch B, granted to him along with Ranch A by the deed from the oil tycoon. The owner of the servient estate is not permitted to obstruct or otherwise interfere with the owner of the dominant estate's reasonable use of the easement. Therefore, the baker will be able to enforce his right to use the dirt road by obtaining an injunction against the planned obstructions.

A State A woman was severely injured by a vacuum cleaner she purchased from a State B manufacturer and she filed a product liability claim against the manufacturer in federal district court. After the close of evidence, lawyers for the woman and the manufacturer submitted requests for jury instructions. Prior to the parties' closing arguments in front of the jury, the court gave each party proposed instructions. After the closing arguments to the jury, the court gave the parties a chance to object to the proposed jury instructions outside of the jury's hearing. The manufacturer argued that the jury instructions were overly confusing, but the judge decided to finalize the instructions and gave the jury the instructions just prior to the jury being discharged. The manufacturer now argues that the judge's procedure in administering the jury instructions was faulty. Will the manufacturer prevail? ANo, because a jury may be instructed of the instructions at any point until the jury is discharged. BNo, because the parties were given a chance to object outside of the jury's hearing. CYes, because the jury was not instructed of the instructions prior to the final arguments. DYes, because the parties were not given a chance to object prior to the final arguments.

Yes, because the parties were not given a chance to object prior to the final arguments. Discussion of correct answer:Pursuant to Rule 51(b), the court must give the parties a chance to object to the proposed jury instructions prior to the final arguments. Here, this did not take place until after the arguments were given.

A company incorporated and headquartered in State A sued a plumber domiciled in State B in a federal court in State A, alleging that the plumber had negligently installed pipes in a manner that resulted in $250,000 in damage to the company's headquarters building. In response to the complaint, the plumber filed a motion to dismiss for lack of personal jurisdiction. The court denied the motion. Thereafter, the plumber did not file an answer or any other response to the company's action. Sixty days after the court's order denying the motion to dismiss, the company asked the clerk to enter default, and the clerk did so. The company applied to the court for the entry of a default judgment and notified the plumber three days before the default judgment hearing. After an ex parte hearing in which the court received evidence on the damages amount, the court entered a default judgment for the full amount sought. Ten days later, the plumber filed a motion to set aside the default judgment. Is the court likely to grant the plumber's motion? ANo, because the court could fix the amount of damages even without hearing the plumber's evidence. BNo, because the plumber failed to plead or otherwise defend against the company's action. CYes, because the plumber was not given adequate notice of the hearing on the company's application for the entry of a default judgment. DYes, because the State A federal court lacked personal jurisdiction over the plumber as a State B citizen.

Yes, because the plumber was not given adequate notice of the hearing on the company's application for the entry of a default judgment. Discussion of correct answer:Answer (C) is correct. If a party against whom a default judgment is sought has appeared personally or by a representative, that party or representative must be served with written notice of the application for default judgment at least seven days before the hearing [Fed. R. Civ. P. 55(b)]. Here, the plumber appeared by filing a motion to dismiss and the company only notified the plumber three days before the hearing. Consequently, the court should grant the plumber's motion.

A trucker was involved in a car accident. He sued a woman who was a mechanic that fixed his truck two hours prior to the accident in federal court for his injuries. The woman responded by asserting a breach of contract claim against the trucker for not paying his bill and filing a third-party complaint against the student who was driving the other car involved in the accident. Can the woman assert the claim against the student? AYes, because the student may be liable to the trucker as well. BYes, because this is a compulsory counterclaim. CNo, because one claim is in contract and one is in tort. DNo, because the plaintiff is master of his or her claim.

Yes, because the student may be liable to the trucker as well. Discussion of correct answer:After a defendant has served her answer, that defendant may bring an impleader claim against a non-party who may be liable for all or part of the plaintiff's claim against the defendant [Fed. R. Civ. P. 14]. The two most common types of impleader claims are for contribution and indemnity. Here, the woman is impleading the student because the student may be jointly liable to the trucker, making this a contribution claim.

The police suspected a woman of dealing in stolen credit cards. An undercover police officer knocked on the woman's door and told her that he was willing to pay for usable credit cards. When the woman asked if he was a police officer, the officer replied, "No," giving her a false name and saying, "You can call Harvey--he'll vouch for me." The woman admitted him to her house. After she left the room to call Harvey, the officer lifted some papers off a desk and underneath discovered three credit cards bearing different names. He seized the cards, and when the woman returned, he arrested her. The woman has moved to suppress the evidence seized on the ground that her Fourth Amendment rights were violated. Should the court grant the motion to suppress? ANo, because it was immediately apparent that the cards were stolen. BNo, because the woman assumed the risk that the officer would seize items in plain sight. CYes, because the woman's rights were violated when the officer lifted the papers. DYes, because the woman's rights were violated when the officer obtained entry by deceit.

Yes, because the woman's rights were violated when the officer lifted the papers. Discussion of correct answer:The Fourth Amendment protects the people from unreasonable search or seizure. A search with a warrant based on probable cause is presumptively reasonable; a warrantless search is presumptively unreasonable. As there was no warrant here, the search for what was in the home and what was under the papers was presumptively unreasonable. However, when police search or seize without a warrant, the presumption of unreasonableness is rebutted if police acted pursuant to an established exception to the warrant requirement. So if an exception is applicable, the search is considered reasonable and does not violate the Fourth Amendment. Here there were two searches (it is very important to analyze each link in the discovery chain to make sure each link complies with the Fourth Amendment). The first search was the entry into the home. This was reasonable because it fell within the scope of the consent exception, which allows a search without probable cause or a warrant based on voluntary consent. Had the officer seen the incriminating credit cards on top of the papers the discovery would have been within the scope of the consensual presence and the immediate probable cause the cards were contraband would have allowed seizure of the cards pursuant to the plain view doctrine. However, the facts indicate the officer found the cards only after moving the papers off of them. When the officer did so, he exceeded the scope of the consent to enter the home (scope is implied by the request: enter the home to engage in a transaction, not to shuffle through papers). The movement of the papers was a new or additional search because it was a physical trespass on the defendant's papers and because the cards were not exposed to the public and therefore were also within the defendant's reasonable expectation of privacy. Because the officer did not have a warrant for that search and because no other exception applies to render that search reasonable, it violated the Fourth Amendment. Therefore the cards were not discovered in valid plain view and must be suppressed.

A father wanted to avoid probate, and so he gave title to the family home, which he owned free and clear of any encumbrances, to his son. In the grant, the father reserved a life estate for himself. The father properly recorded the gift deed to his son. The son wanted to raise some cash for a business venture, and so he altered the deed his father gave him so that it appeared that the son owned the property in fee simple absolute. Based on the altered deed, the son obtained a mortgage on the gifted real estate from a bank who failed to learn of the father's life estate. One year later, the son defaulted on the mortgage. Can the bank foreclose on the son's mortgage on the real estate? AYes, because the father's life estate will be extinguished. BYes, but it can only sell the property subject to the father's life estate. CNo, because the father has a life estate in the property. DNo, because the bank is estopped from foreclosing based on its failure to discover the father's life estate.

Yes, but it can only sell the property subject to the father's life estate. Discussion of correct answer:The son could only mortgage the property interest he owned. Although he had altered the grant deed to so indicate, he did not actually own the property in fee simple absolute when he mortgaged it; he owned a remainder interest in the property. The mortgage did not and could not extinguish the father's life estate. The bank may therefore foreclose on the mortgage and take over the son's title to the property, but can only sell the property subject to the father's life estate.

A wife died from a gunshot wound received while alone at home one evening. Her husband was prosecuted for her murder. After a lengthy trial, the jury acquitted the husband. He then sought to collect under his wife's life insurance policy. The policy contained a double indemnity clause payable in the event the death of the insured resulted directly from bodily injury effected through external violent and accidental means. The husband sued the insurance company when it refused to pay his claim. The husband asserts that the death was within the policy's double indemnity provisions and demands full payment. At trial, the insurance company's lawyer tries to introduce portions of the husband's testimony from his murder trial. The husband's lawyer objects. Is the evidence admissible? ANo, because of double jeopardy. BNo, because of collateral estoppel. CYes, as a statement against interest. DYes, but not through a hearsay exception.

Yes, but not through a hearsay exception. Discussion of correct answer:This is the best choice in this very difficult question. Under Federal Rule of Evidence 804(b)(1), former testimony is hearsay. To be admissible, the declarant must be unavailable in the present proceeding, and the party against whom the former testimony is offered must have had an opportunity to develop the former testimony and a similar motive to develop the former testimony. In this instance, the husband is available, so this hearsay exception cannot apply. However, former testimony may still be admissible for certain purposes without meeting the requirements of unavailability and confrontation when the former testimony is offered for nonhearsay purposes such as to show that the declarant committed perjury, to refresh recollection, or to impeach a witness. It may be used for one of those purposes here.

In a civil trial for fraud arising from a real estate transaction, the defendant claimed not to have been involved in the transaction. The plaintiff called a witness to testify concerning the defendant's involvement in the fraudulent scheme. To the plaintiff's surprise, however, the witness testified that the defendant was not involved and denied having made any statement to the contrary. The plaintiff has now called a second witness to testify that the first witness had stated, while the two were having a dinner conversation, that the defendant was involved in the fraudulent transaction. Is the testimony of the second witness admissible? ANo, because a party cannot impeach the party's own witness. BNo, because it is hearsay not within any exception. CYes, but only to impeach the first witness. DYes, to impeach the first witness and to prove the defendant's involvement.

Yes, but only to impeach the first witness. Discussion of correct answer:Prior statements that are inconsistent with a witness's present testimony impeach the witness's credibility because they tend to show that the witness's trial testimony is not believable. The prior inconsistent statement was not made under oath, and so does not fit the exclusion from hearsay provided by Federal Rule of Evidence 801(d)(1)(A). There is no other hearsay exception that is satisfied under the facts. Therefore the statement is admissible only to impeach the witness and not for its truth.

A retailer brought a federal diversity action against an architect, alleging fraudulent misrepresentations in the architect's design of the retailer's store. The complaint did not include a jury demand. The architect timely moved to dismiss the action for failure to state a claim; he did not file an answer. Twenty days after being served with the motion, the retailer amended the complaint to add a defamation claim based on the architect's recent statements about the retailer in a local newspaper. In the amended complaint, the retailer demanded a jury trial on both claims. Has the retailer properly demanded a jury trial? ANo, because the retailer filed the demand more than 14 days after service of the motion to dismiss. BNo, because the retailer filed the demand more than 14 days after service of the original complaint. CYes, but on the defamation claim only, because the original complaint did not contain a jury demand. DYes, on both claims, because the architect had not answered the original complaint when the retailer filed the amended complaint with the jury demand.

Yes, on both claims, because the architect had not answered the original complaint when the retailer filed the amended complaint with the jury demand. Discussion of correct answer:Answer (D) is correct. If a right to a jury trial exists in an action, any party can exercise the right by filing with the court and serving on the other parties a written demand for a jury trial [Fed. R. Civ. P. 38]. The demand for a jury trial must be made within 14 days after the service of the last pleading directed to the triable issue [Id.]. Here, because the architect has not yet filed an answer to the first or second complaint, the last pleading has not been served on any triable issues. Consequently the 14-day clock has not started running yet.

A homeowner listed his home for sale with a real estate broker. The written six-month exclusive-right-to-sell listing agreement provided for the payment of a commission if the home sold. In accordance with the listing agreement, the broker promptly took reasonable steps to market the home, incurring expenses for her efforts. Five months into the listing period, without involving the broker, the homeowner accepted an offer to purchase from his cousin. The broker learned of the contract only when the sale of the home to the cousin closed, one month after the listing period had expired. Is the broker entitled to any payment? ANo, because the broker engaged in no negotiations with the cousin. BNo, because the closing occurred after the listing period had expired. CYes, but only reimbursement for her expenses. DYes, the full commission, because the homeowner accepted the cousin's offer to purchase during the listing period.

Yes, the full commission, because the homeowner accepted the cousin's offer to purchase during the listing period. Copyright © 2019 by the National Conference of Bar Examiners. All rights reserved. Discussion of correct answer:(D) is correct. The broker is entitled to the full commission because the agreement stated that he had a six-month exclusive right to sell the listing, and the homeowner sold the listing within that six-month period.

A fisherman runs a commercial boatyard in which boat owners store their vessels over the winter. The fisherman sells the boatyard to a retired navy admiral, conveying title by general warranty deed. The retired admiral operates the boatyard for a period of five years, then sells the property to a cruise ship captain, conveying title by quitclaim deed. The cruise ship captain operates the boatyard for a period of one year, at which time he is notified that a grantor prior to the fisherman retains an interest in the warehouse on the grounds of the boatyard and now demands that cruise ship captain vacate the structure. The cruise ship captain is forced to find alternate dry-dock facilities for several yachts housed in the warehouse over the winter. The cruise ship captain confronts the retired admiral and the fisherman about the claimant's interest, but both insist they have no knowledge of any prior claims or encumbrances. One-quarter of the property the cruise ship captain purchased is now unavailable to him, having been taken over by the claimant. The cruise ship captain filed an action seeking damages against the fisherman and the retired admiral. What is the court most likely to award the cruise ship captain?

Damages against the fisherman, because the claimant's ejectment of the cruise ship captain breached the covenant of quiet enjoyment in the fisherman's deed to the retired admiral. Discussion of correct answer: The fisherman conveyed the property to the retired navy man by general warranty deed, which contains the standard covenants of title: the covenant of seisin, of right to convey, against encumbrances, of quiet enjoyment, and of warranty. The covenant of quiet enjoyment warrants that the grantee's possession and enjoyment of the property will not be disturbed by any third party asserting a valid claim to the property. Here, the claimant has disturbed the cruise ship captain's possession and enjoyment of the property by ejecting him from the boatyard's warehouse. The covenant of quiet enjoyment runs with the land; that is, a remote grantee such as the cruise ship captain can sue the original grantor (the fisherman) for damages if a third-party claimant disturbs the grantee's possession and enjoyment. Thus, the court will likely award damages for the fisherman's breach of the covenant.

The day after the defendant was fired from his job, he went to the building where he had worked and waited outside. When he saw his boss through the office window, the defendant fired his shotgun at him, hoping to injure but not kill his boss. The bullet hit the window. However, the building had bulletproof glass. Because there was a noisy thunderstorm in progress, the boss and the others in his office were completely unaware of the attack. Unfortunately for the defendant, a police officer on patrol saw his actions and arrested him. The defendant was charged with assault. What is the likely outcome?

He will be found guilty, because he intended for his shotgun blast to hit someone. Discussion of correct answer: The majority of jurisdictions defines assault as an attempted battery. In attempted battery jurisdictions, actual fear (or even awareness) of the attempt is not necessary to sustain a conviction for assault. Although battery itself is a general intent crime, attempt is a specific intent crime, so actual intent to injure is required. The defendant's act of firing a shotgun at his boss shows an attempt to injure the boss, even though the attempt failed.

A State A citizen was injured in State B when the brakes on the truck that the citizen had rented from a State B rental corporation failed. The citizen sued the corporation in federal district court for $125,000 in damages. The corporation filed an answer generally denying liability. In its motion for summary judgment, the corporation claimed that it was not liable for the full amount of damages because the citizen was contributorily negligent. Will the court allow the corporation to raise contributory negligence?

No, because the corporation waived its defense of contributory negligence. Discussion of correct answer:Pursuant to Fed. R. Civ. P. 12(b), every defense to a claim must be asserted in the responsive pleading if one is required. Thus, an affirmative defense must be pleaded in the initial responsive pleading or it is waived. The first responsive pleading was the corporation's answer. Because the corporation did not raise contributory negligence as a defense in its answer, that defense is no longer available.

A skier had skied for many years. The skier was proceeding down a relatively steep hill on an advanced course by a series of traverses. He was crossing the slope a number of times to diminish the angle of descent — the skiing equivalent of switchbacks in hiking. At the same time a ski instructor, who was also the owner of her own ski school and a member of the ski patrol, came straight down the hill, saw the skier midway across the hill in one of his traverses, and attempted to ski behind the skier. The ski instructor miscalculated the speed at which the skier was traveling, and ran directly into him, fracturing the skier's knee. The ski instructor's conduct will be judged against which of the following standards?

The conduct of a reasonably prudent person with superior skiing knowledge and expertise. Discussion of correct answer:Because the ski instructor is a skiing expert, she will be expected to use her superior knowledge and will be held to the standard of a reasonably prudent person with that special expertise, not merely the standard of a reasonably prudent average skier.

A football team entered into a 10-year lease with a city for use of the city's athletic stadium. Five years into the lease, the team threatened to leave the stadium and move to another city. The city sued the team in federal court, seeking a permanent injunction to prevent the team from breaching its lease and leaving. In its answer, the team included a counterclaim seeking $10 million in damages for losses caused by the city's alleged failure to properly maintain the stadium, as the lease required. The team demanded a jury trial on the counterclaim. The city moved to try its claim for a permanent injunction before the trial on the team's counterclaim. The team objected and moved that the jury trial of its counterclaim be held before the trial of the city's injunction claim. How should the court rule on the parties' motions?

The court should first hold a jury trial of the team's counterclaim, and then a nonjury trial of the issues remaining in the city's claim. Discussion of correct answer: Answer (A) is correct. The right to a jury trial in a civil legal action is guaranteed by the Seventh Amendment to the U.S. Constitution. However, equitable actions, such as an action for an injunction, are not triable by jury as a matter of right. Because the team is making a legal claim for damages, and it is guaranteed a right to a jury trial on this legal issue, the legal claim should be tried in front of a jury first, so the team does not lose its right to a jury trial on the legal issues by prior determination of the city's equitable claims.

A homeowner had a swimming pool installed in his back yard. He often invited neighbors to use the pool on particularly hot days. On one such day one of his neighbors swam to the side of the pool. He placed his hand on the pool light and received a severe shock, causing him to suffer paralysis in his arm which lasted several weeks. The homeowner had recently installed the pool light into the pool when the original pool light had ceased to work. The homeowner had gone to a pool supply store looking for a replacement light and had bought the wrong model. The model pool light the homeowner purchased was manufactured by a light company and was not designed for use in a pool of the size of the one owned by the homeowner. The homeowner had negligently failed to notice that fact in the literature that came with the pool light. The shock to the neighbor was caused by a defect in the wires in the pool light. The light company had bought the wires from a wire company. If the neighbor brings an action for negligence against the light company, which of the following is the light company's strongest defense?

The defective wires were incorporated into the pool light despite the exercise of reasonable care by the light company. Discussion of correct answer:In order to establish that the light company was negligent, the neighbor needs to show that the company owed him a duty, that the duty was breached, that he suffered damages as a result, and that the company's negligence was a cause-in-fact and proximate cause of his injuries. In this case, if the light company showed that it exercised reasonable care in manufacturing the pool light, then one of the elements of the negligence action would be missing. As such, the neighbor would not prevail, making this answer choice the company's strongest defense.

A defendant sought to appeal her conviction for armed robbery. During the defendant's criminal trial, she had asked for, and was denied, a jury instruction on the lesser-included charge of larceny. At trial, she had claimed that she was unarmed. Which of the following would be most important to the defendant's appeal?

Whether the jury could reasonably have concluded that a weapon was not involved. Discussion of correct answer: A criminal defendant is not entitled as a matter of right to a jury instruction for a lesser-included offense. A defendant must request such an instruction and point to testimony or facts that indicate that the request is reasonable. The reasonableness of such an instruction does not hinge upon whether the crime was a felony or misdemeanor, nor does it matter what penalty was imposed on the defendant. Thus, here, the most critical factor in the defendant's appeal is whether the jury could reasonably have concluded that the defendant was unarmed.

A man sued a contractor for breach of contract in state court for damages that resulted from a roof on his home that the contractor improperly installed. In accordance with state law, the man expressly demanded a jury. The contractor then filed a notice of removal to federal court. The man contacted his attorney 20 days later, concerned that he may have lost his opportunity to have his action brought before a jury. Can the man still have his action brought before a jury?

Yes, because the man demanded a jury trial in accordance with state law. Discussion of correct answer:If a party has expressly demanded a jury trial in accordance with state law prior to the case being removed to federal court, the demand need not be renewed after removal. Consequently, the man will still be entitled to a jury trial.

A man with a passion for cooking decided to open a French restaurant. He obtained a bank loan and signed a mortgage deed on a restaurant that was for sale, borrowing $500,000 in his own name to buy the building. Unfortunately, the man was not as good a chef as he imagined, and the restaurant did not make much money. When the man stopped making mortgage payments on the restaurant, the bank ordered sale of the property through foreclosure. At the time of the foreclosure sale, the man owed the bank $450,000. The parties' mortgage agreement stated an upset price of $425,000, but the property was sold to the only offeror for $410,000. If the bank seeks a deficiency judgment against the man, how much money will the bank be able to obtain?

$25,000. Discussion of correct answer:When a mortgagee sells property through a foreclosure sale, the proceeds from the sale are not always sufficient to cover the entire amount owed to the mortgagee by the mortgagor. In this case, the man still owed the bank $450,000, but the property only sold for $410,000. Therefore, the bank can seek a deficiency judgment against the man to make up for the difference. Here, however, the parties have set an upset price, which means that the foreclosure sale price was set at a minimum of $425,000. Even though the property sold for less than the upset price, the man will only be liable to make up the difference between the upset price and the amount he still owed. Therefore, he will only be liable for $25,000 ($450,000 still owed on the mortgage minus the $425,000 upset price.) Thus, this is the correct answer choice.

A homeowner hired a contractor to dig a new well. They agreed that the job was to be completed within two weeks, for a price of $3,000. After digging for one week, the contractor hit solid rock that was impenetrable with his current drill bit. The occurrence of hitting solid rock was rare in this area, and had never happened to the contractor in the thirty previous wells he had dug in the neighborhood. He asked the homeowner for an additional $1,000 to complete the job, which was fair and reasonable in light of the circumstances. The homeowner agreed to the contractor's request. After the job was completed, the contractor sent a bill to the homeowner for $4,000, but the homeowner refused to pay. At modern law, how much is the homeowner required to pay?

$4,000, because the modification was fair and reasonable. Discussion of correct answer: The homeowner will be required to pay $4,000 because of the unforeseen difficulties that arose during the course of performance. Modernly, a promise is enforceable, despite the preexisting duty rule, where circumstances arise not reasonably anticipated by the parties at the time of contracting. Here, the contractor's prior experience in the area, coupled with the rarity of such an occurrence in this locale, make it clear that the modification was fair and reasonable under these circumstances, and therefore the homeowner will be required to pay the modified price.

During a deer-hunting season open to rifle hunters, a hunter saw a deer in the forest. He shot his rifle at the deer, hoping to hit and kill it. Instead, he hit and injured a hiker. The hunter had not realized that the hiker was there. Does the injured hiker have an actionable battery claim against the hunter?

Answer (A) is correct because the hunter did not intend to shoot the hiker, and therefore, the hunter cannot be liable for the intentional tort of battery. A battery claim requires proof that the defendant did an act with the intent and result of creating a harmful or offensive contact with the plaintiff. It is an intentional tort, and thus requires proof of intent. Intent can be shown in several ways. The most basic and familiar sort of intent is where the defendant had the purpose or desire of causing a certain result. For example, if the hunter was trying to shoot and kill the hiker, that obviously would be intentional. The intent required for an intentional tort is also present if the defendant had knowledge that a certain result was substantially certain to occur. For example, if the hunter fired his rifle into a crowd of people, knowing it was highly likely that someone in the crowd would be struck by the bullet, that would be considered intentional. Intent can also be transferred from one person to another, as well as from one intentional tort to another. For example, if the hunter was trying to shoot someone else but missed and hit the hiker, the hunter would be liable for battery to the hiker. In this question, there is no way to find that the hunter had the intent required for battery. He did not have a desire to shoot the hiker, nor did he know that he was substantially certain to hit anyone. He did not know the hiker was there. And the hunter also cannot be liable through transferred intent, because he did not have the intent required for any intentional tort to anyone. If he had been trying to shoot another person and hit the hiker, he could be liable. Similarly, if he had been trying to shoot someone's property (such as shooting at someone's car) and he missed, hitting the hiker, he could be liable for a battery to the hiker through transferred intent. However, the hunter was shooting at a deer, and wildlife are not property that belongs to anyone. The hunter will therefore not be liable because he did not have any of the forms of intent that can create liability for a battery.

A wealthy businesswoman owned several acres of land. As a gift to her much younger sister, the businesswoman divided her property in half and gave her sister the portion of the land that faced south (Lot A), while the businesswoman retained the northern portion (Lot B). Lot A was bordered by a small private dirt road. Lot B faced a large public street. The businesswoman granted her sister an easement over Lot B so that she could easily access the paved road. The easement was written into the sister's deed and duly recorded with the appropriate authorities. After many years, the sister grew up, married, and built her home on Lot A. Her driveway exited onto the dirt road. However, she and her children used the easement to reach the public streets and school bus stops on Paved Avenue. When the businesswoman died, she left Lot B to her sister for life, with the remainder to her own children. The sister, devastated at the loss of her older sister, died a year later. She left her property (Lot A) to her son. The businesswoman's children immediately moved into the house on Lot B and built a fence across the easement, making it clear that they did not want their cousin crossing their land. The sister's son filed suit against them demanding that the fence be removed and that he be allowed to cross Lot B using the easement. Will the sister's son prevail?

Yes, because he inherited the easement when he inherited Lot A from his mother. Discussion of correct answer: This is a particularly difficult question. The sister's son inherited Lot A from the sister. The deed to Lot A contained an express easement over Lot B. As such, the sister's son inherited the easement with the property. Many students mention that for one year, the mother had a life estate in the servient estate--that is, she had possessory interest in both estates. Therefore, is there no termination of the easement if the same person has a fee simple interest in one but only a life estate in the other? In the facts, the businesswoman gave her sister a deed and into that deed was written an easement across parcel B. That easement was to last of infinite duration. In order to extinguish the easement fully, the sister needed to have complete unity of ownership in both parcels A and B. However, she did not receive complete unity of ownership of both parcels. The sister received only a life estate interest in parcel B. As a result, the unity of ownership (in both parcels) lasted no longer than the lifetime of the sister, and so the easement was lost (through merger) only during this time. Therefore, when the sister died, the easement continued to exist and the sister's son can cross parcel B. As such, choice (A) was the best choice here. Generally speaking, merger is usually a means to extinguish an easement. But those situations involve the sale and transfer of a fee interest in the servient tenement itself. This question took a different approach and only granted a life estate interest in the servient tenement.

A realtor working for an international realty company believed that he had not received the same bonuses and work conditions as other realtors in other countries because of his U.S. nationality. He filed an employment discrimination claim in federal court. After the parties had made their initial disclosures and done some discovery, the realty company filed a motion for summary judgment. The realtor's response stated that his pleadings raised a sufficient question of material fact and that the court should not grant the motion. The court granted the motion. Did the court err?

No, because a response to summary judgment cannot rely on the pleadings to raise a material fact. Discussion of correct answer: A court considering a summary judgment motion should view the evidence in the light most favorable to the non-moving party, which would give the realtor the benefit of the doubt here. However, it is not enough for the non-moving party simply to rely on his pleadings in response to a summary judgment claim. The realtor's doing so meant that the court could rightly grant summary judgment.

A woman is on trial for murder. The prosecution calls a police detective to testify against her. On cross-examination, the woman's attorney asks the detective to confirm that the murder victim, shortly before he died, stated aloud that the woman was not the one who killed him, and the detective answers that he is unable to recall whether the victim made such a statement. The woman's attorney seeks to refresh the detective's memory by asking him to read a case report prepared by another police officer immediately following the woman's arrest. Is it permissible for the woman's attorney to request that the detective read the police report?

Yes, but only to refresh the detective's recollection. Discussion of correct answer: At trial, a witness may use a writing (or any other item) to refresh her memory for the purpose of testifying. However, the witness may not read from the writing while testifying, and the writing cannot be entered into evidence unless the adverse party seeks to inspect the writing and have the portions relating to the witness's testimony entered into the record.

Ten months after surgery in a hospital, a patient who had suffered complications from the surgery sued the surgeon and the hospital in federal court for medical malpractice, seeking $750,000 in damages. Timely personal service was made on the surgeon and the hospital. Three months later, during discovery, the patient learned that the hospital was owned by a national health-care company and moved to amend the complaint to substitute the company for the hospital. The company moved to dismiss, arguing that the forum state had enacted a one-year statute of limitations for medical malpractice actions and that the company had been served after the limitations period had expired. The company also noted that the state's highest court has interpreted the limitations statute as forbidding any relation back of amendments adding parties in medical malpractice actions. The patient argued that the Federal Rules of Civil Procedure control, and that they allow relation back under the circumstances of this case. Which law governs whether relation back will be permitted under these circumstances?

Federal law, because the Federal Rules of Civil Procedure govern over conflicting state rules that deny relation back. Discussion of correct answer: The Supreme Court has held that the Federal Rules of Civil Procedure govern over conflicting state rules unless it can be found that the Federal Rule at issue was promulgated in violation of the Rules Enabling Act. Hanna v. Plumer, 380 U.S. 460 (1965). The Act authorizes the Supreme Court to prescribe federal procedural rules so long as they do not "abridge, enlarge or modify any substantive right." 28 U.S.C. Sec. 2072(b). To date, no Federal Rule has been found to be in violation of the Act.

A car manufacturer produced a car that was sold nationwide. Problems with the car's brakes allegedly caused several accidents and injuries. Two individual buyers of the car each filed a class action, in different states, against the manufacturer, asserting the same products liability claims on behalf of all buyers nationwide. One class action was filed in federal court and the other was filed in state court. The parties in the federal action reached a court- approved settlement, and the court entered judgment dismissing the action with prejudice. The manufacturer's attorney has moved to dismiss the state court action on the basis of res judicata (claim preclusion). Should the state court look to federal or state law to decide the effect of the judgment?

Federal law, because the judgment was entered in federal court. Discussion of correct answer: Answer (A) is correct. Claim preclusion (also known as res judicata) prevents relitigation of a claim: (1) between the same parties and those who are in privity with them; (2) arising out of the same transaction or occurrence underlying the prior suit; and (3) that was determined on the merits by a court with proper subject-matter and personal jurisdiction. The claim-preclusive effect of a federal court's dismissal of a claim in federal court based on diversity jurisdiction is determined by federal law [Semtek International, Inc. v. Lockheed Martin Corp., 531 U.S. 497 (2001)]. Consequently, the state court will look to federal law in determining the effect of the judgment.

A brother and sister had feuded throughout their adult lives. Their father hoped that by leaving them their childhood home as tenants in common, they would end their feud. Unfortunately, it had the opposite effect. As soon as the father's estate was settled, the son moved into the property with the sister's knowledge. He lived on the second floor of the house, and rented out the first floor to two tenants. He paid all taxes on the property, changed the locks, and kept all rental payments. A few years later, the sister visited her hometown, hoping to stay in her old bedroom for the weekend, but the brother refused. She reluctantly stayed at a nearby hotel instead. A decade passed, and the sister died, leaving her interest in the house to her friend, who recorded his interest in the property immediately after the sister's estate was settled. The sister was survived by a son. The jurisdiction in which the house is located is a pure notice jurisdiction and has a seven year statute of limitations for adverse possession. The friend now seeks a judicial decision as to the ownership rights of the parties. As to the issue of ownership, how should the court rule?

For the brother, because he ousted the sister. Discussion of correct answer: Ouster requires more than using the property exclusively and paying the taxes. A physical act or an act in direct contravention of the co-tenant's rights must occur. Here, the brother rented the property and withheld from his co-tenant sister her portion of the rental income. He also refused her entry when she came to visit and stay in the house. Before the sister died, it is clear that, based on these facts, she was ousted from the property and the brother came to own the house by adverse possession. Therefore, neither the son, nor the friend can claim an interest.

Congress recently declared war on a foreign republic. In pursuit of this war, Congress enacted a law that instituted rent controls for the duration of the wartime period. The statute provided that the law would be administered by a rent control board. A group of landlords sued the administrator of the rent control board, contending that the rent control law was unconstitutional. Assume that standing and jurisdiction are both proper. How should the court rule?

In favor of the administrator of the rent control board, because Congress has the power to initiate wage, price, and rent control of the civilian economy during wartime pursuant to the War Powers Clause. Discussion of correct answer: The War Powers Clause [U.S. Const. art. I, Sec. 8], in conjunction with the Necessary and Proper Clause, gives Congress very broad authority to initiate whatever measures it deems necessary to provide for the national defense in peacetime as well as in wartime. Congress has the power to initiate wage, price, and rent control of the civilian economy during wartime [Yakus v. United States, 321 U.S. 414 (1944)]. As such, here, Congress's act was permissible.

A computer hacker logged into the social networking account of a travel writer. The hacker chatted with several of the travel writer's friends and told each of them, while pretending to be the travel writer, that he'd been mugged while in Malaysia and that he only had a few minutes left on an internet cafe account to find somebody who could wire him $500, which he would pay back when he returned to the states. One of the travel writer's friends was duped into sending money to an account owned by the computer hacker. Eventually, the hacker was caught and two different states sought to prosecute him for the crime: the state where the hacker was located and the state where the duped friend was located. The computer hacker was tried and convicted in the state where he lived but when the second state attempted to take its turn at conviction, the hacker argued the defense of double jeopardy. How will the court rule?

In favor of the second state, because it is a separate sovereignty. Discussion of correct answer: Double jeopardy is a constitutional defense that prevents the same authority from trying the same defendant for a crime for which he has already stood trial. The theory is to prevent harassment and undue expense. However, if a crime crosses state lines, two separate authorities can prosecute. This is known as the "separate sovereignties" doctrine and it applies when two states, or a state and the federal government, prosecute for the same crime. In this case, the defendant may be tried in both states because double jeopardy does not attach.

A 16-year-old student took his date for the high school prom out to dinner before the dance. It was the first time the student had ever been to a restaurant without his parents, and the first time he had ever been to such an expensive restaurant. The student had badly underestimated the amount that dinner would cost, especially because his date ordered one of the most expensive entrees. He only had the change in his pocket left to tip the waiter after paying for their meal. Their waiter was a particularly belligerent person who, when he saw that the student had left a few coins for his tip, grabbed the coins from the table and threw them in the student's face, shouting, "You miserable, pimple-faced bumpkin! How dare you insult me by leaving this pittance as my gratuity?" When the student attempted to whisper an explanation, the waiter continued loudly, as everyone in the crowded restaurant looked on, "I don't want to hear your protestations of poverty. I could tell from your grotesque manners and from the way you're dressed that you are an uncouth, unlettered ingrate." When the student attempted to leave, the waiter pushed him roughly to the floor, exclaiming, "Pick up your ridiculous money!" The student grabbed at the coins on the floor and fled, humiliated. Unknown to the student, his mother had booked a table in a secluded portion of the same restaurant so that she could videotape his first date. She observed the entire episode with the waiter, suffering severe emotional distress and as a consequence, subsequently brought suit against the waiter for intentional infliction of emotional distress. What result is likely?

Judgment for the waiter, because he was unaware of the mother's presence in the restaurant. Discussion of correct answer:A defendant may be liable for intentional infliction of emotional distress when he engages in extreme and outrageous conduct toward one person, intending to inflict emotional distress upon a family member of the victim whom the defendant knows is present. Whether viewed as a limited application of the "transferred intent" doctrine or of the principle that the defendant must be substantially certain that a family member witnessing his attack upon the victim will suffer emotional distress, the defendant must be aware of the family member's presence. Because the waiter was unaware of the mother's presence in the restaurant, he cannot have had the requisite intent as to her.

A purchaser filed a federal diversity action against a seller, alleging breach of contract. The seller answered the complaint and included as a separate defense an allegation that the purchaser had brought and lost a similar contract claim against a different seller three years earlier, and that this history represented a pattern of filing frivolous lawsuits. The purchaser believes that the earlier lawsuit was factually completely different from the current one and is therefore irrelevant. What is the purchaser's best response to the seller's answer?

Move to strike the separate defense as irrelevant. Discussion of correct answer: Answer (D) is correct. The court may strike from a pleading an insufficient defense or any redundant, immaterial, impertinent, or scandalous matter [Fed. R. Civ. P. 12(f)]. Because the purchaser believes the earlier lawsuit is irrelevant, the purchaser should bring a motion to strike it.

A woman in Mexico with diabetes took a medication that was made by a United States manufacturer at a factory in Mexico. After several years, she discovered that it had worsened her condition and that, as a result, she would need a kidney transplant. She filed suit in the federal district court in the United States where the manufacturer was headquartered. The manufacturer believed that it would be difficult to defend the suit in the court where it was filed because the doctors involved, information about manufacturing processes, and other information related to the suit were in Mexico. If the manufacturer files a motion for forum non conveniens, seeking transfer of the case to Mexico, should the court grant it?

No, because federal courts cannot transfer venue to a foreign country. Discussion of correct answer: The doctrine of forum non conveniens does apply when the most convenient court is a foreign one. It is also the case that deference to a plaintiff's choice of forum is lessened if the plaintiff is a foreign citizen. The problem here is that the court would have no power to transfer venue outside of the U.S. As a result, the court can only dismiss the case and leave the plaintiff to refile elsewhere.

A construction company built an apartment building that collapsed and killed 10 people. The company had only $1 million in assets. The estates of the 10 people killed told the company that they planned to bring 10 separate actions, asking for damages of over $4 million in total. May the company bring an interpleader action to resolve its liability among the plaintiffs?

No, because interpleader is inappropriate in this action. Discussion of correct answer: Interpleader is used where a plaintiff has some holding that would expose the plaintiff to multiple liability from adverse claims. The stakeholding party, or "stakeholder," can commence an action for interpleader to resolve liability where there are two or more adverse claimants. However, interpleader is only applicable where multiple claims demand the same thing or obligation--usually a piece of property, prize, or insurance policy. Here, while the company may only have $1 million in assets, its liability is legally unlimited and interpleader is inappropriate.

A plumber was on trial for arson and attempted rape. The plumber's attorney called a witness to the stand and asked, "In your opinion sir, is the plumber an honest, trustworthy citizen?" Before the witness could answer, the prosecution objected. Should the court allow the testimony?

No, because it is irrelevant. Discussion of correct answer:Given that the plumber is charged with a property crime (arson) and a crime of violence (attempted rape), his honesty is not at issue. Therefore, the testimony as to his character for trustworthiness is irrelevant. While the defendant may offer evidence of his good character inconsistent with the charged offenses, Federal Rule of Evidence 404 does not permit him to offer general evidence of his good character.

A moneylender met one of his debtors on the street. The moneylender demanded that the debtor pay what he owed and threatened to break his kneecaps if he didn't. They began to fight. A bystander, who knew neither of them, arrived just as the moneylender knocked the debtor to the ground and was about to kick him in the head. Seeing that the moneylender was getting the better of the debtor, the bystander pointed a gun at the moneylender and said, "Stop, or I'll shoot." If the moneylender sues the bystander for assault, will he prevail?

No, because it was apparent that the moneylender was about to inflict serious harm on the debtor. Discussion of correct answer: A defendant is entitled to defend another person from an attack by the plaintiff to the same extent that the third person would be lawfully entitled to defend himself from the plaintiff. A defendant otherwise acting in self-defense may only use the degree of force reasonably necessary to avoid the harm threatened by the plaintiff. For example, a defendant could not successfully assert self-defense if he used deadly force against a plaintiff whose conduct did not threaten death or serious bodily harm to that defendant. In this case, because the moneylender threatened serious bodily harm to the debtor, the bystander was justified in her threat to use deadly force.

A defendant was charged with manslaughter. At the preliminary hearing, the magistrate dismissed the charge on the ground that the evidence was insufficient. The prosecutor then brought the case before a grand jury. After hearing the evidence presented by the prosecutor, the grand jury refused to return an indictment. The prosecutor waited a few months until a new grand jury had been impaneled and brought the case before that grand jury, which returned an indictment charging the defendant with manslaughter. The defendant has moved to dismiss the indictment on double jeopardy grounds. Should the motion be granted?

No, because jeopardy had not attached before either grand jury was impaneled. Discussion of correct answer:Jeopardy does not attach at a preliminary hearing. Jeopardy attaches in a jury trial when the jury is sworn, and in a bench trial when the court begins to hear evidence.

A builder brought a federal diversity action against a homeowner for breach of contract, seeking $115,000 in damages. The homeowner answered and counterclaimed for $93,000 in damages for breach of warranty. Four weeks later, when the homeowner had not received any responsive pleading from the builder, the homeowner moved for entry of default and a default judgment on the counterclaim. After the clerk entered default, the homeowner asked the clerk to enter a default judgment. May the clerk enter a default judgment?

No, because only the court can enter a default judgment under these circumstances. Discussion of correct answer:A clerk may enter default judgment only when the amount sought is a sum certain and the defaulting party has failed to appear. Fed. R. Civ. P. 55(b)(1). Here, because the defaulting party--the builder--is the original plaintiff in the action and thus appeared when filing the complaint, the clerk is precluded from entering a default judgment.

A construction company is building a two-story plaza in town. The construction company has built a series of ramps that the carpenters and masons use to move materials from one level to another. These ramps remain set up even when the workers are not on-site. The teenage son of the site manager advises his father that many of his friends are using the ramps to jump into the air on their bicycles and skateboards. The next day, the site manager posts several prominent signs at the site which stated, "Warning--Extreme Danger! No Trespassing. Keep Off Ramps. Authorized Persons Only. No Skateboards or Bicycles." However, the site manager continued to find evidence that children had been riding bikes and skateboards on the property even after posting the signs. One week later, a 12-year-old boy who ignored the signs rode his skateboard over one of the ramps, fell, and suffered serious injuries. Is the construction company liable for the 12-year-old's injuries?

No, because the 12-year-old boy was warned of the danger. Discussion of correct answer:Because the 12-year-old boy saw the signs and was aware of the risk, he will be treated as an adult trespasser, and so the attractive nuisance doctrine will not apply. As such, the boy assumed the risk of injury when he ignored the posted warning signs.

A company made screws in State A. The company sold the screws to the corporation, incorporated in State B. The corporation found that the screws were defective. The defective screws have caused $65,000 in damages. The corporation also discovered that the company did not send the proper number of screws, in violation of the contract between the companies. The corporation paid $100,000 for 1,000 screws, but only received 800 screws, causing $20,000 in damages. The corporation sued the company in federal district court for damages. The company filed a motion to dismiss the case for lack of subject matter jurisdiction. Will the federal district court grant the company's motion to dismiss?

No, because the amount in controversy meets the requirement for federal court jurisdiction. Discussion of correct answer:Where a plaintiff sues a defendant for multiple claims based on the same case or controversy, the separate damages of each claim can be aggregated together. Here, there are two claims regarding the screws. The first is that the screws were defective and caused damage. The second is that the plaintiff did not receive the proper amount of screws in the first place. If the plaintiff only sued on one claim or the other, the amount in controversy--$75,000--would not be met. However, because there are two claims against the defendant, the amount of each claim can be added together. Once the two claims are aggregated, the total amount claimed--$85,000--would meet the amount-in-controversy threshold of $75,000.

A caterer contracts with a wedding party to provide food for 200 guests at a dinner on a Saturday, right after the caterer services a lunch event for 50 people that same morning. The caterer hired only one chef. The chef does complete the lunch for the 50 people, but then the chef becomes extremely upset and leaves the caterer shortly before the dinner service. As a result, the caterer fails to provide the dinner service as expected, only giving appetizers. Is the caterer excused for impossibility?

No, because the caterer should have had a plan in case the chef was unable to complete the work. Discussion of correct answer:Subjective impossibility occurs when performance under the contract is not possible because of some failure by the performing party. The performance obligation, however, is not excused, and will be considered as a breach of the contract. In this case, the caterer failed to hire more than one person to help with a lunch of 50 people and a dinner of 200 people. The caterer could have avoided this problem by hiring another person or having some kind of back-up plan. Because the caterer did not, it is the caterer's own fault that it was impossible to finish the dinner service for the wedding. Therefore, although it was impossible, it was not excusable.

A woman lives in a mixed-use neighborhood. Her street and the surrounding blocks have a variety of commercial businesses and restaurants interspersed among several apartment buildings and a few houses. Many buildings have businesses on the ground floor and apartments on the upper floors. The woman's apartment is on the second floor of one of these buildings. The first floor of her building used to contain a bakery. However, the bakery went out of business and was replaced by a dry cleaner. The woman happens to be extremely allergic to dry-cleaning fluid. In fact, she is unable to have a conversation with anyone wearing dry-cleaned clothing unless they stand at least six feet away from her. Even though the dry cleaner has an off-site cleaning facility and does not do any actual cleaning on the premises, the fumes from the dry-cleaned clothes cause the woman to have a severe allergic reaction any time she opens her windows. The woman complained to the dry cleaner and to city officials, but the operation of a dry cleaner is consistent with all zoning requirements. The woman brought a private nuisance action against the dry cleaner. Is the woman likely to prevail?

No, because the dry-cleaning fumes would not offend persons of normal sensibilities in the community. Discussion of correct answer: Private nuisance is conduct that substantially and unreasonably interferes with another person's use and enjoyment of his land. To bring a private nuisance action, the plaintiff need only have property rights or privileges in the land, so a renter's possessory interest would suffice. For the interference to be categorized as substantial, it must be offensive to a person of normal sensibilities in the community. It is not sufficient for the interference to be offensive to an abnormally sensitive person. The woman's abnormal sensitivity to dry-cleaning fluid and dry-cleaned clothes does not allow her to recover from the dry cleaner.

A homeowner had a chestnut tree on his property. One night, his neighbor, who had been drinking, drove his car into the tree. A police officer investigating the incident told the homeowner that he needed to pound stakes into the ground to support the tree or the tree would topple over. The homeowner pounded in the stakes, but the physical strain of doing that gave him a terrible back injury. Is the neighbor liable for the homeowner's back injury?

No, because the homeowner's injury was not a foreseeable result of the neighbor's negligence. Discussion of correct answer:A tortfeasor owes a duty of reasonable care only to foreseeable plaintiffs; that is, those individuals who are within the risk of harm created by the tortfeasor's unreasonable conduct. The majority view holds that a defendant only owes a duty of care to those who are within the zone of danger. The "zone of danger" means that, under the circumstances, a reasonable defendant would have foreseen a risk of harm to that person. Here, the drunk neighbor might have foreseen other drivers and pedestrians, plus rescuers of injured parties, but he would not have foreseen back injuries to persons dealing with property damage following his accident. The chain of events would be too attenuated.

In a civil trial for professional malpractice, the plaintiff sought to show that the defendant, an engineer, had designed the plaintiff's flour mill with inadequate power. The plaintiff called an expert witness who based his testimony solely on his own professional experience but also asserted, when asked, that the book Smith on Milling Systems was a reliable treatise in the field and consistent with his views. On cross-examination, the defendant asked the witness whether he and Smith were ever wrong. The witness answered, "Nobody's perfect." The defendant asked no further questions. The defendant later called his own expert witness and asked, "Do you accept the Smith book as reliable?" The witness said, "It once was, but it is now badly out of date." The plaintiff requested that the jury be allowed to examine the book and judge for itself the book's reliability. Should the court allow the jury to examine the book?

No, because the jury may consider only passages read to it by counsel or by a witness. Discussion of correct answer:Federal Rule of Evidence 803(18), the learned treatise hearsay exception, provides that if the court finds a publication to be a reliable authority, then "statements" from it may be read into evidence, but the publication may not be received as an exhibit. Thus, the jury is not allowed to bring learned treatises into the jury room. There is a concern that if juries were allowed unrestricted access to the whole publication, they might rely on parts of the publication that are not germane to the case. Moreover, the intent of the rule is that juries need to be guided through the pertinent parts of the publication by the testifying experts.

As a tribute to his great body of legal work, a professor of property law established a library on his 10-acre estate. After several years of operating the library at a loss, the professor decided to sell the property. The professor conveyed the property by a duly executed deed "to librarian, but if the premises cease to be used for educational purposes, then to my alma mater." One night, the librarian served his friend a beer on the premises. The professor's alma mater filed suit claiming ownership. Assume that all interests satisfy the Rule Against Perpetuities. Should the alma mater prevail?

No, because the librarian holds a fee simple subject to an executory interest. Discussion of correct answer:A fee simple subject to an executory interest is an estate that is automatically divested to a third person upon the occurrence of a named event. In this case, the professor deeded the property to the librarian, with the condition that the alma mater receives the property if the premises cease to be used for educational purposes. But in this case, the fact that the librarian served his friend a beer does not indicate that the premises have ceased to be used for educational purposes. The librarian will prevail because he holds the estate in fee simple subject to an executory interest and the named event that would cause transfer of the property has not occurred.

An opera singer sued a theater company in federal court for breach of contract. Feeling that the opera singer's lawsuit was meritless, the theater company made a motion for summary judgment immediately after the parties' initial discovery conference. In opposing the motion, the opera singer's lawyer argued that the motion was not timely filed. The court in which the lawsuit was filed follows the Federal Rules of Civil Procedure. Is the motion untimely?

No, because the motion was made prior to the close of discovery. Discussion of correct answer:A party may move for summary judgment at any time until 30 days after the close of all discovery. Here, the parties have just completed their first discovery conference, and therefore the motion for summary judgment is timely filed.

A local pizza shop placed an old inoperable school bus on its parking lot to use as an outdoor eating area during the summer months. The pizza shop is located on a highway not far from a residential area. The pizza shop began to paint the bus a bright rainbow of colors and spray painted funny sayings on all sides of the school bus. The pizza shop began to renovate the interior, but it would take a few weeks. The seats were removed, but the floor was torn up with holes and tools scattered about. One Sunday morning when the pizza shop was not open, a 10-year-old child entered the bus through an open window. The bus was posted with a sign saying "Warning--Active Construction. DO NOT ENTER. DANGER." However, the child ignored the sign and went in anyway. As he was climbing back out of the bus through the window, the child fell to the ground breaking his arm and wrist. The trial court entered summary judgment in favor of the pizza shop owner, ruling that he owed no duty to the child. The child has filed an appeal. Should the appellate court affirm the trial court's ruling?

No, because the pizza shop owner owed a duty of reasonable care because it was foreseeable that the bus would attract a child trespasser. Discussion of correct answer:Generally, a landowner owes no duty of care to an unknown trespasser. Nor does a landowner have any duty to inspect to protect an unknown trespasser from injury. However, a landowner may owe a child trespasser a heightened standard of care. Under the attractive nuisance doctrine, a landowner has a heightened standard of care to protect children from dangerous artificial conditions conducted on the land. When the doctrine applies, the landowner has a duty to exercise reasonable care. Here, the facts indicate that the brightly colored bus in its current condition could pose an unreasonable risk of danger to children. The facts indicate the pizza shop is near a residential area, thus making it foreseeable that a child would be likely to trespass. Because of the child's age, he is unlikely to appreciate the risk and the risk of the danger of the bus outweighs its social utility. The pizza shop owner would owe the child a duty and would be subject to a heightened standard of care.

In a case involving an allegation of a defective braking system, the plaintiff presents several expert witnesses who establish that her injury could not have occurred unless the brakes in her car had a particular manufacturing defect. The defense presents experts who state that there is no evidence of such an error, but allege that the accident in question occurred because of plaintiff's inattention. At the close of the defense's case, the plaintiff makes a summary judgment motion. Was the plaintiff's action proper?

No, because the plaintiff should have brought a motion for a judgment as a matter of law. Discussion of correct answer: The proper motion to make in this instance would be a motion for a judgment as a matter of law (JMOL), which can be made after all of the non-moving party's evidence has been presented but before a case is submitted to the jury. While a JMOL has the same standard as a summary judgment motion, the difference is that a summary judgment motion is based on evidence that parties may introduce at trial. A JMOL is based on evidence that the parties actually have introduced at trial.

A wildlife biologist lived on a large piece of property where he kept a number of exotic animals needing rehabilitation. The property was not open to the public, and notices were posted throughout the property that dangerous, wild animals were present. One night, a drunk college student thought it would be fun to climb the fence enclosing a zebra on the biologist's property. The student proceeded to try to jump on the zebra's back and take it for a ride, which resulted in the student being bucked off and trampled. The student sued the biologist for his extensive injuries. Is the biologist strictly liable for the student's injuries?

No, because the student was an unknown trespasser. Discussion of correct answer:A defendant may be strictly liable for possession of animals. A defendant can be held strictly liable for personal injuries inflicted by his animal if it has "known dangerous propensities." Wild animals generally have known dangerous propensities for this purpose. A wild animal is one not customarily devoted to the service of humankind at the time and in the place where it is kept. However, if the plaintiff is an unknown trespasser, most jurisdictions do not impose any liability for injuries inflicted by a defendant's animals while the plaintiff is on the defendant's land, even as to animals with known dangerous propensities. In this case, while the zebra is a wild animal with known dangerous propensities, the drunk college student was an unknown trespasser. Therefore, the biologist will not be strictly liable, and this answer choice is correct.

A plaintiff and his cousin were riding in a car driven by the plaintiff when they were suddenly rear-ended by the defendant. The plaintiff was a citizen of State A, and the defendant was a citizen of State B. The plaintiff commenced a lawsuit in the appropriate U.S. District Court of State B (where the accident occurred), claiming $180,000 in injuries and damages as a result of the occurrence. The cousin (a citizen of State B) sought to intervene. However, his injuries were far less severe than those suffered by the plaintiff. The cousin was told by his attorney that he would probably not be able to recover more than $8,000. Will the U.S. District Court grant the cousin's motion to intervene?

No, because there is no subject matter jurisdiction over his claim against the defendant. Discussion of correct answer:Since the cousin's interests will not be impaired as a consequence of the present lawsuit (there would be no res judicata or collateral estoppel effect on him if the defendant is successful), intervention as of right would not be applicable. Thus, the cousin's intervention would only be permissive in nature, and ordinary subject matter requirements must be satisfied. Since the cousin and the defendant are citizens of the same state (State B), diversity would not be satisfied.

A woman robbed a convenience store, after holding the cashier at gunpoint. She got away with some cash and cigarettes. After driving around for a while, she stopped at her ex-boyfriend's apartment. He was home and let her come inside. While the ex-boyfriend heated up some leftover pizza in the microwave for the woman, she told him about the robbery. She exhibited an abundance of bravado and even bragged that if the police caught her, she had a handcuff key hidden on a chain around her neck and she would surely find a way to escape. She showed the key to the ex-boyfriend who recognized it as the key to a lockbox that the woman owned. The woman finished her pizza and left. A couple of hours later, the police visited the ex-boyfriend asking if he had seen the woman that night. He said that he had and told the police the details of her visit, omitting the fact that she claimed to have a handcuff key. When the police caught the woman later that evening, they handcuffed her and put her in the backseat of the police cruiser. She used her key, which actually was a handcuff key, and was able to free her hands. She hit one of the police officers over the head but was immediately subdued, and the officer suffered only a minor injury. Assume that all of her crimes are felonies. Is the ex-boyfriend an accessory to any of the woman's crimes that evening?

No, he is not an accessory. Discussion of correct answer: The ex-boyfriend was neither an accessory after the fact to the robbery, nor an accessory before the fact to the battery of the police officer. He was not an accessory before the fact to the battery because he did not aid, abet, or otherwise encourage the ex-girlfriend to commit the crime. Nor was he an accessory after the fact to the robbery. A felony had been committed, and the ex-boyfriend knew of it. However, he did not meet the third requirement of having personally given aid to hinder the felon's apprehension, conviction, or punishment. He was very forthcoming with the police and did not purposefully hide the fact that the woman had a handcuff key because he did not believe that she had one, rather believing that the key was to a lockbox she owned. The ex-boyfriend, therefore, has the defense of mistake of fact. Therefore, the ex-boyfriend is not an accessory either before or after the fact, and this is the best answer.

For several weeks, a boy and his neighbor, who were both 11 years old, had been working together to build a go-cart. When the go-cart was finally finished, the boys couldn't wait to try it out. They designed the go-cart so that both of them could ride on it at the same time. The boys also figured out how to make a crude steering mechanism by attaching a length of rope to the front axle. They wanted to try the go-cart on a street with a slope. The boy and his neighbor headed over to a cul-de-sac, the only street in their neighborhood with a substantial incline. When they reached the top of the incline, the boys jumped on the go-cart and took off, very quickly picking up speed. At the bottom of the incline, a busy street crossed the cul-de-sac. As the boys neared the bottom of the slope, they noticed that a car driven by a man was about to enter the intersection between the cul-de-sac and the busy street. The boy, who was riding behind his neighbor, saw the man's car coming. Because he believed that the neighbor would not be able to stop or turn the go-cart in time to avoid a collision, he yelled, "Jump off!" When the neighbor didn't move, the boy pushed the neighbor off the go-cart. The neighbor suffered a broken wrist and severe scrapes on his face and right arm. The go-cart, with the boy still on it, shot into the intersection. The boy was struck by the man's car and suffered multiple fractures and internal injuries. If the neighbor's parents, on behalf of the neighbor, sue the boy's parents to recover damages for the neighbor's injuries, will they win?

No, unless the boy acted unreasonably when he pushed the neighbor off the go-cart. Discussion of correct answer:An individual, when engaged in any activity, has a legal duty to act as a reasonably prudent person under the circumstances. Note that a defendant has a duty to act as a reasonably prudent person would in the particular circumstances under which the defendant acted. The existence of an emergency does not relieve the individual from the responsibility of acting reasonably. Minors are expected to act in a manner that is similar to how a reasonable child of the same age, education, intelligence, and experience would have acted. Given the facts of the question, this is the best answer, and the neighbor's parents will not recover damages from the boy's parents unless the boy acted unreasonably when he pushed the neighbor off the go-cart.

A landowner conveyed a large estate "to my daughter, her heirs and assigns, on condition that she produce an heir with her husband, but if she dies without such an heir, then to my personal assistant." Five years after the landowner conveyed the estate to her daughter, the personal assistant and the landowner got into a serious dispute, which resulted in the personal assistant's decision to set up her own rival business. At the same time, the landowner's daughter and her husband divorced. Three years later, the husband died of a stroke brought on by undiagnosed high blood pressure. The daughter remarried to a new business protege of her father. The estate was a heavily forested property with a number of stands of maple trees. The daughter and her new husband walked the property and marked stands to be harvested for wood. The daughter then exploited the timber on the estate for a large profit. Which of the following best describes the daughter's act of exploiting the timber for profit?

Permissible, because the daughter holds in fee simple. Discussion of correct answer: As the holder of a fee simple estate subject to an executory interest in favor of the personal assistant, the daughter could permissibly exploit the timber on the estate without regard for the law of waste. When her husband died, it became impossible for the daughter to produce an heir with him. However, the language of the landowner's conveyance provided that the daughter's present estate would terminate "if she dies without such an heir." Thus, the phrasing indicates that the daughter's estate will terminate upon her death, not when the condition became impossible to fulfill. Therefore, the daughter's present estate in the property continued after her husband's death. As the holder of a fee interest, the daughter can permissibly use the property in any manner she chooses. As such, this answer is correct.

With permission of his single mother, a 10-year-old boy drove his bicycle along the side of a busy highway to his grandmother's house. It started to rain, and a passing driver negligently skidded into the boy, killing him. If the applicable jurisdiction has adopted the rule of contributory negligence and the mother brings a suit against the driver for wrongful death, what is the likely result?

She can collect only if neither she nor the boy were negligent. Discussion of correct answer:In jurisdictions that apply contributory negligence, either the decedent's negligence or the plaintiff's negligence could prevent a recovery in an action for wrongful death. In this question, negligence by either the mother or the boy would bar all recovery.

A consumer from State A filed a $100,000 products liability action in federal court against a manufacturer incorporated and with its principal place of business in State B. The consumer claimed that a flaw in the manufacturer's product had resulted in severe injuries to the consumer. In its answer, the manufacturer asserted a third-party complaint against the product designer, also incorporated and with its principal place of business in State B. Believing that the consumer had sued the wrong defendant, the manufacturer claimed both that the designer was solely responsible for the flaw that had led to the consumer's injuries and that the manufacturer was not at fault. The designer is aware that the manufacturer did not follow all of the designer's specifications when making the product. Which of the following arguments is most likely to achieve the designer's goal of dismissal of the third-party complaint?

The correct answer is: The manufacturer's third-party complaint failed to state a proper third-party claim. Discussion of correct answer: Under Federal Rule 14(a)(1), a defendant may serve a third-party claim only on a nonparty "who is or may be liable to it for all or part of the claim against it." This means that the basis of the claim must be derivative liability (e.g., indemnification or contribution). In order to satisfy the Rule, the manufacturer cannot simply allege that the consumer sued the wrong defendant.

A businessman owns a strip mall located just south of the site chosen for a new football stadium. If the deal goes through, business at the mall will increase dramatically, as traffic to and from the stadium will pass the entrance. An entrepreneur would like to purchase the mall, but hesitates because it is unclear whether the state will issue approval of the plan and appropriate funds to assist the franchise owner. The entrepreneur enters into an option-to-purchase agreement with the businessman in which the entrepreneur pays a $25,000 option fee. Shortly thereafter, the entrepreneur enters into an option-to-purchase agreement with a venture capitalist, who pays a $30,000 option fee. One week later, the state approves the plan for the stadium, and the venture capitalist exercises his option to purchase the mall from the entrepreneur. The entrepreneur then exercises his option to purchase the mall from the businessman and acquires title. The value of the mall is now 20% greater than it was on the date the entrepreneur entered into the option-to-purchase agreement with the businessman. One week later, the deal between the state and the football franchise owner falls through, and the owner announces that he is moving the football franchise to a neighboring state where the legislature is friendlier to sports franchises. The day following this announcement, the entrepreneur tenders the deed to the mall to the venture capitalist, who refuses to perform on grounds that the entrepreneur did not hold marketable title when the venture capitalist entered the option to purchase. The entrepreneur demands that the venture capitalist perform his obligation under the contract, but the venture capitalist refuses. If the entrepreneur files suit for breach of the land-sale contract and the venture capitalist counter-sues, which remedy is the court most likely to grant?

Specific performance to the entrepreneur. Discussion of correct answer: When the entrepreneur entered the option-to-purchase agreement with the venture capitalist, the entrepreneur did not hold marketable title to the mall. However, the entrepreneur was not required to have marketable title until the time of the closing. The venture capitalist has exercised his option to purchase the mall, but the closing has not yet taken place. When the entrepreneur exercised his option to purchase the mall and acquired title from the businessman, his title to the mall became marketable, and the venture capitalist's refusal to perform the contract on marketability grounds was a breach of the land-sale contract. Because the non-breaching party wishes to enforce the contract, the court will most likely grant the entrepreneur specific performance for the buyer's breach of contract.

A store owner operates a retail store specializing in ethnic crafts and other tourist items. The store owner executed a written contract in March with a supplier whereby the supplier would deliver 1,000 hand-carved fertility statues, a popular item with the seasonal tourist crowd. Delivery was set for May 1, and the contract price was $1,000 for the lot. In early April, the supplier called the store owner to let him know that the wood carvers had unexpectedly gone on strike. The supplier wasn't sure that he would be able to deliver all of the 1,000 pieces on May 1 as scheduled. Because there was still time for the store owner to find an alternative distributor, the store owner asked the supplier during that same phone call for assurances that the supplier could deliver all of the 1,000 statues on schedule. If not, the store owner wanted to cancel the contract. The supplier said that he'd have to check. The next day, the supplier called the store owner and told him that he would do his best to have the 1,000 statues on schedule, one way or another. Still insecure after the supplier's uncertain spoken assurance, the store owner called the supplier on April 12 and told him that he would be going elsewhere for his statues. He then ordered 1,000 statues from a different distributor. On April 15, the store owner received a letter from the supplier asking for assurances that the store owner would accept delivery of the statues on May 1. The store owner called the supplier and reiterated that he was not going to be getting his statues from the supplier. On April 20, the supplier filed suit for damages for breach of contract. The store owner retracted his repudiation on April 25. Who is likely to prevail in the lawsuit?

Supplier, because the store owner breach the contract.

A teenager was driving while three of the teenager's friends were passengers in the car. The friends were distracting the teenager, and while the teenager was driving on the freeway, his car skidded out of control and crossed the center median. The car crashed head-on into an oncoming vehicle being driven by an elderly man, leaving the elderly man with two broken legs. The accident occurred in the Southern District of State A. The elderly man was a resident of State B and sought to bring suit against the four teenagers for damages arising from the automobile accident. Two of the teenage defendants resided in the Eastern District of State A, one resided in the Northern District of State A, and one resided in the Western District of State A. The elderly man wanted to file his claim in federal court based on diversity of the parties. The elderly man may file his action in which judicial district(s)?

The Northern, Southern, Eastern, or Western Districts of State A. Discussion of correct answer:For civil diversity actions, venue is available in the judicial district in which any defendant resides, provided that all defendants are located in the same state. Here, given that the defendants are all State A residents and respectively reside in the Northern, Eastern, and Western Districts of State A, venue is proper in any of these districts. Venue is also appropriate in the federal district in which a substantial part of the events or omissions giving rise to the claim occurred or a substantial part of the property that is the subject of the action is located. In this instance, given that the accident in question occurred in the Southern District of State A, venue is also appropriate in this district. Therefore, the State B resident may bring the action against all of the defendants in the Northern, Southern, Eastern, or Western Districts of State A.

A businessman inherited a piece of property in State A. He decided to build a ski chalet for him to stay in once or twice each winter. He hired a contractor, a citizen of State A, to build the ski chalet. The businessman was a resident of State B and all of his discussions with the contractor took place in State B. Halfway through the project, the businessman decided that he needed to spend money in other areas and abandoned the project without compensating the contractor. The contractor filed a breach of contract suit in the only federal district court in State A. Which of the following statements is most accurate?

The State A court has jurisdiction over the businessman, because he has sufficient minimum contacts with the state. Discussion of correct answer:The businessman owns and uses property in State A, and it is from that property that the cause of action arose. The businessman could reasonably anticipate being haled into court in State A. Thus, personal jurisdiction over him is proper. Incorrect. The court has quasi in rem jurisdiction over the land on which the businessman's ski chalet is being constructed, because the land is located within the state. The following elements are required for quasi in rem jurisdiction: (1) there is real or personal property of value located within the state or territorial limits of the federal court; (2) the defendant owns the property; (3) the property is being attached or seized; and (4) the defendant has been provided proper notice of the proceedings. In addition, minimum contacts are required. Here, the land is located within the state, but it is not being seized.

At the end of a long and enjoyable day at the amusement park, a boyfriend and girlfriend decided to end the day by walking through the fun house. However, the two discovered that they only had enough money left to purchase one admission ticket. The couple came up with a plan for the girlfriend to buy a ticket, go into the fun house, and let the boyfriend in through the emergency exit. The girlfriend entered the fun house after purchasing her admission ticket, found the emergency exit, opened it, and let her boyfriend in. Nearby, a maintenance worker finished repairing an animated goblin character that leaped out at passersby. He activated it just as the girlfriend and boyfriend approached. The goblin malfunctioned and leaped out more than normal, hitting the boyfriend and causing a bruise. The boyfriend sued the owners of the fun house for the injury. Which of the following, if true, would be most helpful to the boyfriend?

The boyfriend's presence had been discovered by the maintenance worker before he was injured by the goblin. Discussion of correct answer:Even though the maintenance worker may not have known that the boyfriend's presence was unauthorized, the boyfriend would at least have been either a discovered trespasser or licensee. In either status, the boyfriend would be owed a duty to be warned of known artificial conditions that are unreasonably dangerous and would be owed a duty of reasonable care to be protected from injuries arising from activities on the land. Activating the goblin could be considered an artificial condition or activity. The language in this choice describes the boyfriend as a discovered trespasser and yields the strongest set of facts for his recovery.

As a holiday neared, a businessperson realized that her candy store was selling more chocolate bunnies than she anticipated. She sent an email message to a chocolate production company's email order center, which requested immediate shipment of 500 chocolate bunnies at the catalog price of $1.00 per bunny. At the time the owner of the company received the message, he had already sold nearly all of his chocolate bunny inventory. However, he still had 500 chocolate ducks in stock. He promptly ships the 500 chocolate ducks to the businessperson with a note stating that he has sold all of his chocolate bunnies, but that he is sending 500 chocolate ducks at a catalog price of $1.00 per duck in the hope that these will meet the businessperson's needs. At the point that the businessperson timely receives that shipment of chocolate ducks and the accompanying note, which statement best describes the rights and responsibilities of the businessperson and the owner of the chocolate production company?

The businessperson may accept the shipment of chocolate ducks and pay the owner the catalog price or she may reject the shipment, preventing the formation of a contract. Discussion of correct answer:Offers to buy goods for prompt shipment "shall be construed as inviting acceptance either by prompt promise to ship or by the prompt or current shipment of conforming or nonconforming goods." (UCC Section 2-206). Thus, even a shipment of nonconforming goods operates as an acceptance. (It is also a breach of contract.) However, the shipment of nonconforming goods will not operate as an acceptance if the seller "seasonably notifies the buyer that the shipment is offered only as an accommodation to the buyer." In such a case, the shipment of nonconforming goods is considered a counteroffer, which the buyer may either accept or reject. If the buyer rejects the nonconforming shipment, no contract is formed, and the buyer has no rights against the seller. In this question, the owner sent the nonconforming goods in response to the businessperson's offer, accompanied by notice indicating that the nonconforming goods were offered as an accommodation. The businessperson may accept the shipment of chocolate ducks and pay the owner the catalog price or she may reject the shipment, thereby preventing the formation of a contract.

A defendant is on trial for robbing a particular bank with his best friend. The friend has left the country and cannot be found. The prosecutor has called the friend's mother to testify to a conversation she had with her son the day before he left the country. She implored him not to go, but he said, "Mom, I have to go. I was involved in a robbery at [the bank], and I don't want them to catch me." The prosecutor has other evidence indicating that the defendant and the friend were together on the morning of the robbery. Is the statement by the friend to his mother admissible?

The correct answer is:Yes, as a statement against penal interest that is not testimonial under the confrontation clause. Discussion of correct answer:Statements against interest are not excluded by the rule against hearsay if the declarant is unavailable as a witness. A statement against interest is one that [Fed. R. Evid. 804(b)(3)]: (1) a reasonable person in the declarant's position would have made only if the person believed it to be true because, when made, it was so contrary to the declarant's proprietary or pecuniary interest or had so great a tendency to invalidate the declarant's claim against someone else or to expose the declarant to civil or criminal liability; and (2) is supported by corroborating circumstances that clearly indicate its trustworthiness, if it is offered in a criminal case as one that tends to expose the declarant to criminal liability. However, under the Confrontation Clause of the Sixth Amendment, testimonial hearsay is inadmissible against a criminal defendant who has not had an opportunity to cross-examine the declarant. In determining whether the evidence is testimonial, the Supreme Court has emphasized that "the question is whether, in light of all the circumstances, viewed objectively, the 'primary purpose' of the conversation was to 'creat[e] an out-of-court substitute for trial testimony'" [Ohio v. Clark, 576 U.S. 237 (2015)]. Here, the friend is not a party to the case, and he is unavailable to testify as a witness because he cannot be found. The admission to committing the bank robbery is something that is so contrary to his own interest that he would not have made it if he didn't believe it to be true. The friend's statement against interest would subject the defendant to criminal liability, but the facts also indicate that the prosecutor has other evidence to corroborate the friend's statement. Further, the friend's statements to his mother do not run afoul of the Confrontation Clause because the primary purpose of the conversation was not to gather information to substitute testimony. Rather, this conversation was one between a concerned mother and her son. As such, the statement by the friend to his mother is admissible.

A man bought a cold medicine from a store and had an allergic reaction to it which required him to be hospitalized for ten days. The man, a citizen of State X, brought an action in the United States District Court in State X against the store and the corporation that manufactured the cold medicine. The store was located in State Y and the corporation was a State Z corporation. The plaintiff requested damages in the sum of $80,000. Process was served personally on the owner of the store in State Y and on the president of the corporation at her office in State Z. Thereafter, the corporation filed a cross-claim against the store for $90,000 alleged to be due for merchandise previously sold by the corporation to the store. How should the court rule on a motion to strike the corporation's cross-claim against the store?

The court should grant the motion, because the cross-claim is unrelated to the plaintiff's action. Discussion of correct answer:Because the corporation's cross-claim against the store is unrelated to the plaintiffs' action, it is not permissible under Fed. R. Civ. P. 13(g). Cross-claims may be permitted only if it is related to the same transaction as the plaintiff's lawsuit.

A worker was injured when a machine he was using on the job malfunctioned. The worker brought a federal diversity action against both the machine's manufacturer and the company responsible for the machine's maintenance. At trial, the worker submitted a proposed jury instruction on negligence. The court did not accept the proposed instruction and instead gave a negligence instruction that the worker's attorney believed was less favorable and legally incorrect. The attorney did not object to the negligence instruction before it was given. The jury returned a verdict for the defendants. The worker has moved for a new trial on the ground that the court's negligence instruction was improper. What argument has the best chance of persuading the court to grant the motion?

The court's negligence instruction was plain error that affected the worker's substantial rights. Discussion of correct answer: Answer (C) is correct. A motion for a new trial under Fed. R. Civ. P. 59 is generally appropriate in the following instances: (1) in order to avoid an inevitable appeal and reversal if the trial judge has committed reversible error; (2) when a jury verdict is so excessive as to demonstrate that the jury has misunderstood its duty or has acted with extreme prejudice; (3) if evidence of jury misconduct exists, and Fed. R. Evid. 606(b) limits judicial inquiry to external influences on the deliberation process; or (4) when the verdict is against the clear weight of the evidence. If the negligence instruction was plain error, it could result in an inevitable appeal and reversal. Consequently, this argument has the best change of persuading the court to grant the motion. The other three generally appropriate instances for a new trial are not involved in this particular claim.

A company makes and sells air-conditioning units. It had been several months since the company had heard from one of its regular regional distributors. The company sent the distributor a letter indicating that the company estimated that the distributor had probably sold all of the air conditioners it had in stock and that, based on this estimation, the company was sending a shipment of 250 units. The company said each unit would cost $225 and that the company was planning to ship them to the distributor in lots of 50, so that the distributor would owe $11,250 per shipment. The company indicated that it hoped this would keep the distributor supplied with enough units to last it for a while. The distributor did not respond to the company's offer, but did receive the first shipment of air conditioners. The distributor sold the first 50 units to other dealers in the region and kept none of them in stock at its company. The company heard about the distributor's sale of the units to dealers in the area and prepared the next shipment of 50 air conditioners. Under these circumstances, what is the distributor's obligation to the company?

The distributor bound itself to a contract to accept all five shipments at $11,250 each because it sold the first shipment. Discussion of correct answer: The company and the distributor had previous dealings with each other, which gives a basis for their current dealings. If both parties have previously performed on a contract for the sale of goods, and if both parties are aware of the terms required for performance and do not object to them, this establishes their previous performance and dealings. Furthermore, the determination of the terms or meaning of the agreement can be established if any performance is accepted and acquiesced to by the other party without objection under UCC Section 2-208(1). The distributor has regularly distributed the company's air conditioners, so the company based its decision to ship 250 air conditioners to the distributor based on their previous dealings. Therefore, the company would be justified in taking the distributor's sale of the first shipment of air conditioners as an acceptance of the terms offered in the company's letter.

An owner of a pizza parlor hired a delivery driver to deliver pizzas. One evening while the delivery driver was returning from delivering a pizza, he negligently ran through a red light and collided with a car seriously injuring a passenger. If the passenger wins a judgment against the owner and the owner pays the judgment, how much, if any, can the owner recover from the delivery driver?

The entire amount paid to the passenger. Discussion of correct answer:Judgment has been entered against the owner and in favor of the passenger on a vicarious liability theory. The owner has not been negligent in his own actions, but is held responsible for the delivery driver's actions. Accordingly, the owner can obtain complete indemnification from the delivery driver.

Six years ago a contractor installed a heating system into a building. Recently the heating system malfunctioned and the pipes burst in the building causing $80,000 worth of damage. The owner of the building filed suit in the appropriate U.S. District Court. The contractor timely filed an answer denying negligence and asserting the statute of limitations as an affirmative defense. A former employee of the contractor, who worked for the contractor at the time of the installation, provided the building owner with an affidavit stating that the contractor knew that the model of the heating system that was installed had significant disadvantages compared to other models the contractor could have used. The building owner filed a timely motion for summary judgment relying on the former employee's affidavit. The court denied the owner's motion. Why was the denial of the motion for summary judgment proper?

The former employee's affidavit did not address the contractor's affirmative defense, and genuine issues of fact remain as to the negligence of the contractor based upon the former employee's affidavit. Discussion of correct answer:Pursuant to Federal Rule of Civil Procedure 56, a party is entitled to a summary judgment where as a consequence of there being "no genuine dispute as to any material fact," the party is entitled to judgment as a matter of law. Here, even if the facts in the former employee's affidavit are accepted as true, genuine issues of material fact remain as to whether the contractor was negligent. In addition, the affirmative defense was not addressed. For these reasons, it was proper for the court to deny the motion for summary judgment.

In an effort to boost declining customer traffic, a local mall installed a carousel. Because it was the only carousel for hundreds of miles, the mall owners hoped children would drag their parents to the mall. The mall hired a technician company to install, operate, and maintain the carousel. The plan worked beyond anyone's expectations as traffic in the mall increased 200%. One day, a five-year-old girl came to the mall with her mother. After her mother bought hiking boots, the girl got her promised ride on the carousel. As the girl was boarding the carousel, she tripped and fell against part of the mechanism that made the horses go up and down. Someone had neglected to replace the protective housing that normally covered the machinery. The girl's hand was caught in the moving gears. A mall employee was walking by the carousel when the girl fell and was the closest person to the scene. Had the employee grabbed the girl quickly, the girl would only have suffered superficial cuts and bruises. Instead, the employee stared motionlessly at the girl, mouth gaping in amazement. As a result of the delay, the girl's hand was pulled further into the gear mechanism and three fingers were broken. If the girl's mother files suit against the mall, who will prevail?

The girl, if an employee of the technician company was negligent in not replacing the engine housing. Discussion of correct answer:The mall hired the technician company, an independent contractor, to install, operate, and maintain their carousel. In most jurisdictions, the owner of a commercial enterprise open to business invitees cannot delegate responsibility for the maintenance of the public premises to an independent contractor. If the contractor is negligent in the construction or maintenance of the carousel, the business will also be liable.

A bartender was very angry with his girlfriend because she ended their relationship to go out with another man. As punishment, the bartender decided to provide the police with false information so that the girlfriend would be charged with robbery. At the criminal proceedings, the jury found the girlfriend innocent. On advice of counsel, the girlfriend sued the bartender for malicious prosecution. The bartender was then found liable for malicious prosecution. What type of damages can the girl recover?

The girlfriend can recover for the harm to her reputation resulting from the accusation brought against her and for the emotional distress resulting from the bringing of the proceedings. Discussion of correct answer:When a plaintiff establishes the essential elements of a cause of action for malicious prosecution, she is entitled to recover damages for: (1) the harm to her reputation resulting from the accusation brought against her; and (2) the emotional distress resulting from the bringing of the proceedings. In this case, the girlfriend established the essential elements of the cause of action for malicious prosecution and the man was found liable. As such, she will be able to recover for both the harm to her reputation and for emotional distress.

A homeowner owned property upon which there were several extremely large boulders. To build a residence on the property as he desired, the homeowner would have to remove them. The homeowner entered into a contract with a man, who owned a powerful crane and a fleet of trucks, to remove the boulders and deliver them to the city zoo, where they were to be used as part of the enclosure for a rhinoceros habitat. The man lifted the boulders with his crane and placed them on flatbed trucks which he customarily used for hauling bulky objects. Because the boulders were unusually heavy, they overwhelmed the ability of one of the trucks' suspension system to maintain stability, and as the truck attempted to make a gentle turn on the way to the zoo, it overturned, spilling the boulders it carried. One of these rolled through a neighbor's front yard and smashed into his house, causing extensive damage. If the neighbor brings an action against the homeowner for the damage caused by the boulder, which of the following is his best theory of liability?

The homeowner is liable for the man's negligence if the man was engaged in an activity which was inherently dangerous and required special precautions. Discussion of correct answer:An exception exists when the contractor is engaged in a peculiarly hazardous activity that presents risks that require unusual precautions, often referred to as an "inherently dangerous" activity. Such activities include, but are not limited to, abnormally dangerous pursuits for which strict liability might be applicable, but are a broader category--the rule applies to any activity which poses a unique and serious risk. Here, the unusually heavy boulders required special precautions to insure that the extra weight, beyond what the flat bed trucks would normally carry, would not destabilize the truck and result in damage from falling boulders. In such an instance, the homeowner is vicariously liable for the man's negligent failure to undertake those special precautions. Discussion

A house painter decided to get a dog. Before she could adopt one, however, she needed to fence in her back yard. She had lived next door to the same neighbor, a seamstress, for five years. The house painter consulted with the seamstress about where the property line between their houses was located. Neither one of them was certain but because there was only 15 feet between their houses, they just agreed to split it down the middle. The fence was built and the house painter got her dog. Seventeen years later, the house painter still lived on the property, but the seamstress had recently sold her house and moved. The fence was in serious need of repair, so the house painter decided to replace it altogether. She had the fence removed, but when the fence company came to erect the new fence, the new neighbor stopped them and claimed that it would have to be moved two feet closer to the house painter's house because for all these years, the old fence had been encroaching on his property. He'd discovered the mistake with a survey. The house painter disagreed and furthermore, did not want to move the fence because she was growing award-winning roses right where the new neighbor wanted her to move the fence. If the neighbors go to court to settle the boundary dispute, who will prevail?

The house painter will prevail, because the boundary line became fixed by the agreement between the house painter and the seamstress 17 years earlier. Discussion of correct answer: Oral agreements to settle a boundary dispute are enforceable if the parties subsequently accept the line for a sufficient time. The seamstress and the house painter did not know where the line was located, so they agreed on a boundary line and for 17 years, that agreement stood. The court will very likely find that enough time has passed since the agreement to make it enforceable in this case. When the new neighbor bought his house, he took it subject to that agreement. If the survey was performed before closing, showing that the fence appeared to be in the wrong place, he could have negotiated a different price with the seamstress, but he may not force a change in the boundary line now.

One morning while a grandmother was driving to visit her grandchildren, a motorist ran a stop sign and hit her car broadside. The grandmother sustained severe trauma to her head and was in the hospital for several weeks. Eventually, she was well enough to go home. However, the trauma to her head caused an injury to her inner ear. As a result, the grandmother occasionally had trouble with her balance. One day, while the grandmother was at home, she had a sudden attack of vertigo, lost her balance, and fell. Unbeknownst to the motorist, the grandmother had osteoporosis, which made her bones very fragile. As a result, when she fell, she broke her hip. The grandmother sued the motorist for damages. For which of her physical injuries will the grandmother be able to recover from the motorist?

The injuries to her head and to her hip, because the motorist's negligent conduct was the cause in fact of the grandmother's injuries. Discussion of correct answer: Under the so-called "thin-skulled" or "eggshell" plaintiff rule, a defendant is liable for the full consequences of a plaintiff's injury even though, due to the plaintiff's peculiar susceptibility to harm (of which the defendant was unaware), those consequences were more severe than they would have been in a normal person. In this case, but for the motorist's negligence, the grandmother's injuries would not have occurred. Thus, the motorist's negligent conduct was the cause in fact of the grandmother's injuries. Because, under the circumstances, the doctrine of proximate cause provides no basis for limiting the motorist's liability, he is liable for all of the grandmother's injuries. As such, this answer is correct.

While swimming in a lake, a 17-year-old was struck by a jet skier and suffered serious injuries. While the teenager was in the hospital, the jet skier's insurer offered to settle with her for $15,000 because the insurer believed the teenager was an adult based on her representation. The teenager signed a release with the insurer and received a notification from the company that it would pay her by check within 60 days. She left the hospital shortly thereafter. The hospital then presented her with a bill for $15,000. The teenager, in turn, gave the hospital a promissory note for $15,000. The teenager also indicated in her note that it was due in 60 days and that she assigned to the hospital the settlement she would receive from the insurance company. A few weeks later, the insurer discovered that since the teenager was a minor, she did not have the capacity to sign the release. The insurance company took the position that because the teenager had fraudulently represented that she was an adult, the release was invalid. The insurance company refused to pay the $15,000 to either the teenager or the hospital. Which defenses can the insurance company raise?

The insurance company can raise the defense of the teenager's fraud, but not incapacity, as against the hospital's claim. Discussion of correct answer:The basic rule concerning the rights of an assignee against an obligor is that an assignee gets whatever rights to the contract his assignor had and the assignee takes subject to whatever defenses the obligor could have raised against the assignor (e.g., lack of consideration, incapacity, fraud, duress, mistake, etc.). Therefore, if the insurance company in this case can invalidate the agreement to pay the teenager the $15,000 on the basis of the teenager's fraud, the insurance company can also assert this defense against the hospital, to whom the right to collect the $15,000 was assigned. However, only the minor (or an assignee of the minor) has the right to disaffirm the contract based on incapacity; the other party to the agreement cannot assert the minor's lack of capacity as a means of voiding the contract. As such, the insurance company could not assert the teenager's incapacity as a defense against the hospital's claim.

A woman was interested in purchasing a wooded parcel of land owned by her co-worker. After intense negotiations, the parties entered into a purchase and sale agreement, which provided that the woman would pay her co-worker $100,000 in exchange for a warranty deed for the property from the co-worker. This amount was roughly the fair market value of the property at the time. The woman intended to clear the land and sell the lumber for a substantial profit. But before the closing date and before the woman took possession of the land, a fire started on neighboring property, through no fault of either party, which destroyed all of the timber on the land. This fire reduced the value of the land by nearly half. The woman now seeks to avoid the contract in a specific performance action brought by the co-worker. What is the best explanation to support a judgment for the woman?

The jurisdiction has adopted the Uniform Vendor and Purchaser Risk of Loss Act. Discussion of correct answer:If the jurisdiction has adopted the modern Uniform Vendor and Purchaser Risk of Loss Act, the purchaser is deemed the true owner of the land (and thus bears the risk of loss) only as of the date of the closing or the date the purchaser goes into possession of the land, whichever comes first. As the fire took place before the closing and before the woman took possession of the land, the co-worker would be treated as the owner at the time of the fire and would, therefore, bear the risk of loss.

Two days after a man filed an action against two out-of-state defendants, one of the out-of-state defendants agreed to settle. The man then voluntarily dismissed the entire action as to both defendants. The second defendant wished to continue the lawsuit, and asked the court to rule against dismissal. Which of the following is most accurate?

The man may dismiss the action, because the second defendant had not yet answered. Discussion of correct answer:Under Fed. R. Civ. P. 41, a plaintiff may voluntarily dismiss an action by filing a notice of dismissal at any time prior to service of an adverse party's answer or motion for summary judgment. Here, the plaintiff has agreed to settle with one party, and has yet to receive an answer from the second.

On the last day of its legislative session for 2020, a state legislature enacted a statute, effective January 1, 2021. It provided that, because all-natural ingredients were healthier than artificial ingredients, all movie theaters had to use real butter, rather than hydrogenated oils or powdered butter substitutes, on popcorn sold in the state's movie theaters. The governor signed the bill into law. On November 1, 2020, an association of movie theaters in the state filed a complaint in state court challenging the constitutionality of the newly enacted statute. What is the state's best argument for dismissal of the complaint?

The matter was not ripe for adjudication. Discussion of correct answer: A case or controversy is a real and substantial dispute that touches the legal relations of parties having adverse interests and that can be resolved by a judicial decree of a conclusive character. A controversy must be ripe for decision so that courts do not waste time deciding constitutional issues that might never need deciding. Thus, a person asking a court to hold a statute unconstitutional must be able to show not only that the statute in question is invalid, but also that he has sustained or is in immediate danger of sustaining some direct injury as a result of its enforcement. Here, because the statute in question was to go into effect on January 1, 2021, and the complaint was filed on November 1, 2020, the state's best argument is that the controversy was not yet ripe for adjudication.

A longtime resident of a close-knit community told another resident that a local businessman never gave anyone a fair deal. The businessman sued the resident for defamation. Which of the following facts would serve to strengthen the businessman's case against the resident?

The resident elaborated that he was referring to the businessman's tendency to cheat during neighborhood poker games. Discussion of correct answer: Where the plaintiff in a defamation action is a private party and the subject matter is not a matter of public concern, the plaintiff need not prove actual malice. In contrast, a public figure plaintiff must prove that the defendant acted with malice (knowing falsity or recklessness as to truth or falsity). In this case, if the resident elaborated that he was referring to the businessman's tendency to cheat during neighborhood poker games, it would appear that the resident's comment related to a private matter, rather than a public matter, so the businessman would not be required to prove malice. Therefore, this fact would tend to support the businessman's defamation claim.

An attorney filed a complaint in federal court on behalf of his clients, alleging a breach of fiduciary duty involving a will. The complaint was based on diversity, stating "the plaintiffs and some of the defendants are citizens of different states." The plaintiffs were citizens of State A, and the complaint stated that one of the defendants was a citizen of State B. The complaint did not allege the other defendants' citizenship precisely, but it stated that many of them resided in State A. The complaint went on to say that finding out the citizenship of each defendant would be unduly burdensome on the plaintiffs. Upon receiving the complaint, the attorney for the State B defendant immediately filed a motion for sanctions with the court, alleging that the complaint "as filed, does not state appropriate grounds for diversity jurisdiction." Courts in this jurisdiction routinely applied the safe harbor provision to complaints. Ultimately, the court granted the sanctions. The plaintiffs' attorney appealed, arguing the sanctions were inappropriate. How should the appeals court rule?

The sanctions were inappropriate, because the motion needed to be served on the plaintiffs first. Discussion of correct answer: A party may not file a motion for sanctions without first serving the motion upon the opposing party and providing the opposing party with 21 days to withdraw or correct the offending pleading, written motion, or other paper. This is called the "safe harbor provision." However, not all courts apply the safe harbor provision to complaints. Here, because the courts in this jurisdiction apply the safe harbor provision to complaints and the defendant's attorney immediately filed the motion for sanctions with the court, the granting of sanctions was inappropriate.

A father owned a large ranch in fee simple absolute. He had always intended that his son and daughter would take over the ranch when he could no longer operate it alone. To that end, the father had his lawyer prepare a deed conveying the ranch to his children "as tenants in common." When the father presented his children with the deed, he asked them to give each other the right of first refusal before transferring either of their interests to a third party. The son and daughter agreed, placing their hands on a Bible and solemnly swearing to honor their father's request. The son and daughter ran the ranch together for five years without incident. However, the son preferred technology to nature and eventually left to work in the city, giving the daughter no indication of when, if ever, he would return. The daughter continued to operate the ranch successfully without the son's assistance, and enjoyed substantial profits that she did not share with him. After four years of operating the ranch on her own, the daughter decided that she could no longer handle the large property without help and conveyed her interest to her faithful ranch-hand. The son was angry to learn of the daughter's conveyance of her interest to the ranch-hand, and he sought a declaratory judgment deeming the conveyance invalid in light of the daughter's agreement to give the son the right of first refusal. What is the most likely outcome of the lawsuit?

The son will not prevail, because the right of first refusal agreement was unenforceable as violating the Statute of Frauds. Discussion of correct answer: A promise to sell or transfer an interest in land must satisfy the Statute of Frauds requirements that the agreement be in writing, signed by the party to be charged, and contain the essential contract terms. A promise to give another the right of first refusal is an agreement concerning an interest in land that is within the Statute of Frauds. The oral promise by the son and daughter, even if sworn on a Bible, does not satisfy the statute's requirements. Therefore, the son will fail in his effort to void the daughter's conveyance of the ranch to the ranch-hand.

A grandfather was leasing a garage where he kept his cars and motorcycles. The grandfather knew that he had a good deal, because he had been renting that garage for a long time. One day, the grandfather learned that his son was going to need space for some of the son's cars. Since the grandfather wanted to move to another city, he decided to transfer the lease to his son. The grandfather signed a contract with the owner of the garage by which the contract would be transferred over to the son in one year and the son would receive the same rate. The contract also stated that the contract could not be modified after three months. Which of the following is correct?

The son's rights in the garage lease will vest in three months. Discussion of correct answer:Under contemporary law, third-party beneficiaries may have standing to sue the promisor for breach, even though the promise was made to the promisee and not to the third-party beneficiary. The question here is when do the beneficiary's rights vest. If an express term in a contract provides for vesting, then vesting with regard to an intended beneficiary will occur when that term is satisfied. In this case, the contract stated that the contract cannot be modified after three months, so the son's rights will vest after three months.

Before setting out on a morning hike in the mountains, a hiker slathered her face with sunscreen to protect herself from the strong sun. Along the trail, the hiker encountered a group of Cub Scouts, some of whom were playing with water pistols. One of the Cub Scouts accidentally sprayed the hiker with water, and sunscreen leaked into her eyes, causing her a great deal of pain and a temporary loss of vision. The hiker stopped on the trail to wipe her eyes just as a competitive trail runner was sprinting down the trail with a large backpack. To avoid colliding with hikers on the trail, the trail runner routinely yelled, "Coming down, move to the side, please!" Although the hiker heard the trail runner's call, she could not see sufficiently to know in which direction she should move, and the trail runner barreled into her, sending the hiker tumbling down the trail. The hiker suffered a fractured pelvis and severe concussion in the fall. The trail runner suffered a sprained ankle and shattered knee that never fully healed, which forced him to give up trail running. The hiker filed a negligence claim against the trail runner and the Cub Scouts. The trail runner filed a counterclaim against the hiker, asserting that she was negligent in failing to move to the side of the trail to wipe the sunscreen from her eyes. Which of the following arguments will be most helpful to the hiker in defending against the trail runner's counterclaim?

The trail runner assumed the risk of injury by running down steep mountain trails. Discussion of correct answer: The hiker's strongest defense against the trail runner's counterclaim is that the trail runner assumed the risks inherent in trail running, including the risk of colliding with other hikers on the trail. Inherent risks are those risks that are unavoidable despite the exercise of all due care. Here, regardless of his attempts to warn hikers, it was certainly foreseeable that the trail runner would encounter hikers on the trail, some of whom would not manage to heed his warning in time to avoid a collision. As such, this is the best answer.

While driving well within the speed limit, a driver came around a sharp curve on a mountain road on a clear quiet night and suddenly encountered an enormous elk. It was impossible for the driver to stop in time to avoid hitting the elk, which was killed instantly. The driver recognized that the elk in the road posed a risk of additional collisions by subsequent drivers, but he made no attempt to remove it or set up some sort of warning device. Just as the driver was driving away, a woman, also proceeding within the speed limit, rounded the same curve in a subcompact car and, being similarly unable to stop in time, slammed into the elk carcass and was seriously injured. A third driver was driving right behind the woman, heard the woman's squealing brakes before he actually rounded the curve, and was able to avoid a third collision. The third driver ran to the woman's rescue but, having successfully removed the woman from the perilous position in the subcompact in the middle of the road, suffered a heart attack. If the third driver sues the woman for his damages resulting from the heart attack, what will be the probable outcome?

The woman will win, because she was not negligent. Discussion of correct answer:The woman can only be liable for the third driver's injury if the woman breached a duty of care to the third driver. The woman was driving within the speed limit and was unable to avoid hitting the elk carcass because of its proximity to the curve. There is no evidence that the woman was negligent. Rescuers may be owed a duty of care but recovery requires that the duty of care be breached.

A woman and a man, who worked for the same company, regularly commuted to work together. Each drove on alternate days. One day when the woman was to drive, she arrived at the man's house at the appointed time. Shortly after they left, the woman informed him that she had received a notice in the mail the day before from the manufacturer of her car warning her that, due to the manufacturer's negligence, there was a defect in her car's brake system and she should not drive the car until the brakes were repaired. The man stated they should return to his house and take his car, but the woman replied that there was a dealership next to their place of work and she would have the brakes repaired during the day. She further stated that she was sure the brakes would hold up on the trip to work. Shortly thereafter, the traffic in front of the woman came to a stop. When she applied the brakes they failed, causing the woman's car to strike the stopped car in front of her. The man was injured in the accident. If the man brings a negligence action against the manufacturer, which of the following most accurately describes the significance of the woman's negligence to the litigation?

The woman's negligence was a foreseeable danger the risk of which was increased by the manufacturer's negligence. Discussion of correct answer:The woman's negligence in driving the car knowing of the potential problem is an independent, intervening force which contributed to the man's injuries, but it does not relieve the manufacturer of liability because it is the type of danger the risk of which would foreseeably be increased by the manufacturer's negligence.

A tomato genetics stock researcher had developed and produced a variety of square tomato. The researcher believed that this would revolutionize sandwich-making. On May 1, the researcher orally agreed to sell five tons of this square tomato to a sandwich shop manager for $5,000. The $5,000 was payable on May 31. On May 5, the researcher dictated the agreement to the researcher's secretary, who was more familiar with typing research papers than with typing contracts. She inadvertently typed in $4,000, instead of $5,000 as the price. Neither the researcher nor the manager noticed the mistake in the purchase price. Both signed the agreement as it was typed. The manager refused to pay more than $4,000 for the square tomatoes, and the researcher filed an action for the additional $1,000. Which of the following is the strongest argument in favor of the researcher?

There was a mistake in integration. Discussion of correct answer:Here, the secretary's error in typing $4,000 instead of $5,000 would be viewed as a mistake in integration, or a "scrivener's error." Reformation is available to cure a mistake of this sort. Thus, if it can be determined that the parties intended that the final agreement contain a sale price of $5,000, that is the price the manager will be required to pay.

A railroad worker's widow brought a wrongful death action in federal court against the railroad, claiming that its negligence had caused her husband's death. At trial, the widow offered the testimony of a coworker of the husband. The coworker testified that he had seen the rail car on which the husband was riding slow down and the cars behind it gain speed. The coworker also stated that he later heard a loud crash, but did not turn around to look because loud noises were common in the yard. Three other railroad employees testified that no collision had occurred. At the close of the evidence, the railroad moved for judgment as a matter of law, which was denied, and the case was submitted to the jury. The jury returned a verdict for the widow. The railroad has made a renewed motion for judgment as a matter of law. What standard should the court apply to determine how to rule on the motion?

Whether there is substantial evidence in the record to support the verdict, resolving all disputed issues in the widow's favor. Discussion of correct answer: Answer (D) is correct because the standard for a judgment as a matter of law (JMOL) is identical to that of a motion for summary judgment, although a JMOL motion is based on evidence actually introduced at trial. A JMOL motion will be granted if the court finds that a reasonable jury would not have a legally sufficient evidentiary basis to find for the nonmoving party [Fed. R. Civ. P. 50(a)]. In ruling on the motion, a court will view the evidence in the light most favorable to the nonmoving party to determine whether the movant is entitled to prevail as a matter of law. Consequently, the court will look at whether there is substantial evidence in the record to support the verdict, resolving all disputed issues in the widow's favor because she is the nonmoving party.

A congressional study of the sports programs in public schools throughout the country verified anecdotal evidence that non-white students are discouraged from participating in certain sports, such as golf, tennis, and field hockey, while being encouraged to develop skills in basketball, track, and soccer. As a result, Congress created the Commission for Non-Biased Participation in Sports to promote the active participation of all school-age children in every sport offered in the public schools. The commission was empowered to oversee the administration of sports programs of school districts and to order compliance with federal regulations promulgated by the commission. Headed by an executive director appointed by the president, the commission rendered decisions on school programs brought before the commission for violation of federal regulations. Appeals were made to the Vice President, with any decision subject to judicial review by the federal courts. If a school district charged with violations of the commission's regulations challenges the legislation creating the commission, is the court likely to uphold the legislation creating the commission?

Yes, because Congress is authorized to enact legislation to prohibit violations of Fourteenth Amendment rights. Discussion of correct answer: Congress has the authority to enforce civil rights pursuant to the Fourteenth Amendment Enabling Clause. The study of school district sports programs conducted by Congress showed discrimination based on race or national origin in violation of nonwhite students' equal protection rights. Because the state-run sports programs have been shown to interfere with students' Fourteenth Amendment rights, Congress has the authority to legislate to eliminate the restrictions placed on the students' exercise of those rights.

A man sued an aircraft manufacturer for injuries suffered when a plane they manufactured crashed in France. All of the witnesses and the wreckage of the plane are in France. France has jurisdiction over cases where the accident occurred within the country. The plaintiff is a citizen of State A and sued in the United States District Court for the District of State B, where the manufacturer is incorporated, has its principal place of business, and operates its only manufacturing facility. The manufacturer filed a motion to dismiss the case under the doctrine of forum non conveniens, arguing that the case should be brought in France, where the witnesses are and the accident occurred. May the court grant the motion to dismiss?

Yes, because France would be a much more convenient forum. Discussion of correct answer:Dismissal on the basis of the forum non conveniens doctrine is appropriate when there is a much more convenient forum outside the United States federal court system. Here, although the needs for venue, personal jurisdiction, and subject matter jurisdiction are all satisfied where the plaintiff brought the case, France would be a much more convenient forum because the witnesses and the wreckage are there. The court may dismiss the case in contemplation of it being brought in this much more convenient forum.

A bakery incorporated and headquartered in State A had a dispute with a mill incorporated and headquartered in State B over the quality of the flour the mill had delivered to the bakery. The bakery sued the mill in a federal court in State A for breach of contract, seeking $100,000 in damages. The contract between the bakery and the mill contained a clause designating State B courts as the sole venue for litigating disputes arising under the contract. Under precedent of the highest court in State A, forum-selection clauses are unenforceable as against public policy; under U.S. Supreme Court precedent, such clauses are enforceable. The mill has moved to transfer the case to a federal court in State B, citing the forum-selection clause in the parties' contract and asserting the facts that the flour was produced in State B and that the majority of likely witnesses are in State B. Is the court likely to grant the mill's motion?

Yes, because federal law governs transfers of venue, and it would be more convenient for the witnesses and parties to litigate the claim in State B. Discussion of correct answer: Answer (D) is correct. Federal law governs transfers of venue. If an action is brought in a particular venue, a federal court may transfer the action to any other district in which it might have been brought "for the convenience of parties and witness, in the interest of justice" [28 U.S.C. Sec. 1404(a)]. While federal courts usually enforce valid forum-selection clauses, the federal court sitting in State A would follow the law of State A and find that the forum-selection clause is unenforceable as against public policy. However, because the majority of witnesses are in State B and the bakery could have sued the mill in State B as a State B citizen, it would be proper for the federal court to transfer the claim to State B.

A defendant is arrested and charged with embezzlement. At trial, the prosecution calls a woman to the stand. The woman testifies that, shortly before the defendant was arrested, he told her, "I'm going to have to get out of the country or something because I embezzled money from my boss." After the woman testifies as to the defendant's statement, the prosecution begins questioning the woman about her relationship to the defendant, expecting her to testify that she and the defendant had a long-term extramarital relationship. Is the woman's testimony about her relationship to the defendant admissible?

Yes, because it is relevant to the issue of why the defendant would make such a statement to the woman. Discussion of correct answer: Evidence is relevant if it tends to make a material fact more probable or less probable than it would be without the evidence. With some significant exceptions, relevant evidence is admissible. Here, the testimony about the relationship between the woman and the defendant is relevant to the issue of the defendant's guilt, because, if they had a close relationship, it is more probable that the defendant made the incriminating statement to the woman. Also, knowing the witness's bias or interest helps the jury determine the witness's veracity.

A woman went shopping for a birthday present for her husband at the local mall. The mall has a lighted parking area but there is no security staff that patrols the parking lot. The mall is located in an area with a high crime rate. As the woman was getting into her car, a criminal grabbed her purse and placed a gun to her head. The criminal threatened to shoot her if she did not give him her shopping bags. She resisted and he hit her with the gun before fleeing with her purse and the presents she bought for her husband. In a negligence suit for damages against the mall, will the woman likely prevail?

Yes, because it was foreseeable that in a high crime area criminals could pose a danger to customers going to their vehicles in the parking lot. Discussion of correct answer:Generally, the criminal or tortious acts of a third party constitute a superseding intervening cause that breaks the chain of causation. However, when the facts and circumstances demonstrate that the criminal act is foreseeable, the chain of causation will not be broken. The facts indicate that the mall is located in a high crime area and the mall does not use security guards. Therefore, these facts and circumstances indicate that a criminal act is foreseeable.

The plaintiff purchased a small, decorative glass table from her favorite home furniture and decoration store. The table is very small and the glass that makes up the surface of the table is very thin. The store displayed the table in its "decorations" section. When the plaintiff brought the table home, she placed it in her living room. A few days later, when the plaintiff came home from work, she lightly tossed her purse on the table. The table shattered, and the plaintiff was injured by shards of glass. If the plaintiff brings an action against the store under a products liability theory, will she succeed?

Yes, because it was foreseeable the plaintiff would use the table for this purpose. Discussion of correct answer:Misuse is a defense to a claim of products liability. If a plaintiff uses a product in a manner that is neither intended nor foreseeable, she has misused the product and it cannot be defective. While the intended use for the table in this case is for the table to be a decoration, it is foreseeable that a person would set something on the table (even lightly toss something soft onto the table, like a purse); thus, there is no misuse. As such, the plaintiff will likely prevail.

A gun owner owned an outdoor firing range on the outskirts of a city. While the gun owner was on vacation, the city hosted the fifth annual mercenary soldiers convention. Several of the participants stopped by the gun owner's firing range and rented and fired automatic weapons, which were legal in the jurisdiction. The convention members were not content to fire at the silhouette targets on the firing range. They blasted away at passing birds and at the tops of some nearby trees. The firing range staff took the weapons away from anyone shooting at anything other than a firing range target. The next day, more mercenaries came to the range, and in the course of firing their rented automatic weapons, shot and killed a valuable bull, owned by a farmer, which was kept in an adjacent field. The mercenaries promptly disappeared. The farmer has since brought an action against the gun owner for damages resulting from destruction of the bull. Should the farmer recover?

Yes, because landowners are strictly liable for injuries to property on adjacent lands caused by abnormally dangerous activities conducted on their property. Discussion of correct answer:Land occupiers are not strictly liable for acts on their premises except when the land occupiers are engaged in an abnormally dangerous activity like blasting or nuclear waste storage. Conducting a firing range would be considered an abnormally dangerous activity. Hence the farmer will recover.

A waste disposal company deliberately had one of their ships deposit over a thousand barrels of contaminated waste into a navigable river that is used by many other ships to transport their goods. The dumping took place in State A and the river flows through many states in the region. A town sued the company in federal court, claiming severe damage to its beaches and its tourist business as a result of this dumping. Does the federal court have jurisdiction over the matter?

Yes, because the case has a maritime nexus. Discussion of correct answer:Federal courts have original jurisdiction over admiralty or maritime cases. Admiralty jurisdiction requires that a case have a "maritime nexus." A maritime nexus requires that the incident giving rise to the case had a "potentially disruptive effect on maritime commerce," and the general character of the activity giving rise to the incident shows a "substantial relationship to traditional maritime activity" [Jerome B. Grubart, Inc. v. Great Lakes Dredge & Dock Co., 513 U.S. 527 (1995)]. Here, the action is potentially disruptive to commerce on a large navigable river, and transporting waste by boat is a traditional maritime activity.

The plaintiff sued the defendant in federal court. Diversity jurisdiction existed. The defendant filed an answer and a compulsory counterclaim based upon state law. The amount in controversy on the counterclaim was $50,000. The plaintiff then asked the defendant to stipulate to dismissal of the case voluntarily without prejudice. The defendant refused. The plaintiff filed a motion to dismiss the case voluntarily without prejudice. The court denied the motion. Was the court correct to deny the motion to dismiss?

Yes, because the court would not have had jurisdiction to adjudicate the counterclaim. Discussion of correct answer: Under Federal Rule of Civil Procedure 41(a)(2), a federal court has the power to grant a motion to dismiss a claim voluntarily without prejudice, with one exception. The exception applies here: the court may not dismiss the case over the objection of the defendant if the defendant has filed a counterclaim that could not remain pending for independent adjudication. This defendant filed a counterclaim and objected to dismissal of the plaintiff's claim. Subject-matter jurisdiction over the counterclaim existed only because of its relationship to the plaintiff's claim. A compulsory counterclaim must arise from the same events as the original claim, and so the counterclaim would have been part of the same case or controversy as the original claim. Therefore, although the amount in controversy was insufficient to allow the court to assert diversity jurisdiction over this state law claim by the defendant, there would have been supplemental jurisdiction over it. The basis for that supplemental jurisdiction would disappear if the court granted the plaintiff's motion to dismiss. Because the counterclaim could not remain pending for independent adjudication, the court may not grant the motion to dismiss the original claim.

An employee worked as a sales associate in a high-end antiques shop owned by the employer. Business had been slow lately, and the employer was anxious to make a big sale soon. One afternoon, he approached the employee and told her that a client was coming to view the blue eighteenth-century jelly cupboard and that they really needed that sale. The employee, suddenly inspired, exclaimed that she knew that the client adored raspberry truffles and that the employee was going to pick some up so that the client wouldn't be able to resist the cupboard. The employee dashed out the door to the nearby candy store. However, in the candy store, the employee realized that in her haste she had neglected to ask the employer for cash from the shop's petty cash box--and the client would be coming any minute, so there was no time to go back. The employee decided that this sale was important enough to "bend the rules." She slid a large box of the truffles under her coat and moved toward the exit, but the removal of the truffles caused a much larger display to come tumbling down, resulting in hundreds of dollars in damage to merchandise. The store proprietor witnessed the entire event. He chased down the employee, contacted the police, and he also filed suit against the employer for the damages the candy store proprietor had sustained as the result of the employee's actions. Will the employer be held vicariously liable for the employee's actions?

Yes, because the employee's errand was related to the shop business and was therefore within the scope of her employment. Discussion of correct answer:Under the respondeat superior doctrine, an employer is liable even for the intentionally tortious acts of his employees, so long as they are committed within the scope of employment. Here, although the employee's actions were intentionally tortious, they were closely connected and related to the shop business and were therefore within the scope of her employment. As such, the employer will be vicariously liable for the employee's act.

A man sued a woman after a hunting accident left him severely injured. The man was a citizen of State A, while the woman was a citizen of State B. The man's injuries totaled over $300,000 in damages. The man brought his suit in a federal court in State A. In his suit, the man asked for special damages for loss of consortium. The majority of states did not allow such damages, but a small minority did so. The minority that did so had adopted legislation specifically stating that such damages were available. State A had yet to pass legislation on the matter, but State A courts had historically followed the minority rule. Would the man be entitled to pursue such damages in his claim?

Yes, because the federal court's jurisdiction is based on diversity. Discussion of correct answer:The Erie doctrine states that a federal court that hears a state law claim in a case based on diversity or supplemental jurisdiction must apply the substantive law of the state in which the court sits [Erie R.R. Co. v. Tompkins, 304 U.S. 64 (1938)]. Here, the claim is for personal injury, which is a state law claim. However, it can be heard in federal court because there is complete diversity of citizenship and the amount in controversy is over $75,000. Importantly, State A courts allow the damages, and there is no rule stating that a federal court can only apply the statutory laws of a state as opposed to its common law.

A club manager and an investor decided to establish a new nightclub. The club manager donated the use of his loft, and the investor provided the initial start-up capital. The investor did not wish to be bothered with day-to-day operations, so he and the club manager entered into a written agreement limiting the investor's employment duties to that of official host at the new club. As the host, the investor would be responsible for maintaining a well-ordered club environment. The nightclub was a big success, and the club was packed with people on a nightly basis. To prevent the club manager from burning out from exhaustion due to the late club hours, the investor and the club manager verbally agreed that the investor would take a turn running the club on the first Friday of every month. On the first Friday in September, the investor noticed an argument developing at a table where a man and a woman were seated. He approached the table and asked if there was anything he could do for the nightclub guests. The man, who was seated at the table with his date, said, "You can stop serving this swill." When the date told the man to calm down, the man became so angry that he hurled his cocktail glass to the floor, smashing it. The investor ordered the man to leave, and when the man refused, the investor physically forced him out of the nightclub. The man broke his leg when the investor tossed the man out onto the street. The man sues the club manager and the investor. Will the club manager be liable for the man's injury?

Yes, because the investor and the club manager entered into a joint venture. Discussion of correct answer:When two or more individuals agree to enter into an undertaking in the performance of which they have a community of interest and mutual right of control, they are said to be engaging in a joint venture. Because such an arrangement is similar to a business partnership, each member is vicariously liable for any torts committed by the others within the scope of the enterprise. Here, the investor and the club manager jointly created the nightclub. The club manager provided the use of his loft, and the investor provided the start-up capital. That they both contributed to the business venture from which they would both benefit shows their joint undertaking. The investor and the club manager had a community of interest in the nightclub. As host, the investor was responsible for maintaining a well-ordered environment. While the investor initially attempted to limit the scope of his employment, he and the club manager subsequently agreed to add to the investor's duties by his managing the club on first Friday evenings. Because they agreed that the investor would run the club on certain occasions, his duties, risks, and liabilities also expanded. Both parties had a mutual right of control in the nightclub operations when they formed a joint venture. Because the investor was acting within the scope of his employment agreement and within the scope of the enterprise when he escorted the man from the nightclub and tossed him on the street, the club manager will be vicariously liable for any torts committed by the investor. Therefore, the club manager will be vicariously liable for the man's injuries and damages. This is the correct answer.

A homeowner granted an easement to the local utility company to grade a road across the edge of her property, on a bluff behind her house. The road was graded according to proper safety protocols, and the utility company used it for many years. However, the utility company failed to maintain the road. Over the years, the trucks which the utility company sent over the road grew larger and heavier, and the edges of the road gradually began to erode. Many years later, the homeowner noticed a small landslide from the side of the road following an unusually heavy rain. She contacted the utility company but received no answer. Because this was the first time such a thing had happened, and the amount of rainfall was unusual, she forgot all about the incident. A year later, during a storm, the edge of the road crumbled and started a landslide. The homeowner was away at the time, but when she came home, she found several inches of mud covering everything on the floor. She sued the utility company, arguing that its negligence caused the landslide which ruined many of her possessions. Will the homeowner prevail?

Yes, because the owner of the easement is responsible for reasonable maintenance and repairs of any improvements made to the easement. Discussion of correct answer:The owner of the easement has a duty to maintain the easement. In this case, the utility company failed to maintain the road, thereby causing the erosion and subsequent landslide which resulted in the homeowner's property damage. As such, the utility company is liable for the homeowner's damages.

A plaintiff filed an action in federal district court and served the defendant with the summons and complaint. The defendant moved to dismiss the complaint for failure to state a claim. Instead of opposing the motion to dismiss, the plaintiff voluntarily dismissed the action and filed a new action, alleging the same claims but also addressing the pleading defects outlined in the defendant's motion to dismiss. The defendant then moved to dismiss the second action, and the plaintiff again voluntarily dismissed the second action instead of filing opposition papers. The plaintiff then filed a third action, alleging the same claims but also including additional allegations that were responsive to the defendant's second motion. The defendant has moved to dismiss the third action; the plaintiff opposes the motion. Is the court likely to grant the defendant's motion?

Yes, because the plaintiff's previously dismissed actions asserting the same claims operate as an adjudication on the merits. Discussion of correct answer: Answer (D) is correct. If a plaintiff has previously dismissed an action based on or including the same claim, a dismissal acts as an adjudication on the merits [Fed. R. Civ. P. 41(a)(1)(B)]. Because the plaintiff here had already dismissed the same claims before, this dismissal acts as an adjudication on the merits and the defendant is entitled to dismissal.

A plaintiff obtained an ex parte temporary restraining order (TRO) in a federal civil action and posted a bond the same day. One day later, the plaintiff served the defendant with copies of the summons and complaint, the TRO, and supporting documents. The court scheduled a hearing on the preliminary injunction to occur 14 days after the TRO had been issued. Two days before the hearing, the plaintiff moved to extend the TRO and postpone the hearing for one week on the ground that its principal witness would be unavailable to testify on the scheduled day due to a planned vacation. The defendant opposed the motion and moved to dissolve the TRO. Is the court likely to grant the defendant's motion to dissolve the TRO?

Yes, because the plaintiff's principal witness's vacation is not good cause to extend the TRO. Discussion of correct answer: Under Federal Rule of Civil Procedure 65(b), a TRO expires after 14 days. An extension of the TRO before the hearing is dependent on a showing of good cause. The plaintiff should have known the availability of its principal witness before seeking a TRO. The witness's vacation was planned, so there was no surprise in the witness's unavailability. Therefore, the plaintiff cannot show good cause for extending the TRO.

Two ranchers, both citizens of State A, brought an action in a state court in State A against a developer, a citizen of State B. The ranchers alleged a state-law tort claim for water runoff damage to their properties caused by construction on the developer's neighboring property. The first rancher claimed $250,000 in damages and the second rancher claimed $50,000. In their complaint, the ranchers cited federal law regarding the calculation of damages due to water runoff. The developer timely removed the action to federal court. Is removal proper?

Yes, because the ranchers are diverse from the developer and both ranchers' claims arise from the same facts. Discussion of correct answer:The Supreme Court has held that the supplemental jurisdiction statute, 28 U.S.C. Sec. 1367, authorizes jurisdiction over claims that otherwise would not meet the amount-in-controversy requirement for federal-diversity jurisdiction. See Exxon Mobil Corp. v. Allapattah Services, 545 U.S. 546 (2005). The supplemental jurisdiction statute requires that the insufficient claims be so related to claims in the action that are within the court's original jurisdiction that they form part of the "same case or controversy." In this action, both ranchers' claims arise from water runoff caused by the same construction on the neighboring property and thus meet that standard. Because the supplemental claim (the second rancher's $50,000 claim) is being asserted by a co-plaintiff joined under Federal Rule of Civil Procedure 20, 28 U.S.C. Sec. 1367(b) does not operate to withhold supplemental jurisdiction.

A devoted daughter had not heard from her elderly father for several days. Concerned, the daughter decided to check on him. She entered her father's house and found him on the kitchen floor, unconscious, with a large bruise on his temple. She called 911, and the father was transported to the hospital for treatment. As she was leaving her father's house, the daughter noticed that the television and DVD player that she had given her father for Christmas were missing. The daughter contacted the police and reported the theft of the items. Two days later, police officers came to a suspect's home with a warrant for his arrest on drug charges. When the police entered the suspect's apartment, they immediately saw a television and DVD player in the middle of the living room. The television and DVD player were identified as belonging to the father, and the police arrested the suspect, who was subsequently charged with burglary and battery. At trial, the suspect claimed that he had found the television and DVD player in a trash can in an alley. Because the father had not seen his attacker's face, he was unable to identify the suspect as his attacker, and there was no physical evidence connecting the suspect to the incident. The suspect was acquitted of the burglary and battery charges. A short time later, the father, who had never fully recovered from the attack, died from a blood clot caused by the injury, and the suspect was charged with felony murder. At trial, the suspect moved for dismissal of the case on the ground that the felony murder charge violated the Double Jeopardy Clause. Should the suspect's motion be granted?

Yes, because the suspect was acquitted of the burglary charge. Discussion of correct answer: The Fifth Amendment prohibition on double jeopardy forbids a defendant from being tried twice for the same crime arising from the same set of facts. It incorporates the concept of collateral estoppel--a defendant may not be tried for a different crime arising out of the same criminal conduct if a previous prosecution necessarily determined factual issues required for conviction in the defendant's favor. For instance, for a defendant to be guilty of felony murder, the defendant must be found guilty of the underlying felony. Thus, in this case, for the suspect to be convicted of felony murder relating to a burglary, the prosecution would need to re-litigate facts pertaining to the underlying burglary charge. Given that these facts that had already been determined in the suspect's favor during his trial for burglary and battery, the felony murder charge violates the prohibition against double jeopardy and should be dismissed.

A woman wanted to kill a business competitor. She contacted a man who she believed was willing to commit murder for hire and offered him $50,000 to kill the competitor. The man agreed to do so and accepted $25,000 as a down payment. Unbeknownst to the woman, the man was an undercover police officer. In a jurisdiction that has adopted the unilateral theory of conspiracy, is the woman guilty of conspiracy to murder the business competitor?

Yes, because the woman believed that she had an agreement with the man that would bring about the competitor's death. Discussion of correct answer:In jurisdictions that recognize unilateral conspiracies, it is enough that one person agree with another person to commit a crime (and in some jurisdictions, that an overt act in furtherance of that agreement be committed). It is no defense to unilateral conspiracy that the other person was feigning agreement or acting in an undercover capacity. Here, the woman agreed to commit a crime and she committed an overt act in furtherance of that agreement when she paid the man $25,000. She therefore is guilty of conspiracy in a jurisdiction that recognizes unilateral conspiracies.

One evening, a woman was having dinner at a restaurant. After finishing her meal, she went to the coatroom to retrieve her overcoat. However, the woman inadvertently took another patron's overcoat, which was the same color and design as her own. Honestly believing that she had her coat, the woman left the restaurant without noticing the difference. When she arrived home about one hour later, she discovered the mistake. Because it was nearly midnight, she decided to wait and return the coat the next day. The following morning, the woman was driving back to the restaurant with the overcoat when she was involved in a near fatal automobile accident. The overcoat was ripped to shreds and destroyed. If a claim is brought by the coat's owner against the woman to recover the value of the coat, will the owner prevail?

Yes, because the woman intended to take the coat and it was destroyed while in her possession. Discussion of correct answer:This is a very tricky question in which students are required to distinguish between crimes and torts. Because the owner is asserting a claim against the woman for damages, this is a civil action. So, the question really being asked is, "Is the woman liable for conversion?" The answer is yes. The general rule is that a defendant is not relieved of liability for conversion (or trespass to chattels) because of a mistake of law or fact except when such mistake was created by the plaintiff. This means that the woman will remain liable for conversion because the overcoat was ruined while in her possession. Even though the dispossession resulted from the woman's mistake of fact, there was nevertheless a conversion.

A State A athlete filmed a commercial for a State B corporation. Later, the corporation began to use the athlete's image and material from his autobiography in other commercials as well, despite the fact that the athlete had only contracted with the corporation for the one commercial. The athlete sued the corporation in State B state court, alleging that its use of the material from his autobiography violated federal copyright laws. He also asserted a state law contract claim that the corporation had failed to pay expenses of $100,000 associated with the original commercial. Can the corporation successfully remove the case to federal court?

Yes, because there is a federal question. Discussion of correct answer:A federal court has subject-matter jurisdiction where there is a federal question. Here, the athlete has asserted a claim based on federal copyright law, therefore there is a federal question. While there may be a question of whether the court would have supplemental jurisdiction over the contract claim, even if the court does not, it will sever the claims and hear the claims over which it does have jurisdiction.

A lawyer was in possession of a valuable antique watch that was claimed by numerous people. The lawyer did not claim any ownership in the antique watch and was thus just a stakeholder. The claimants were citizens of States A, B, C, D, and E. The lawyer was a citizen of State A. The antique watch was valued at $20,000. The lawyer wished only to file an interpleader action against all of the claimants to determine who owned the antique watch. Would there be federal subject matter jurisdiction over an interpleader action?

Yes, because there is minimal diversity among the claimants and more than $500 is in controversy. Discussion of correct answer:Although there is such a thing as "rule" interpleader that brings no special rules of jurisdiction, Congress has also created statutory interpleader. That type of action has relaxed rules of jurisdiction. All that is necessary is minimal diversity among the claimants and at least $500 in controversy. Here, the claimants are all diverse from one another. The lawyer's citizenship does not matter, because the lawyer is merely a stakeholder and is not a claimant. The amount in controversy is the value of the antique watch and exceeds $500.

At a woman's trial for bank robbery, the prosecutor has called a private security guard for the bank who has testified, without objection, that while he was on a coffee break, the woman's brother rushed up to him and said, "Come quickly! My sister is robbing the bank!" The woman now seeks to call a witness to testify that the brother later told the witness, "I got my sister into trouble by telling a security guard that she was robbing the bank, but now I realize I was mistaken." The brother is unavailable to testify. Is the witness's testimony admissible?

Yes, but only as an inconsistent statement to impeach the brother's credibility. Discussion of correct answer:It is ordinarily true that a witness impeached with a prior inconsistent statement must be given an opportunity to explain or deny the statement. That is not possible, however, when a hearsay declarant is not produced at trial. Therefore Rule 806 provides that the ordinary requirement of a "fair opportunity to explain or deny" is not applicable to hearsay declarants who are being impeached with prior inconsistent statements. The inconsistent statement is probative of the brother's credibility, and Rule 806 permits such impeachment.

A driver and his passenger were driving to work one morning. As they were traveling at a speed of 20 miles per hour (which was within the posted speed limit), the passenger suddenly pointed to an overturned vehicle along the side of the highway and said, "Look at that car upside down." The driver turned to look at the overturned vehicle. As he was looking toward the side of the road, the driver failed to see an abandoned vehicle with a flat tire in the highway about 200 feet in front of his oncoming auto. Seconds later, the driver crashed into the rear of the abandoned auto and was injured. If the driver asserts a claim against the owner of the abandoned auto, will the driver recover?

Yes, but only for a portion of damages, because the abandoned auto was in plain view. Discussion of correct answer:It is important to note that the MBE instructions for Torts state as follows, "Examinees should assume that joint and several liability, with pure comparative negligence, is the relevant rule unless otherwise indicated." Therefore, you must assume that pure comparative negligence applies to this fact pattern, because you are not told otherwise. As a general rule, comparative negligence statutes have the effect of apportioning damages based on the parties' respective degrees of fault. For example, if the defendant's fault is found to be twice as great as that of the plaintiff, the latter will recover two-thirds of his damages, and himself bear the remainder of his loss. In the present case, the driver was negligent in failing to keep proper lookout while driving. By the same token, the defendant was also at fault, because he left an abandoned auto in the middle of the highway. As a consequence, the plaintiff's recovery will be diminished in proportion to his negligence.

An experienced mechanic was hired by a company to replace a defective ignition switch on a motorized three-wheel forklift. The forklift had been purchased by the company two years ago from a distributor of heavy-duty factory equipment. The ignition switch that the mechanic was working on was located near the two rear wheels. After removing the old defective switch, the mechanic then attempted to insert the new ignition switch, which he had purchased from a parts distributor. When the new ignition switch popped out of the socket, the mechanic tried to reinsert it. As he did so, the forklift engine suddenly started and dragged the mechanic underneath the rear axle, injuring him. The new ignition switch that the mechanic was installing had been specifically designed to fit the model he was fixing. If the mechanic brings a claim against the manufacturer alleging that the new ignition switch was defective and unreasonably dangerous, will he recover?

Yes, if such an accident was foreseeable and the manufacturer failed to warn purchasers of the danger. Discussion of correct answer:Most important, students must recognize that this question involves strict products liability. The mechanic is alleging that the ignition switch was "defective" and "unreasonably dangerous"--the required elements for strict products liability. As a general rule, the only defense in strict liability is assumption of risk, not contributory negligence. By process of elimination, this is the best answer choice.

A businessman is the owner of a small winery. For several years, he has been dumping grape sludge, the waste product of the fermenting process, in a pit about 100 yards from the neighbor's property. The sludge continues to ferment, giving off a highly pungent and offensive odor. The neighbor finds it impossible to enjoy any activities in her yard because of the odor. If the neighbor's four-year-old daughter ran onto the businessman's land while chasing a butterfly, fell into the sludge pit, and was injured, would she recover?

Yes, if the businessman knew young children had entered his land before and were likely to be injured. Discussion of correct answer:The neighbor's daughter was injured when she trespassed on the businessman's land. However, if the businessman knew or had reason to know children were likely to trespass and knew or had reason to know that there were conditions on the premises dangerous to children, he may owe the trespassing children a duty of reasonable care. Other factors to examine are whether the children, because of their age, could not recognize or appreciate the danger, and whether the burden of correcting the danger was small in relation to the risk to the children. Because this answer choice states two of these requirements, and the other alternatives are more clearly incorrect, this answer choice is the best answer.

A defendant has been charged with selling cocaine to a police informant. At trial, the alleged cocaine no longer exists, and the only evidence that the substance sold was cocaine is the informant's testimony that it tasted like cocaine and gave her a cocaine-like sensation. The informant has no formal training in identifying controlled substances. Should the court admit the informant's opinion testimony that the substance was cocaine?

Yes, if the court determines that the informant has sufficient knowledge and experience to identify cocaine. Discussion of correct answer:A witness who is qualified as an expert by knowledge, skill, experience, training, or education may testify in the form of an opinion or otherwise if [Fed. R. Evid. 702]: (1) the expert's scientific, technical, or other specialized knowledge will help the trier of fact to understand the evidence or to determine a fact in issue; (2) the testimony is based on sufficient facts or data; (3) the testimony is the product of reliable principles and methods; and (4) the expert has reliably applied the principles and methods to the facts of the case. An expert is defined broadly and may also be a skilled witness, drawing upon all forms of specialized knowledge. Here, if the police informant has sufficient knowledge and experience to identify cocaine, she may testify as an expert or skilled witness. She is not required to have formal training in order to testify as an expert. Therefore, if the police informant has sufficient knowledge and experience to identify cocaine, then her testimony should be admitted into evidence as expert testimony. Question of law for a judge, not jury


Conjuntos de estudio relacionados

BHM - Exam 3 (Questions from Book)

View Set

Health information management 8,9,10

View Set

Naming Compounds (methyl, ethyl, etc)

View Set

Chapter 49: Nursing Care of a Family when a Child has a Neurologic Disorder

View Set

AUD - Identifying and Assessing the Risk of Mat Mis

View Set